Examen de preparacion

January 3, 2018 | Author: LuisErnestoSalinas | Category: Peptic Ulcer, Hypothyroidism, Major Depressive Disorder, Thyroid Stimulating Hormone, Thyroid
Share Embed Donate


Short Description

examen diagnostico de curso lasalle para preparacion para el ENARM...

Description

Universidad La Salle. Facultad Mexicana de Medicina. Curso de Extensión Universitaria para la Preparación del Examen Nacional para Aspirantes a Residencias Médicas. Examen Diagnóstico.

Nombre: Examen diagnóstico. Número de intentos: 3. Vigencia: 5 de Febrero del 2014. Horario: 9:00 a.m. a 9:00 p.m. Programar aleatorio. (5 bloques con 20 preguntas cada uno).

1.- Femenino de 27 años de edad con deseo de un embarazo, antecedentes de G3, A2, P1, se le realiza una histerosalpingografía, se constata que existe un síndrome de Asherman. Ello significa que se trata de:

a) b) c) d)

Útero bicorne Sinequias uterinas Endometriosis en la trompa Insuficiencia istmico cervical

El síndrome de Asherman es una enfermedad ginecológica rara que se caracteriza por la presencia de sinequias (adherencias) intrauterinas que pueden ocasionar amenorrea (ausencia de períodos menstruales regulares) e infertilidad.

En 1894 Heinrich Fritsch describe por primera vez la presencia de sinequias intrauterinas de tipo postraumático, en una paciente que desarrolló una amenorrea secundaria a un curetaje. Posteriormente en 1927 Bass informó de veinte casos de atresia (oclusión de una abertura natural) cervical tras abortos inducidos, pero no fue hasta 1948, cuando Joseph G. Asherman recopiló la información hasta entonces existente y acuñó el nombre con el que se conoce actualmente a la enfermedad.

Asherman describió originalmente dos tipos diferentes de amenorrea secundaria, en función de su etiología (estudio de las causas de las enfermedades): la amenorrea traumática atrética, debida a estenosis del orificio cervical interno y la amenorrea debida a adherencias intrauterinas. Posteriormente ambas entidades se agruparon en una única entidad bajo el nombre de síndrome de Asherman.

Suele presentarse en mayor proporción tras dilataciones y curetajes uterinos de repetición y sobre todo si se realizan durante el embarazo o si existe infección uterina en el momento en el que se realizan estas intervenciones.

Las adherencias intrauterinas pueden producirse debido a cualquier factor que lleve a una destrucción de las paredes del miometrio (capa muscular de la pared del útero). Sin embargo, hay que distinguir entre factores predisponentes, siendo el principal de ellos el embarazo y factores causales, entre los que se encuentran: traumatismos uterinos, intervenciones quirúrgicas que afecten al útero, agentes físicos o químicos e infecciones uterinas por tuberculosis o esquistosomiasis. En cualquier caso, el factor más importante es el trauma uterino en el momento del parto o el puerperio.

El cuadro clínico es muy variable y las manifestaciones clínicas varían con el grado de oclusión de la cavidad uterina y la severidad de las adherencias, pudiendo presentarse: esterilidad cuando la oclusión de la cavidad uterina incluye porciones proximales (más cerca de un centro, tronco o línea media) de las trompas de Falopio o cuando las adherencias impiden la nidación del huevo; las pacientes presentan con frecuencia amenorrea, oligomenorrea (disminución de la frecuencia de las menstruaciones), dismenorrea (menstruación dolorosa) y abortos repetidos. Hysteroscopic treatment of severe Asherman's syndrome and subsequent fertility. Capella-Allouc S; Hum Reprod.

2.- Se trata de femenino de 31 años de edad, segunda gesta a término sin anormalidades en el transcurso de éste. Inicia trabajo de parto de forma espontánea, con evolución normal hasta que se rompe la bolsa, con una dilatación de 4 cm. A partir de entonces, comienza con hemorragia de sangre roja, en moderada cantidad y aparecen signos de sufrimiento fetal agudo. El estado general de la mujer es bueno y la dinámica uterina es normal. Este cuadro corresponde a: a) b) c) d)

Abruptio placentae. Rotura uterina. Rotura de vasa previa. Placenta previa central

La vasa previa es una condición de alto riesgo obstétrico en la cual vasos fetales o placentarios cruzan el segmento uterino por debajo de presentación. Estos vasos están desprotegidos de gelatina de Wharton o tejido placentario, lo que los hace altamente vulnerables y susceptibles de ruptura o laceraciones en cualquier período del embarazo, principalmente en el momento del parto. También es frecuente la compresión

de estos vasos, especialmente durante el tercer trimestre de la gestación, lo que puede condicionar asfixia y muerte fetal.

Esta condición ocurre como resultado de que vasos velamentosos cruzan por el segmento uterino debido a una inserción velamentosa del cordón, situación en la cual el cordón umbilical se inserta en las membranas ovulares en vez del tejido placentario (vasa previa tipo I), o por el cruce de vasos fetalesentre uno o más lóbulos accesorios de la placenta (vasa previa tipo II) (Figura 1).

Figura 1 . A, Vasa previa tipo I, debida a inserción velamentosa. B, Vasa previa tipo II, debida a cotiledones aberrrantes. Reproducido de Daly-Jones y cols. Ultrasound 2008.

Vasa previa se puede presentar si existe alguna (o ninguna) de las siguientes condiciones: placenta baja (que puede ser causa de abortos previos seguidos por legrado o por operaciones uterinas, que provocan cicatrices en el útero), placenta bilobada o de lóbulo succensuriado, embarazos resultado de fertilización in vitro, o embarazos múltiples (5-6). El sangrado por vasa previa no es doloroso. Otros sangrados por complicaciones o por nacimiento no necesariamente son sin dolor.

Referencias: 1. Oyalese Y, Smulian JC. Placenta previa, placenta acreta, and vasa previa. Obstet Gynecol 2006; 107: 927-941. 2. Oyalesse KO, Turner M, Less C, Campbell S. Vasa previa: an avoidable obstetric tragedy. Obstet Gynecol Surv 1999; 54: 138-145. 3. Sepúlveda W, Sebire NJ, Harris R, Nyberg DA. The placenta, umbilical cord, and membranas. In Diagnostic Imaging of Fetal Anomalies, Nyberg DA, MaGahan JP, Pretorius DH, Pilu G (eds). Philadelphia, PA: Lippicont Williams & Wilkins 2003, 85-132. 4. Daly-Jones E, John A, Leahy A, McKenna C, Sepulveda W. Vasa praevia; a preventable tragedy. Ultrasound 2006; 16: 8-14.

5. Derbala Y, Grochal F, Jeanty P. Vasa previa. J Prenat Med 2007; 1: 2-13. 6. Fung TY, Lau TK. Poor perinatal outcome associated with vasa previa. It is preventable? A report of three cases and review of the literature. Ultrasound Obstet Gynecol 1998; 12: 430-433. 7. Robert JA, Sepulveda W. Fetal exsanguination from ruptured vasa previa: still a catastrophic event in modern obstetrics. J Obstet Gynaecol 2003; 23: 574. 8. Cordero DR, Helfgott AW, Landy HJ, et al. A non-hemorrhagic manifestation of vasa previa: a clinicopathologic case report. Obstet Gynecol 1993; 82: 698-700. 9. Schachter M, Tovbin Y, Arieli S, et al. In vitro fertilization as a risk factor for vasa previa. Fertil Steril 2002; 78: 642-643.

3.- Masculino en edad preescolar. Acude al servicio de consulta externa. Antecedentes: historia de geofagia. Hace 2 semanas presenta tos húmeda y "silbido del pecho". E.f.: temp. :37.2 ºc., mala higiene personal, tórax con sibilancias espiratorias bilaterales. Se reportan labs. BH con anemia y eosinofilia. El diagnóstico más probable en éste paciente es:

a) b) c) d)

Ascaris. Sx de loeffer Tenias. Tricocefalos. Enterobius.

En 1932 Loeffler describe un síndrome caracterizado por sintomatología respiratoria compatible con "pulmón sibilante", radiografía de tórax con infiltrado intersticial bilateral, difuso, abigarrado y transitorio compatible con neumonitis asociada de modo obligatorio a eosinofilia periférica. Este síndrome clásicamente se presenta cuando existe migración parasitaria alveolo capilar pulmonar y es causado por larvas de parásitos con ciclo hístico tisular como Ascaris l, Strongyloides s, Ancylostoma duodenale, Necator americanus y Filarias (1-2-3) excepcionalmente está asociado al síndrome de larva migrans cutánea (1). La invasión por larvas de helmintos puede originar infiltración pulmonar, con marcada dificultad respiratoria por el espasmo bronco-alveolar como respuesta a la invasión parasitaria (pulmón sibilante), cuadro pulmonar inflamatorio transitorio, con respuesta alérgica local que explica y exige para su diagnóstico, una intensa eosinofilía periférica(5-67) Este síndrome es más frecuente en personas que se infectan por primera vez o que viven en zonas no endémicas; por hipersensibilidad a las larvas presentándose como una neumonía eosinofílica, aguda, benigna y con infiltrados pulmonares migratorios y transitorios(7), no existiendo restricción para ninguna edad, describiéndose aun en neonatos ya desde el año 2001 con cuadro clínico similar al de niños de mayor edad (4).

Cuadro I. Principales causas de eosinofilia Parasitosis Helmintiasis con fase tisular

Infecciones micóticas Coccidioidomicosis Enfermedades alérgicas Asma bronquial Rinitis alérgica Alergia a medicamentos Edema angioneurótico Fiebre del heno Neoplasias Enfermedad de Hodking Carcinomas Sarcomas Tumores óseos Tumores de ovario Leucemia mieloide crónica Leucemia eosinofílica Enfermedades gastrointestinales Gastritis eosinofílica Enfermedades inmunológicas Artritis reumatoide Vasculitis Síndrome de Wiskott-Aldrich Síndrome de Hiper-IgE Déficit selectivo de IgA Injerto contra huésped Enfermedades respiratorias Aspergilosis broncopulmonar alérgica Sinusitis aspergilar alérgica Neumonía crónica eosinófila Intoxicaciones Intoxicación con fósforo Inhalación de humos Enfermedades dermatológicas Urticaria aguda Pénfigo Penfigoide ampolloso Síndrome de Wells Dermatitis atópica Hiperplasia angiolinfoide con eosinofilia Herpes gestationis Convalecencia de infecciones Escarlatina Fiebre reumática Otros Eosinofilia post-irradiación Síndrome hipereosinofílico REFERENCIAS

1.- Del Giudice P, Desalvador F, Bernard E, Caumes E, Vandenbos F, et al. 2002. Löffler's syndrome and cutaneous larva migrans: a rare association. British J Dermatol 2002; 147: 385- 7. 2. Botero D, Restrepo M. Parasitosis Humanas. 3° Ed., Medellín Colombia: Corporación para Investigaciones Biológicas 1998. 3.- Noemí I, Atias A. Eosinofilia y parasitosis. En Atias A. Parasitologia Médica. Mediterraneo, Santiago-Chile. 2000. 4.- Fujimura J, Murakami Y, Tsuda A, Chiba T, Migita M, Fukunaga Y. 2001.A neonate with Loeffler syndrome.Journal of Perinatology 2001; 21: 207-8. 5.- Hunninghake GW, RichersonHB. Neumonitis por hipersensibilidad y neumonias eosinófilas. En Harrison TR. Principios de Medicina Interna. 15°Ed. McGraw-Hill -Interamericana de España, Madrid- España. 2001. 6.- Chusid MJ. Eosinophilia in childhood. Immunol and Allergy Clinics North America 1999; 19: 327-46. 7.- Rothenberg ME. Eosinophilia N Engl J Med 1998; 338: 1592. 8.- Noemi IH. Eosinofilia y parasitosis. Rev. Chil. Pediatr. 1999; 70: 1-7. 9.- Hotez PJ, Broker S, Bethony JM, Bottazzi ME, Loukas A. Hookworm Infection. N Engl. J Med 2004; 351(8): 799-808. 10.- Yilderan A, Ikinciogullari A. In the light of recent advances: eosinophil, eosinophilia and idiopathic hypereosinophilic syndrome. Turk Haematol 2005; 22(3): 107-16.

4.- Femenino de 3 meses de edad que hace tres días inicia con rinorrea hialina, tos y estornudos. El día de ayer presenta taquipnea con 56 rpm, tiraje intercostal con aleteo nasal. A su ingreso se observan distrés respiratorio con sibilancias inspiratorias y espiratorias y algunos crepitantes bilaterales. Una Rx de tórax muestra hiperinsuflacción bilateral con una atelectasia laminar y corazón pequeño. Presenta: pH 7,24; pCO2: 58 mmg; CO3H: 21 mEq/L. ¿El diagnóstico más probable es? a) b) c) d)

Bronquiolitis Bronconeumonía bilateral. Crisis asmática de origen infeccioso. Neumonitis intersticial.

La bronquiolitis es la enfermedad del tracto respiratorio inferior más frecuente en los dos primeros años de vida. Aunque su mortalidad es baja, su elevada morbilidad origina una gran demanda asistencial y causa un importante número de ingresos hospitalarios. Se presenta de forma epidémica durante el invierno y principio de la primavera. Su etiología es viral, y el virus respiratorio sincitial (VRS) es el agente que con más frecuencia se aísla. El diagnóstico es eminentemente clínico. En la mayoría de los casos, cursa de forma benigna y autolimitada; las formas más graves se presentan en lactantes más pequeños y en los que tienen algún factor de riesgo asociado, en estos casos la tasa de hospitalización es alta.

El cuadro comienza como un catarro de vías altas, con estornudo, tos, rinorrea y en ocasiones fiebre; y en el transcurso de 2 ó 3 días se intensifica la tos, aparecen los síntomas de obstrucción de la vía respiratoria respiratoria inferior con aumento del trabajo respiratorio, taquipnea e irritabilidad.

En los casos más graves, la dificultad respiratoria es marcada y presenta rechazo de las tomas de alimento y postración. La apnea puede ser la primera manifestación de bronquiolitis en lactantes pequeños. Dada la evolución del curso clínico de la bronquiolitis, se recomienda reevaluar a las 24-48 horas a todos los lactantes menores de 12 meses que, en ambiente epidémico consulten por un catarro de vías altas. Además, es conveniente advertir a los padres de la necesidad de consultar ante la presencia de dificultad respiratoria, agitación, mal color, vómitos o rechazo de la alimentación. A la exploración física, podemos encontrar retracciones costales (tiraje subcostal e intercostal) y la auscultación pulmonar puede ser normal, aunque en la mayoría de los casos presenta espiración alargada, roncus, sibilantes y crepitantes bilaterales. Los sibilantes pueden escucharse con el oído desnudo, sin ayuda del fonendoscopio. No existe un patrón radiológico típico en la bronquiolitis. Los hallazgos radiológicos más comunes son: el atrapamiento aéreo, engrosamientos peribronquiales, infiltrados intersticiales y atelectasias laminares o segmentarias. El hemograma es inespecífico y sólo estaría indicado realizarlo en aquellos casos en los que sospechemos alguna complicación. Para el diagnóstico de la hipoxemia se recomienda la pulsioximetría transcutánea, ya que tiene muy buena correlación con la PaO2 sanguínea. Se considera hipoxia leve del 96-98%; moderada, del 93-95%, y grave, menos de 93%. González Caballero D, González Pérez- Yarza E. Bronquiolitis aguda: bases para un protocolo racional. An Esp Pediatría 2001; 55 (4): 355-64.

5.- Se trata de paciente femenino en quien se sospecha la presencia de insuficiencia suprarrenal primaria, en éste caso las alteraciones de laboratorio que podemos encontrar son: a) Hipercalcemia e hipernatremia b) Hipernatremia e hiperglucemia c) Hipocalcemia y leucopenia d) Hiperkalemia e hipoglucemia

En la insuficiencia suprarrenal primaria se observa disminución en los niveles séricos de sodio, cloruro y bicarbonato, mientras que el potasio sérico se eleva. Esta se debe a los efectos combinados de déficit de aldosterona, reducción del filtrado glomerular y acidosis. Y sobre todo durante el estrés puede haber disminución en los niveles de glucosa.



Datos de laboratorio

1.

Hiponatremia: los niveles séricos bajos de sodio se debe a su pérdida por la orina por déficit de aldosterona y al desplazamiento del sodio hacia el compartimento intracelular. Esta pérdida de sodio extravascular reduce el volumen plasmático y acentúa la hipotensión.

2. Hiperkalemia: aumento de los niveles séricos de potasio. Se debe a los efectos combinados del déficit de aldosterona, la reducción del filtrado glomerular y la acidosis. 3. Hipocortisolemia: los niveles de cortisol y aldosterona son bajos y no aumentan con la administración de ACTH. 4. Hipercalcemia: aumento de los niveles séricos de calcio. Ocurre en un 10-20% de los pacientes de causa desconocida. 5. Cambios electrocardiográficos: suelen ser inespecíficos, aunque con lentificación generalizada del trazado. 6. Hemograma: puede haber anemia normocítica, linfocitosis relativa y eosinofilia moderada. 7. Prueba de estimulación de ACTH: prueba principal que confirma el diagnóstico de insuficiencia suprarrenal, al evaluar la capacidad de las suprarrenales para producir esteroides, que suelen estar ausentes o disminuidos tanto en sangre como en orina tras la estimulación de ACTH. 8. Determinación de la ACTH: en la insuficiencia suprarrenal primaria o Enfermedad de Addison, la ACTH y sus péptidos afines, están elevados en plasma ante la pérdida del mecanismo de retroalimentación del eje hipotálamo-hipófisariosuprarrenal. 9. hipertermia: la hormona del hipotálamo no controla la homeostasis

Williams GH, Dluhy RG. Enfermedades de la corteza suprarrenal. En Jameson JL (ed): Harrison. Endocrinología. 1a ed. Madrid. MacGraw-Hill España, 2006: 137-138.

6.- Femenino de 39 años inicia recientemente con fatiga, somnolencia, piel seca, estreñimiento y aumento de peso de 5 kg. Su tiroides está firme y tiene el doble del tamaño normal. ¿Cuál de las siguientes pruebas de laboratorio confirma el presunto diagnóstico de hipotiroidismo?

a) b) c) d)

Tiroxina sérica (T4) Hormona estimulante de tiroides (TSH) en suero Triyodotironina sérica (T3) Captación de resina T3

Hipotiroidismo La instauración es habitualmente lenta y progresiva. Los síntomas se relacionan con una disminución en la actividad funcional de todos los sistemas del organismo. Los más clásicos son cansancio, intolerancia al frío (carácter muy friolero), apatía e indiferencia, depresión, disminución de memoria y de la capacidad de concentración mental, piel seca, cabello seco y quebradizo, fragilidad de uñas, palidez de piel, aumento de peso, estreñimiento pertinaz y somnolencia excesiva. En situaciones extremas puede evolucionar hacia la insuficiencia cardiaca, la hinchazón generalizada (mixedema), insuficiencia respiratoria y abocar al coma mixedematoso con pérdida de conocimiento que conlleva un alto grado de mortalidad.

Al igual que el resto de enfermedades del tiroides, el hipotiroidismo es más frecuente en el sexo femenino. Es a partir de los 40-50 años cuando las mujeres tienden a desarrollar con más frecuencia hipotiroidismo de causa autoinmune (tiroiditis de Hashimoto). El periodo postparto es igualmente propenso a la aparición de este problema. La cirugía de tiroides y la aplicación de yodo radioactivo representan situaciones de riesgo para el desarrollo de hipotiroidismo, lo que obliga a controlar evolutivamente la función tiroidea en estos casos.Los recién nacidos de madres hipertiroideas, hayan recibido o no tratamiento antitiroideo durante la gestación, deben ser evaluados en este sentido. Las personas en las que se detectan anticuerpos antitiroideos (antimicrosomales, antitiroglobulina) tienden a desarrollar con el tiempo alteraciones de la función tiroidea, por lo que deben ser evaluados crónicamente de forma periódica. La determinación de TSH es el parámetro más sensible para el diagnóstico del hipotiroidismo. Su elevación es indicativa de que la función del tiroides es insuficiente. Este fenómeno se produce antes de que comiencen a descender en la sangre las concentraciones de hormonas tiroideas. Generalmente, en el hipotiroidismo establecido, además de la elevación de TSH, se produce un descenso de T4. El nivel de T3 con frecuencia se encuentra dentro de la normalidad. Así pues, cuando aparecen síntomas sugestivos, el médico solicitará una determinación de TSH que es el mejor método para descartar que exista hipotiroidismo. Puede acompañarse de una determinación de T4 y de anticuerpos antitiroideos si se desea conocer si la causa se debe a fenómenos de autoinmunidad. En los casos de hipotiroidismo secundario debido a disminución de la secreción de TSH por parte de la hipófisis, el diagnóstico se basa en confirmar concentraciones disminuidas de T4 y TSH en la sangre. Cuando la elevación de TSH se acompaña de niveles normales de T4 la condición es conocida con el nombre de hipotiroidismo subclínico. Si existe bocio puede ser conveniente realizar una ecografía tiroidea. Cuando existe sospecha de alteraciones en el desarrollo de la glándula o de deficiencia enzimática, puede ser útil obtener una gammagrafía tiroidea. Si se confirma un diagnóstico de hipotiroidismo de causa autoinmune, es habitual evaluar la asociación de alteraciones en otras glándulas como las suprarrenales, paratiroides o gónadas. REFERENCIAS (1.) Anderson R, Harnes J. 1975. Thyroid hormones secretion rates in growing and mature goats. J Anim Sci 40: 11301135. (2.) Anke M, Henning A, Grun M, Partschefeld M, Groppel B. 1977. Der einluss des mangan, zink, kupfer, jod, selen, molybdan und nickelmangels aauf die fortpflanzuggsleistung des wiederkauers. Mathem Natur Reihe (Leipzig) 26: 283-292. (3.) Balbuena O. 2003. Nutrición Mineral del Ganado. Sitio Argentino de Producción Animal: 1-5, www.produccionanimal. com.ar. (4.) Beckett GJ, Beddows SE, Morrice PC, Nicol F, Arthur JR. 1987. Inhibition of hepatic deiodination of thyroxine is caused by selenium deficiency in rats. Biochem J 248: 443447.

(5.) Brem JJ, Pochon DO, Roux JP, Trulls H. 1998. Exploración diagnóstica de la función

tiroidea en ovinos. Rev Vet 8/9: 23-26. (6.) Castillo V. 2001. Cambios de la función tiroidea en cachorros alimentados con dietas comerciales con alto contenido de yodo. On line: http://www.idealibrary.com

7.- Masculino de 54 años que acude con resultados de manometría esofágica los que muestran un aumento de la presión basal del esfínter esofágico inferior (EEI) junto a una disminución franca de su relajación con la deglución, el diagnóstico más probable es: a) Esclerodermia con afectación esofágica. b) Enfermedad por reflujo gastroesofágico c) Espasmo esofágico difuso. d) Acalasia.

Diagnóstico de acalasia Aspecto radiográfico Un esofagograma puede mostrar la disminución de peristalsis, la dilatación del esófago proximal y el estrechamiento del esófago en su parte inferior. El paciente se traga una solución de bario, con fluoroscopia continua que son grabaciones de rayos X para observar el flujo del fluido a lo largo del esófago, sin que se observe el movimiento peristáltico normal del esófago. Hay un agudo estrechamiento en el esfínter esofágico inferior y reducción del diámetro en la unión gastro-esofágica. La imagen que proyecta se denomina clásicamente «en pico de loro» o «en cola de ratón». Por encima de la reducción, el esófago a menudo se observa con una dilatación de diversos grados a medida que poco a poco se va estirando en el tiempo. Por la falta de movimientos peristálticos, se suele observar en la radiografía un margen entre aire y líquido. Manometría esofágica Debido a su sensibilidad, el diagnóstico es confirmado por medio de una manometría esofágica, que mide las presiones del esófago mediante una sonda nasoesofágica y permite comparar las presiones en situación basal y durante la deglución.4 Se inserta un tubo delgado a través de la nariz, y se le instruye al paciente a deglutir varias veces. La sonda mide las contracciones musculares en diferentes partes del esófago durante el acto de la deglución. La manometría revela la falla del EEI para relajarse con cada deglución y la falta de peristaltismo funcional del músculo liso en el esófago. Para descartar complicaciones se suele acudir a una endoscopia digestiva alta.

Esquema manométrico de acalasia demostrando contracciones aperistálticas, un aumento de la presión intraesofágica y el fallo de la relajación del esfínter esofágico inferior. Debido a la similitud en los síntomas, la acalasia se puede confundir con trastornos más comunes, tales como la enfermedad de reflujo gastroesofágico, la hernia de hiato, e incluso trastornos psicosomáticos. REFERENCIAS BIBLIOGRÁFICAS 1.

García Gutiérrez A. Acalasia de esófago. http://www.sld.cu/galerias/pdf/uvs/cirured/acalasia..pdf Consultado Mayo 27, 2006.

Disponible

en:

Fareras Rozman, et al. Acalasia esofágica.Tratado de Medicina Interna. 15 Edición. España: Ediciones Harcourt; 2003. Sección 2. Cap 20.p.354-9.

8.- Ingresa al servicio de urgencias masculino de 47 años con hemorragia digestiva alta. No hay antecedentes de consumo de AINE. La endoscopia revela úlcera gástrica en incisura angularis con un punto de hematina y mínimos restos de sangre oscura en el estómago. Se realizan biopsias del margen de la úlcera y una biopsia antral para prueba rápida de ureasa positiva. La actitud más correcta ante este paciente es:

a) Iniciar con sucralfato y Ranitidina 150 mg/día, tratamiento anti-Helicobacter pylori si la histología confirma la presencia del germen. b) Esclerosis endoscópica de la úlcera seguida de tratamiento con omeprazol, 20 mg/día durante 28 días. c) Iniciar tratamiento con Omeprazol 40mg/día durante 1 mes. d) Se debe indicar anti-Helicobacter pylori durante 14 días, seguido de un antisecretor hasta que se confirme la erradicación del germen.

El Helicobacter pylori (HP) es una bacteria microaerófila, gramnegativa, de crecimiento lento y forma helicoidal con abundantes flagelos. Fue descubierta por dos médicos autralianos. Robin Warren y Barry Marshall; trabajando en colaboración, detectaron que este microorganismo se encontraba en casi todos los pacientes con inflamación gástrica, úlcera duodenal o gástrica. Basándose en estos resultados propusieron que HP estaba implicado en la etiología de estas enfermedades. Antes de 1982, se pensaba que la mayor causa de la úlcera péptica era el

estrés y el estilo de vida. Ahora se sabe que HP está implicado en más del 90% de las úlceras duodenales y hasta el 80% de las úlceras gástricas. Gracias a los descubrimientos de Marshall y Warren, la úlcera péptica no es una enfermedad crónica sino que puede ser curada con una pauta de tratamiento con antibióticos y con inhibidores de la secreción ácida. Afecta al 50 % de la población mundial, ha sido identificado como el agente causal de la úlcera péptica y se ha clasificado además como carcinógeno tipo I. Como resultado de su interferencia con la secreción de ácido por el estómago, esta bacteria es capaz de generar deficiencias en la absorción de nutrientes y vincularse con la aparición de manifestaciones carenciales o con el agente causal de enfermedades crónicas

El objetivo del tratamiento médico será promover la cicatrización de la úlcera para prevenir la recurrencia de la hemorragia. Así, la ranitidina puede prevenir la úlcera duodenal en pacientes tomadores de AINES, pero es ineficaz en la prevención de la ulcera gástrica. Por el contrario, la famotidina protege la mucosa gástrica contra tratamientos cortos de aspirina o naproxeno. Si se demuestra que existe infección por H.Pylori estará indicada su erradicación para evitar recurrencias. Amoxicilina + Klaritromicina OD+ esomeprazol o lanzoprazol Erradicación en 14 dias 100% De existir una complicación, como la hemorragia digestiva, los pacientes deberán ser internados, suprimir el aporte oral, efectuar el diagnóstico de hemorragia, determinar la cuantía de la pérdida hemática y realizar un adecuado control de la hemodinamia.

Lahaie RG, Gaudreau C. Helicobacter pylori antibiotic resistance: trends over time. Canadian Journal of Gastroenterology. 2000;14(10):895–899. Manes G, Balzano A, Iaquinto G, et al. Accuracy of the stool antigen test in the diagnosis of Helicobacter pylori infection before treatment and in patients on omeprazole therapy. Alimentary Pharmacology and Therapeutics. 2001;15(1):73–79. McManus TJ. Helicobacter pylori: an emerging infectious disease. Nurse Practitioner. 2000;25(8):42–46.

9.- Hombre de 30 años de edad. Refiere frecuente irritación conjuntival, comezón en el borde palpebral, aparición habitual de orzuelos y enrojecimineto crónico de la conjuntiva. A la exploración se observa, entre las pestañas y al tallarlas, descamación en forma de caspa, escasa secreción en los ángulos e hiperemia tarsal y conjuntival bulbar leves. El diagnóstico más probable es:

a) b) c) d)

Conjuntivitis bacteriana Conjuntivitis viral Blefaroconjuntivitis Conjuntivitis primaveral

Las blefaroconjuntivitis (blefaritis) son una de las patologías más frecuentemente encontradas en la población general. Consisten en infecciones crónicas de las glándulas sebáceas del borde libre del párpado, pudiendo ser:  Ant er ior es, que pr esent an escamas y anillos quer át icos alr ededor de las pest añas y generalmente son estafilocócicas.  Post er ior es, o meibomit is, son secundar ias a la inf ección de las glándulas de Meibomio; pueden ser tratadas con algún antibiótico tópico y/o sistémico y en ocasiones algún esteroide tópico, pero lo más importante en todos los casos son las medidas generales de aseo. PAC de Oftalmología Parte B libro 4

10.- Femenino de 78 años de edad con antecedente de insuficiencia cardiaca congestiva presenta angina de pecho. Sus medicamentos se ajustan con: furosemida, digoxina, nitroglicerina y potasio complementario. Poco después presenta cefaleas pulsátiles intermitentes que incapacitan a la paciente. Cuál de las siguientes medidas debe iniciarse para su control: a) Suspender la nitroglicerina b) Comenzar la administración de propranolol c) Comenzar la administración de ergotamina sublingual d) Realizar biopsia de arteria temporal

Allen R. M. MMS Medicina Interna. 5ª. Edición. National Medical Series. Mc. Graw Hill. 2006. (capítulo 11IV B 1 a (2), d (1) (a), 4 a-b). La nitroglicerina puede causar cefaleas "vasculares" terebrantes; por tanto, la más sencilla opción de tratamiento es suspender el preparado de nitroglicerina y utilizar, si es posible, un medicamento cardíaco alternativo. Siempre debe considerarse la arteritis temporal como posible causa de cefalea en pacientes mayores de 50 años. Favorecen este diagnóstico el aumento de la velocidad de eritrosedimentación; la claudicación mandibular; las artralgias y las mialgias; y una arteria temporal hipersensible e indurada. El propranolol es un antimigrañoso eficaz. Sin embargo, antes de prescribir estos fármacos deben descartarse los posibles factores que la precipitan. La ergotamina es un tratamiento abortivo eficaz para migraña, pero es un vasoconstrictor y no debe usarse en pacientes con angina de pecho. En la valoración de ancianos con cefalea de inicio reciente debe considerarse realizar una tomografía computadorizada del cerebro. En este caso, si es posible suprimir la cefalea al suspender la nitroglicerina, no es necesaria una CT.

11.- Hombre de 36 años con presencia de desarrollo de manchas lenticulares de color café, rosado y blanco, levemente descamativas, se diagnostica micosis crónica asintomática por Malassezia furfur. La localización anatómica más frecuente de esta patología es:

a) b) c) d)

Dorso de manos y pies. Codos y rodillas Cara y cuello. Pecho y espalda.

La pitiriasis versicolor (PV) es una micosis superficial crónica, caracterizada por máculas hipo o hiperpigmentadas levemente descamativas. El término “versicolor” se refiere al color variable de las lesiones que pueden ser café, rosadas o blancas. Esta enfermedad tiene distribución mundial y es uno de los desórdenes más comunes en la pigmentación de la piel. La mayor prevalencia y el mayor porcentaje de recurrencias se observan en áreas cálidas y húmedas. Eichstedt, en 1846, fue el primero en reconocer la naturaleza fúngica de la PV, al describir un hongo asociado a esta afección. Sin embargo, el género Malassezia, con Malassezia fufur (M. furfur) como especie tipo, fue creado por Baillon medio siglo después, en 1889, en honor a Luis Malassez. Pitiriasis versicolor, a veces denominada Tinea versicolor, es uno de los trastornos de pigmentación más comunes en el mundo. Es una micosis crónica, leve y usualmente asintomática. Se produce por el crecimiento de la levadura lipofílica Malassezia furfur en la capa más superficial de la piel o estrato córneo.

La infección se caracteriza por el desarrollo de manchas o máculas lenticulares de color café, rosado o blanco, levemente descamativas, en tronco y brazos.

BIBLIOGRAFÍA 1. Ashbee HR, Evans E.G. Inmunology of Diseases Associated with Malassezia species. Clin Microbiol Rew. 2002;21-57. 2. Crespo Erchiga V, Delgado Florencio V. Malassezia species in skin diseases. Curr Opinión Infect Dis, 2002, 15:133-142.

3. Guèho E, Boekhout T, Ashbee HR, Guillot J, Van Belckum A, Faergeman J. The role of Malassezia species in the ecology of human skin and as pathogen. Med Mycol 1998;36 (supp1):220-229. 4. Crespo Erchiga V, Ojeda Martos A, Vera Casaño A, Crespo Erchiga A, Sánchez Fajardo F, Guèho E. Mycology of pityriasis versicolor. J. Mycol. Med. 1999; 9:143-148. 5. Katoh T, Irimajiri J. Pityriasis versicolor and Malassezia folliculitis. Nippon Ishinkin Gakkai Zasshi. 1999;40:69-71. 6. Gupta AK, Batra R, Bluhm R, Boekhout T, Dawson T. Skin diseases associated with Malassezia species. J Am Acad Dermatol 2004; 51:785-798.

12.- A 7 year old girl who presented skin lesions such macules and papules that evolve into blisters within hours. The mother states that introduced upper respiratory infection 2 weeks ago. The diagnosis you do it? a) b) c) d)

Herpes Zoster. Rubella Scarlet fever Varicella

Varicela Manifestaciones clínicas: la infección primaria produce fiebre moderada, y un exantema vesicular genereralizado y pruriginoso. Las lesiones se observan en distintos estados (mácula, pápula, vesícula y costras) y duran alrededor de una semana. Es posible también apreciar vesículas o úlceras en la mucosa oral. La reactivación del virus es responsable del herpes zoster. Agente etiológico: virus varicella zoster (ADN) perteneciente a la familia Herpesviridae Epidemiología: los humanos son la única fuente de contagio para este agente. La transmisión es a través del contacto persona a persona y por la ruta respiratoria. Los brotes aparecen hacia fines del invierno y en primavera. El período de incubación varía entre 10 a 21 días, y usualmente es de dos semanas. El momento de mayor contagiosidad ocurre desde dos días antes de que el exantema aparezca, hasta que las lesiones se encuentren en etapa de costra. En niños inmunodeprimidos, los períodos de incubación, de contagiosidad y de erupción pueden ser más largos.

Diagnóstico: el diagnóstico es fundamentalmente clínico; sin embargo, existen situaciones de diagnóstico diferencial de lesiones vesiculares en el huésped inmunodeprimido, en que la

inmunofluorescencia directa permite hacer el diagnóstico rápido de infección por virus varicela y así, tomar decisiones terapéuticas precoces. Aislamiento del paciente hospitalizado: Aislamiento respiratorio y de contacto por al menos cinco días desde que comienza el exantema o hasta que todas las vesículas estén en estado de costra. Tratamiento: en el huésped inmunocompetente el manejo del paciente con infección aguda es sintomático. Si se requiere uso de antipiréticos se recomienda usar sólo paracetamol, por la posible riesgo de desarrollo del Síndrome de Reye con el uso de ácido acetilsalicílico. Debe prevenirse la sobreinfección bacteriana de las lesiones, evitando el prurito y el grataje, manteniendo las uñas cortas, baño diario y usando antihistamínicos. El uso de aciclovir está indicado en sujetos con riesgo de desarrollar complicaciones (inmunodeprimidos, adolescentes, adultos, pacientes con terapia crónica con salicílicos y esteroides) y debe iniciarse en las primeras 24 hrs del exantema. El uso de aciclovir puede también considerarse en el segundo caso intrafamiliar ya que en estos pacientes se ha observado una evolución más severa de la enfermedad.

Bibliografía Report of the Committee on Infectious Diseases, 25h edition, Red Book 2000. American Academy of Pediatrics. Fifth (human parvovirus) and sixth (herpesvirus 6) diseases. Koch WC. Curr Opin Infect Dis 2001, Jun; 14 (3): 343-356. Primary human herpesvirus 8 in immunocompetent children. Andreoni M, Sarmati L, Nicastri E, El Sawaf G, El Zalabani M, Uccella I, et al. JAMA 2002 Mar 13; 287 (10): 1295-300.

Varicella vaccine update. AAP. Pediatrics 2000, Jan 105: 136-141.

13.- Se trata de preescolar de 5 años de edad, con antecedente de neumonía por Pneumocystis c., la profilaxis de ésta patología en éste paciente, se encuentra indicada cuando las cifras de linfocitos ajustados son de: a) b) c) d)

< 1500 Células/µl < 500 Células/µl < 750 Células/µl < 200 Células/µl

La NPC en niños y en el embarazo La NPC es infección diagnóstica de SIDA en un alto porcentaje de niños, sobre todo en el primer año de vida. Los niños menores de 1 año con CD4+ por debajo de 1500//l t ienen un 90% de riesgo de padecerla. Las manifestaciones clínicas, el diagnóstico y el tratamiento no difieren de los del adulto. Para la prevención deben seguirse las siguientes recomendaciones: • • • •

• •

Se aconseja la profilaxis primaria en todos los niños menores de 1 año hijos de madre VIH+ La profilaxis debería comenzar a las 4-6 semanas de vida y suspenderse en caso de que posteriormente no resulten estar infectados. Los niños infectados y aquellos cuyo estado de infección por VIH se desconozca deberán seguir recibiendo tratamiento profiláctico durante el primer año de vida. En los mayores de esta edad se considerará continuar con la profilaxis atendiendo al recuento de linfocitos CD4+ según la edad: de 1 a 5 años, cuando el recuento sea < 500 células/l o el por cent aj e < 15%; en niños de 6 a 12 años, cuando el r ecuent o sea inferior a 200 células/ l o el por cent aj e < 15%. No se ha estudiado la seguridad de interrumpir la profilaxis en niños infectados con el VIH y sometidos a tratamiento antirretroviral. Los niños con un historial que incluya episodios de NPC deberán ser tratados de por vida con quimioprofilaxis para evitar casos de recurrencia.

NCP en el embarazo Como ya se ha dicho, la quimioprofilaxis anti-NPC debe administrarse a las mujeres embarazadas al igual que al resto de adultos y adolescentes. En este caso, el agente profiláctico recomendado sigue siendo el cotrimoxazol, con dapsona como alternativa. Debido a la posibilidad teórica de una posible teratogenicidad asociada a la exposición a los fármacos durante el primer trimestre de gestación, durante dicho período puede considerarse la alternativa de pentamidina en aerosol debido a que dicho agente no se absorbe sistémicamente por lo que el feto en desarrollo no sufre exposición al fármaco.

Lectura recomendada: Neumonía por Pneumocystis carinii en niños infectados por el virus de inmunodeficiencia humana (VIH). Gac Med Mex 2004; 140 (1): 59-70

14.- Femenino de 20 años la cual refiere que desde hace varios meses presenta astenia, cansancio, pérdida de apetito y dificultades para concentrarse en los estudios. Al interrogatorio refiere que a perdido interés en los estudios, frecuenta menos a sus amigos, con pesimismo en la mayoría de sus actividades. ¿Cuál es el diagnóstico más probable?

a) Anorexia nerviosa b) Trastorno de ansiedad. c) Distimia. d) Depresión mayor

LA DISTIMIA La distimia es un estado de ánimo crónicamente deprimido, menos grave que la depresión y que no cumple los criterios para una depresión mayor, o lo hace sólo en períodos muy cortos. Su evolución suele ser de más de dos años. Se caracteriza por un abatimiento prolongado del estado de ánimo en que el sujeto distímico se describe a sí mismo como “triste” o “desanimado”, perdiendo el interés por las cosas y viéndose a menudo como inútil y poco interesante. Posee síntomas persistentes o intermitentes, de intensidad más leve comparación a la depresión mayor. Aiskal (1983) la define como “mal humor” y se caracteriza porque “el individuo está habitualmente triste, introvertido, melancólico, excesivamente consciente, incapaz de alegría y preocupado por su insuficiencia personal”. Los criterios de diagnóstico de Distimia son los que a continuación se detallan: A. Estado de ánimo crónicamente depresivo la mayor parte del día de la mayoría de los días, manifestado por el sujeto u observado por los demás, durante al menos 2 años. Nota:

En los niños y adolescentes el estado de ánimo puede ser irritable y la duración debe ser de al menos 1 año.

B. Presencia, mientras está deprimido, de dos (o más) de los siguientes síntomas: 1. 2. 3. 4. 5. 6.

Pérdida o aumento de apetito Insomnio o hipersomnia Falta de energía o fatiga Baja autoestima Dificultades para concentrarse o para tomar decisiones Sentimientos de desesperanza

C. Durante el período de 2 años (1 año en niños y adolescentes) de la alteración, el sujeto no ha estado sin síntomas de los Criterios A y B durante más de 2 meses seguidos. D. No ha habido ningún episodio depresivo mayor durante los primeros 2 años de la alteración (1 año para niños y adolescentes); por ejemplo, la alteración no se explica mejor por la presencia de un trastorno depresivo mayor crónico o un trastorno depresivo mayor, en remisión parcial. Nota: Antes de la aparición del episodio distímico pudo haber un episodio depresivo mayor previo que ha remitido totalmente (ningún signo o síntoma significativos durante 2 meses). Además, tras los primeros 2 años (1 año en niños y adolescentes) de trastorno distímico, puede haber episodios de trastorno depresivo mayor superpuestos, en cuyo caso cabe realizar ambos diagnósticos si se cumplen los criterios para un episodio depresivo mayor. E. Nunca ha habido un episodio maníaco, un episodio mixto o un episodio hipomaníaco y nunca se han cumplido los criterios para el trastorno ciclotímíco. F. La alteración no aparece exclusivamente en el transcurso de un trastorno psicótico crónico, como son la esquizofrenia o el trastorno delirante. G. Los síntomas no son debidos a los efectos fisiológicos directos de una sustancia (p. ej., una droga, un medicamento) o a enfermedad médica (p. ej., hipotiroidismo). H. Los síntomas causan un malestar clínicamente significativo o deterioro social, laboral o de otras áreas importantes de la actividad del individuo. Como vemos, la Distimia presenta síntomas más o menos similares a la Depresión mayor, pero se diferencian entre sí respecto a: Tipo de evolución: el estado de ánimo depresivo es crónico (no presenta intervalos libres de síntomas o mejorías significativas) y dura por lo menos 2 años. Severidad de los síntomas: los mismos suelen ser leves o moderados, sin una alteración significativa de las relaciones familiares, sociales y laborales del individuo.

Referencias Bibliográficas - Diagnostic and Statistical Manual of Mental Disorders, 4° edition (DSM-IV TR). American Psychiatric Association. American Psychiatric Press, 2000. - Kaplan and Sadock's Synopsis of Psychiatry, 9° edition. Lippincott Williams & Wilkins Press, 2003.

15.- Se trata de femenino de 54 años con datos clínicos de trombosis venosa profunda ¿Cuál de los siguientes procedimientos es la primera prueba que debe solicitar el médico para establecer el diagnóstico? a) b) c) d)

Pletismografía de impedancia Cateterismo cardíaco Venografía con contraste Estudios de ventilación y perfusión pulmonares

La pletismografía de las extremidades inferiores es una prueba incruenta útil para establecer el diagnóstico de trombosis venosa profunda. La venografía con medio de contraste proporciona el diagnóstico definitivo casi en cualquier caso; sin embargo, esta prueba cruenta puede, de hecho, causar tromboflebitis en una minoría de los casos. El cateterismo cardíaco, el gammagrama pulmonar y la tomografía computadorizada (CT) no son útiles para diagnosticar trombosis venosa profunda.

Allen R. M. MMS Medicina Interna. 5ª. Edición. National Medical Series. Mc. Graw Hill. 2006. (capítulo 1 VIIIA 4 a-b),

16.- Masculino de 30 años de edad adicto a las drogas intravenosas presenta debilidad del hemicuerpo derecho y cefalea en un periodo de dos días. La exploración revela un individuo mal nutrido y afebril con hemiparesia derecha leve. ¿Cuál de los siguientes es el diagnóstico más probable?

a) b) c) d)

Endocarditis bacteriana Absceso cerebral Meningitis por virus de la inmunodeficiencia humana (VIH) Meningitis criptocócica

Los adictos a drogas intravenosas están propensos a sufrir bacteriemia, que a su vez puede producir absceso cerebral y disfunción neurológica progresiva. Los pacientes por lo común se encuentran afebriles a no ser que haya endocarditis acompañante u otro origen endovascular de infección. Los adictos a drogas intravenosas tienden a presentar endocarditis bacteriana y pueden cursar con déficit neurológicos en forma apoplética debido a embolia séptica del cerebro. Sin embargo, por lo regular tienen fiebre. La meningitis por VIH produce cefalea y datos de irritación meníngea, pero no se presenta déficit neurológico focal. La meningitis criptocócica se manifiesta con conducta alterada y cefalea y los pacientes están afebriles. Sin embargo, es raro que haya datos de seudoapoplejía. Por último, el uso de drogas intravenosas puede provocar embolia de cuerpo extraño pero con problemas neurológicos apopléticos. Un émbolo puede llegar al cerebro por un cortocircuito cardíaco de derecha a izquierda o bien por una malformación arteriovenosa pulmonar si la inyección es venosa. El émbolo puede entrar a la circulación cerebral de manera directa en caso de inyección intracarotídea.

Referencias Nath A. Brain abscess and parameningeal infections. In: Goldman L, Ausiello D, eds. Cecil Medicine. 23rd ed. Philadelphia, Pa: Saunders Elsevier; 2007: chap 438 Allen R. M. MMS Medicina Interna. 5ª. Edición. National Medical Series. Mc. Graw Hill. 2006. (Capítulo 11 XVI B 1-2).

17.- Mujer de 60 años con antecedentes de cuadro respiratorio alto hace 3 semanas. Desde hace dos presenta vértigo de inicio súbito intenso y persistente, con varias horas de duración e inestabilidad a la marcha, no refiere tinitus ni hipoacusia. El diagnostico clínico más probable es: a) b) c) d)

Mastoiditis. Otoesclerosis. Neuronitis vestibular. Otitis media.

Neuronitis Vestibular (NV) Es una causa relativamente frecuente de SV. Hay compromiso de etiología viral del nervio vestibular, a nivel del ganglio de Scarpa (ganglio vestibular). Se ve preferentemente en adultos (14). Cuadro clínico: inicio súbito de vértigo intenso y persistente, acompañado de náuseas y vómitos e inestabilidad en la marcha (15). Sin tinnitus ni hipoacusia. Son crisis de días o semanas de duración, que se presentan esporádicamente en un lapso de hasta 3 años desde el inicio de los síntomas (16). Fisiopatología: se postula que la NV se produce en respuesta a una infección viral o a cambios inflamatorios posteriores en el ganglio de Scarpa. La NV no tiene correlación o base histopatológica conocida.

Diagnóstico: se basa principalmente en la clínica, ya que no existe un examen específico. En el examen de VIII par lo único positivo suele ser la presencia de desequilibrio en la marcha, paresia vestibular unilateral y excepcionalmente VPPB asociado (17,18). Hay que tener presente que algunos tumores del ángulo ponto-cerebeloso puede manifestarse como cuadros de “neuronitis vestibular” de difícil manejo (19,20). Tratamiento: se debe indicar reposo con licencia médica, ya que las caídas y traumatismos que pueden sufrir los pacientes constituyen complicaciones generalmente peores que la enfermedad causal. Se utilizan sedantes vestibulares, tanto en el episodio inicial como en las reagudizaciones posteriores (21). Algunas alternativas de tratamiento farmacológico:  difenidol (Vontrol), 25 mg vo cada 8 hrs.  cinarizina, 25 mg vo cada 12 hrs. También puede ser necesaria la rehabilitación vestibular a largo plazo. Síndromes Vestibulares Periféricos: Segunda parte Diagnóstico Diferencial y Etiologías J. Ignacio Méndez 1, Héctor Riveros 2, M. José Concha

18.- Se trata de femenino de 50 años con antecedente: Dx. De síndrome anémico sin causa aparente. Exámenes de Laboratorio Reportan: Hb de 8,5 g/dl, VCM de 75 fl, y HCM de 25 pg. El examen de laboratorio que conforma el origen del diagnóstico de esta paciente es la determinación de: a) Sideremia. b) Ferritina. c)

Saturación de la transferrina.

d) Hemoglobina A2.

La anemia por déficit de hierro ¿es un proceso que aparece de forma rápida o de forma más bien lenta? La anemia por falta de hierro se va estableciendo de forma gradual, cuando el hierro que se pierde supera al hierro aportado por la dieta; lo primero que ocurre es que el organismo empieza a utilizar las reservas de hierro. En esta etapa la ferritina empieza a descender, pero el hierro sérico y la TIBC generalmente permanecen inalterados no estableciéndose todavía anemia. En el momento en que el déficit de hierro empeora, el hierro sérico disminuye mientras que la transferrina y la TIBC aumentan, empezando las células de la serie roja (hematíes) a palidecer (hipocromia) y a disminuir de volumen (microcitosis), existiendo todavía un número suficiente de células de la serie roja. Si la deficiencia de hierro continúa o empeora aparece entonces la anemia. Mourey L. Manual de procedimientos del Laboratorio Clínico. México:

IMSS, 158. Última edición.

19.- Se trata de masculino de 58 años de edad con diagnostico de HPB, se niega a tomar tratamiento farmacológico y elige una RTUP, la complicación más frecuente de este procedimiento es: a) b) c) d)

Contractura de cuello vesical Impotencia Incontinencia Eyaculación retrógrada

Resección Transuretral (RTU): es el gold standard de las intervenciones. __ En general es una cirujía corta. __ 90% mejoría importante de los síntomas. __ Morbimortalidad baja del 0,2%. __ El 20 % debe reoperarse a los 10 años aproximadamente. __ Complicaciones: impotencia 10%, eyaculación retrograda 50%, Incontinencia 4%.

BIBLIOGRAFIA: 1. Medina JJ, Parra RO, Moore RG. Benign prostatic hyperplasia (the aging prostate). Med Clin North Am 1999 Sep;83(5):1213-29 2. Oesterling JE. Benign prostatic hyperplasia. Medical and minimally invasive treatment options. N Engl J Med 1995 Jan 12;332(2):99-109

20.- Se reportaron 45 casos de varicela entre los alumnos de primer grado de una escuela secundaria durante la segunda quincena del mes de agosto de 2012. La tasa de ataque es de 26.4%. Esta situación nos indica que estamos ante un(a): a) b) c) d)

Epidemia Endemia Brote Pandemia

De acuerdo a la NOM- 017 un brote se define como la ocurrencia de dos o más casos asociados epidemiológicamente (tiempo, lugar y persona) entre sí. La medida cuantitativa de la extensión de un brote es la Tasa de Ataque (TA) que se calcula dividiendo el número de casos nuevos entre el total de personas expuestas por 100.

Norma Oficial Mexicana NOM-017-SSA2 -1994, Para la vigilancia epidemiológica. Apartados 3.1.6 y 3.1.78.1

21.- En consulta externa de su hospital, recibe usted los resultados histopatológicos de una paciente de 22 años de edad que acudió a revisión rutinaria, los resultados reportan imagen histológica de coilocitos lo cual sugiere infección por:

a) b) c) d)

Herpes virus tipo 2 Citomegalovirus Vaginosis bacteriana. Virus del papiloma humano

Células del epitelio cervical con un halo perinuclear típico secundario a la infección por el virus del papiloma humano ( VPH ). Su presencia determina el diagnóstico de lesión intraepitelial de bajo grado ( LSIL ) en la patología cervical e indica la infección de las células por el VPH. Para que una célula sea considerada como coilocítica, debe combinar la atipía nuclear con la presencia de vacuola perinuclear, ya que esta última característica puede observarse en procesos no relacionados al HPV como ser infecciones por Trichomonas o Cándidas, sin asociación a atipías nucleares. El coilocito es la manifestación clásica de la infección por VPH en la célula. Fue descrito por primera vez por Koss y Durfee en 1956. Esta célula también ha sido llamada “célula en balón”. El coilocito es una célula epitelial escamosa, más comúnmente superficial e intermedia, aunque también puede verse en células parabasales y metaplásicas. Esta célula presenta cambios típicos tanto en su núcleo como en su citoplasma, pierde los bordes angulados usuales de la célula escamosa superficial y su forma tiende a ser redondeada y ovoide. El citoplasma muestra una condensación periférica que le da un aspecto en “asa de alambre”, es opaco, denso y de aspecto céreo, anfofílico, acidofílico o de color rojo/naranja brillante. Además se observa una gran cavidad o halo con un margen muy bien definido, de forma oval o ligeramente festoneado. El núcleo de la célula se localiza de manera excéntrica, lo que lo convierte en un halo paranuclear, no perinuclear. Ocasionalmente puede encontrarse material fagocitado dentro del espacio coilocítico.

Lesión Intraepitelial de Bajo Grado. Alteraciones Celulares compatibles con Infección (Coilocitos). Schlecht, N.F., Kulaga, S., Robitaille, J., Ferreira, S., Santos, M., Miyamura, R.A., Duarte-Franco, E., Rohan, T.E., Ferenczy, A., Villa, L.L., & Franco, E.L. (2002) Persistent Human Papillomavirus Infection as a Predictor of Cervical Intraepithelial Neoplasia. JAMA, 286, 3106-3114

22.- Mujer de 32 años, con un cuadro clínico de sangrado menstrual irregular, dismenorrea, dispareunia y una esterilidad de 3 años de evolución, con resultados de laboratorio hormonal normal y un estudio ecográfico transvaginal que informa de un útero normal y sendas formaciones quística ováricas bilaterales de 4 cms. La orientación diagnóstica es: a) Hemorragia uterina disfuncional. b) Síndrome del ovario poliquístico. c) Endometriosis. d) Quistes dermoides bilaterales. Causas y síntomas de endometriosis: Las causas de la endometriosis aún no se conocen. Las células del revestimiento interno del útero de alguna manera se desplazan hasta zonas externas al mismo y siguen creciendo. Este desplazamiento podría quizás deberse a que pequeños fragmentos del revestimiento uterino, desprendidos durante la menstruación, retrocedan hacia las trompas de Falopio en dirección a los ovarios hasta entrar en la cavidad abdominal, en lugar de salir con el flujo menstrual a través de la vagina. La endometriosis causa dolor en la parte inferior del abdomen y la zona pélvica, irregularidades menstruales (como manchar antes de la menstruación) e infertilidad.

Algunas mujeres con endometriosis grave no presentan síntomas, mientras que otras con la enfermedad en grado mínimo sufren un dolor invalidante. Con frecuencia, el dolor menstrual debido a la endometriosis no aparece hasta años después de desarrollar la enfermedad. En algunos casos, se constata dolor durante el coito (dispareunia), antes o durante la menstruación. El tejido endometrial adherido al intestino grueso o a la vejiga urinaria puede provocar hinchazón abdominal, dolor durante las deposiciones, hemorragia rectal durante la menstruación o dolor en la parte inferior del abdomen durante la micción. Así mismo, cuando el tejido se localiza en un ovario o una estructura cercana puede dar lugar a la formación de una masa llena de sangre (endometrioma). En ocasiones, el endometrioma se rompe bruscamente o se escapa algo de su contenido, lo que causa un agudo y repentino dolor abdominal.

23.- Se trata de masculino de 25 días de nacido al cual se desprendió el cordón umbilical a los 9 días y cuyo ombligo no ha cicatrizado. Presenta en este nivel una tumoración roja, prominente, circular y que segrega un contenido alcalino. Refiere la madre que a veces nota en esta tumoración ruido de gases. ¿El diagnóstico más probable es?

a) b) c) d)

Persistencia del conducto onfalomesentérico. Fístula vésico-umbilical. Granuloma umbilical. Onfalocele.

Durante la vida fetal el conducto onfalomesentérico une al saco vitelino con el intestino medio y se cierra normalmente para desaparecer por completo. Se encuentra conectado con el intestino primitivo en el saco amniótico. En el desarrollo embriológico normal, el conducto onfalomesentérico involuciona entre las 5a y 7a semanas de vida intrauterina. Un fracaso en la regresión produce varias anormalidades, en dependencia del lugar donde se localice este fallo: en el lado umbilical o en el intestinal. Los vestigios del conducto onfalomesentérico (vitelino) pueden presentarse como anomalías relacionadas con la pared abdominal. Sin embargo, puede ocurrir que todo o parte del conducto fetal se mantenga y entonces se produzca sintomatología clínica. También puede persistir como una estructura permeable en toda su longitud o mantenerse como un divertículo o quiste cuando persiste en su periferia, parte central o media; o quizás quede representado simplemente por un resto de epitelio intestinal ectópico a nivel umbilical o como cordón fibroso. La explicación embriológica de la persistencia del conducto onfalomesentérico puede explicarse diciendo que el conducto onfaloentérico o vitelino que representa una comunicación entre el vértice del asa umbilical fetal y el saco vitelino se oblitera y desaparece junto con sus vasos acompañantes arteria y vena onfalomesentéricas hacíanla

séptima semana-fetal. Sin embargo, cuando este proceso no ocurre, da lugar a la patología conocida de las estructuras remanentes del conducto onfalomesentérico. Después de los cuidados iniciales en la edad del recién nacido que se han dado al cordón umbilical y una vez que éste se ha desprendido, esta zona recibe poca atención en los días subsiguientes; sin embargo, puede ser asiento de muchas lesiones, tanto congénitas como adquiridas, con lascuales el médico debe estar familiarizado para poder diagnosticarlas. El diagnóstico se hace mediante la observación, cuando el recién nacido realiza esfuerzos como el provocado por el llanto, pues se produce la salida de material fecal o gases por el ombligo.

REFERENCIAS: 1. Pomeranz A. Anomalies, abnormalities, and care of the umbilicus. Pediatr Clin N Am 51 (2004) 819. 2. Rowe M, Chapter 48: Disorders of the umbilicus. Essentials of Pediatric Surgery, 1995. 3. Care of the umbilical cord, a review of the evidence. OMS. 1998. 4. Cushing A. Omphalitis: a review. Pediatr Infect Dis J 1985, 4(3); 282-285 5. Janssen P. To dye or not to Dye: A randomized, Clinical Trial of a Triple Dye/alcohol Regime Versus Dry cord care. Pediatrics 2003; 111, 15-20. 6. Zupan J. Topical umbilical cord care at birth The Cochrane Database of Systematic Reviews, 2004. Issue 3. 7. Sawardekar K. Changing spectrum of neonatal omphalitis. Pediatr Infect Dis J, 2004;23:22-6 8. Soto G. Anomalías uracales: un diagnóstico rara vez planteado. Pediatría al día 2001;17(5) 331) 9. Cilento BG Jr. Urachal anomalies: defining the best diagnostic modality. Urology 1998, 52 (1) 120-2. 10. Ossandón F. Malformaciones congénitas: patología umbilical. Pediatría al día 1995;11(2) 11. Lotan G. Double ligature: A treatment for pedunculated umbilical granulomas in children. Am Fam Physician 2002;65:2067-8 12. Novoa A. El pediatra ante un lactante con caída tardía del cordón umbilical. Arch arg Pediatr 2004; 102(3).

24.- Se trata de preescolar de 4 años de edad presenta un peso y talla en el percentil 3, se refiere por presentar anorexia, irritabilidad, apatía al juego, se le observa con palidez, el laboratorio reporta una hemoglobina de 7.3g/dl, hierro sérico de 36mcg/dl y saturación de transferían de 12%, ¿El tipo de anemia más probable es?

a) b) c) d)

Hemolítica Perniciosa Ferropénica Drepanocítica

Es el tipo de anemia más frecuente en la población general. Se trata de una anemia caracterizada por la disminución o ausencia de los depósitos de hierro, baja concentración de hierro sérico, baja saturación de transferrina y baja concentración de la Hb o del hematocrito. La ferropenia se manifiesta con un síndrome característico: SNC: irritabilidad, labilidad emocional, disminución de la concentración y la memoria, cefalea, ataxia, parestesias, trastornos del sueño y la alimentación (PICA: apetencia desmedida por sustancias poco habituales. Es muy característica la pica de hielo o pagofagia), aumento de la sensibilidad al frío, síndrome de piernas inquietas…

Piel y mucosas: coloración azulada de la esclerótica, fragilidad de uñas y cabello, coiloniquia o aspecto cóncavo de las uñas, glositis, queilitis angular o rágades, atrofia gástrica, en casos graves producción de membranas esofágicas (síndrome de Plummer-Vinso). Etiología: La anemia ferropénica puede deberse a: Aumento de la utilización: gestación, lactancia, crecimiento corporal rápido en la infancia y adolescencia. Pérdidas fisiológicas: menstruación Pérdidas patológicas: Hemorragia digestiva. El sangrado digestivo crónico es la causa más frecuente en este grupo: hemorragias gástricas por medicamentos (AAS, AINES, corticoides asociados a AINES…), hernia hiatal, diverticulosis, hemorroides. En pacientes mayores de 60 años debe considerarse la posibilidad de neoplasia, sobre todo de colon y la angiodisplasia intestinal. Genito-urinarias Aparato respiratorio Hemólisis intravascular Alteraciones en la absorción: Dietas insuficientes: Debe interrogarse siempre acerca de los hábitos dietéticos. El hierro presente en alimentos de origen vegetal como cereales y legumbres tiene baja biodisponibilidad (algunos alimentos como las lentejas tienen excesiva fama de riqueza en hierro). Incluso en carnes y pescados el

contenido en Fe es relativamente bajo. En gestantes, niños, mujeres en edad fértil, la dieta puede ser un factor ostmenopáusi o causal de deficiencia de hierro. En mujeres ostmenopáusicas y en el varón la dieta sería causa excepcional de ferropenia. Absorción defectuosa: gastrectomías parciales o totales, enfermedad inflamatoria intestinal, enfermedad celíaca (causa relativamente frecuente de anemia ferropénica); en algunos casos podría estar indicada la detección de anticuerpos antiendomisio y antigliadina para descartar una enfermedad celíaca no diagnosticada. Diagnóstico: El nivel de ferritina sérico bajo es indicativo de una situación de depleción de hierro y es la prueba definitiva de la existencia de anemia por déficit de hierro (AF). Los demás parámetros no permiten diferenciar con seguridad la AF de la anemia por enfermedad crónica. Generalmente aparece microcitosis, aunque hasta en el 30% de las anemias ferropénicas, en algunas poblaciones, el VCM puede ser >80; asimismo en el 10% de las microcitosis los parámetros relacionados con el hierro son normales. Otros datos a tener en cuenta: El frotis periférico habitualmente muestra anisocitosis y poiquilocitosis. Cuando la microcitosis se asocia a aumento de glóbulos rojos, sobre todo si es mantenida, pensar en talasemia. Puede haber trombocitosis reactiva.

Bibliografía 1.

Abramson S, Abramson N. Common' Uncommon Anemias. Am Fam Physc 1999; 59 (4). 851-8 2. Adamson JW. Anemia and Polycytemia. En: Kaasper DL, Braunwald E (Edit.). Harrison´s principles of internal medicine. 16th. ed. Nwe Cork: McGraw-Hill; 2005. p.329-336 3. Hernández Nieto L, Hernández García MT, Juncá Piera J, Vives-Corrons JL, MartínVega C. Enfermedades del sistema eritrocitario: anemias. En: Farreras Valentí P, Rozman C. (Dir.). Medicina Interna. Barcelona: Elsevier; 2004. p. 1644-1669 4. Glader B. Anemia: General Considerations. En: Wintrobe´s Clinical Hematology. 11th. Philadelphia: Lippincott Williams & Wilkins; 2004. p. 947-978.

25.- Femenino de 28 años. Acude a consulta por hirsutismo e irregularidades menstruales. Antecedente familiar de DM tipo 2 en madre y una hermana. Su padecimiento empezó desde la adolescencia con ritmo menstrual de 40 – 60 x 5 -7 días. EF: peso 87 kg, estatura 1.54 m, TA 130/80, FC 72x’, acantosis nigricans en cuello y axilas, no galactorrea, perímetro abdominal de 112 cm. Vello supralabial, en patilla, en línea media infraumbilical y en cara interna de muslos. Laboratorio: glucosa 104 mg/dL, resto de química sanguínea normal, Bh normal. Prolactina 16 ng/dL, testosterona: 78 ng/dL (25 a 50 ng/dl).

El diagnóstico probable es:

a) b) c) d)

Síndrome de ovarios poliquísticos por resistencia a la insulina Hiperprolactinemia por microprolactinoma Virilización por tumor productor de testosterona Diabetes mellitus secundaria a la obesidad

El síndrome de ovarios poliquísticos (SOPQ) afecta aproximadamente a un 4% de mujeres en edad reproductiva y se caracteriza por anovulación crónica e hiperandrogenismo. Es la causa más común de infertilidad en mujeres. Se caracteriza clínicamente por acné, alopecia, hirsutismo, irregularidades menstruales e infertilidad. Los hallazgos de laboratorio más frecuentes son: aumento de la hormona luteinizante (LH), aumento de la relación LH/FSH (hormona folículoestimulante), aumento de andrógenos (tanto ováricos como adrenales) y de estrógenos circulantes. Otros hallazgos de laboratorio habituales son una prueba tolerancia oral a la glucosa anormal y alteraciones en el perfil lipídico. Todo esto junto con las imágenes ecocardiográficas características definen al síndrome. La terapéutica permite dos grandes enfoques que pueden superponerse: la corrección de las manifestaciones de hiperandrogenismo y el tratamiento de las alteraciones del eje reproductivo (anovulación, esterilidad). Los antiandrógenos están fundamentalmente indicados para tratar los síntomas virilizantes. Las alternativas para inducir la ovulación son numerosas: al citrato de clomifeno y a la antigua resección en cuña se agregan las gonadotrofinas humanas, pulsos de GnRH (hormona liberadora de gonadotrofinas), medidas o fármacos para modificar los niveles de insulina, y finalmente técnicas quirúrgicas endoscópicas para reducir la masa ovárica.

Revista de Posgrado de la VIa Cátedra de Medicina - N° 125 – Marzo 2003 Pág. 37-40 SINDROME DE OVARIOS POLIQUISTICOS Dra. Sandra Beneyto, Dra. María Andrea Ferreyra, Dr. Andrés Galfrascoli, Dr. Andrés González, Dra. Susana Sosa

26.- Se trata de femenino de 22 años que acude a consulta por pérdida de peso, diarrea, palpitaciones, temblor y labilidad emocional. No tiene antecedentes importantes. Refiere evacuaciones diarreicas de 5 a 7 por día, sin moco o sangre. EF: peso 52 kg, estatura 1.59, FC 108x’, TA: 120/65, retracción palpebral, hiperemia conjuntival y de carúncula, cuello con tiroides aumentada de tamaño 3 veces de forma difusa, extremidades superiores con hiperhidrosis palmar, temblor fino distal, reflejos exaltados

El diagnóstico probable es: a) b) c) d)

Parasitosis del tipo de la giardiasis Bocio multinodular tóxico Taquicardia paroxística Enfermedad de Graves

El bocio tóxico difuso (BTD) constituye la forma más frecuente de hiperfunción de la glándula tiroidea (70 % de los casos), que puede aparecer a cualquier edad, aunque por lo general aparece entre la tercera y cuarta década de la vida. Esta enfermedad es más frecuente en la mujer, donde se observa un predominio de 7:1 en relación con los hombres en regiones no bociógenas. Esta relación se reduce en las zonas de bocio endémico. Los factores genéticos desempeñan un papel esencial en la etiología y existe una predisposición familiar a esta enfermedad de Graves-Basedow. El BTD se caracteriza por la presencia de hipertiroidismo, bocio difuso y elástico, oftalmopatía, dermopatía, acropaquia tiroidea y onicolisis. Es importante el diagnóstico y tratamiento precoz del hipertiroidismo para evitar complicaciones, principalmente las cardiovasculares. Criterios diagnósticos y terapéuticos Sospecha clínica • Síntomas de mayor valor diagnóstico 󲐀 Intolerancia al calor 󲐀 Palpitaciones 󲐀 Nerviosismo 󲐀 Insomnio 󲐀 Irritabilidad 󲐀 Hiperquinesia 󲐀 Fatigabilidad fácil 󲐀 Polifagia 󲐀 Perdida de peso 󲐀 Hiperdefecación y/o diarreas • Signos de mayor valor diagnóstico 󲐀 Bocio 󲐀 Taquicardia 󲐀 Temblor de las manos 󲐀 Debilidad muscular 󲐀 Piel húmeda y caliente 󲐀 Hiperquinesia 󲐀 Pelo fino y frágil Bibliografía: 1. Bryer-Ash M. Evaluation of the patient with a suspected thyroid disorder. Obstet Gynecol Clin North Am 2001 Jun;28(2):421-38

2. Weber AL, Randolph G, Aksoy FG. The thyroid and parathyroid glands. CT and MR imaging and correlation with pathology and clinical findings. Radiol Clin North Am 2000 Sep;38(5):1105-29 3. Jarlov AE, Nygaard B, Hegedus L, Hartling SG, Hansen JM. Observer variation in the clinical and laboratory evaluation of patients with thyroid dysfunction and goiter. Thyroid 1998 May;8(5):393-8 4. Costa AJ. Interpreting thyroid tests. Am Fam Physician 1995 Dec;52(8):2325-30

27.- Hombre de 55 años, el cual presenta distensión abdominal en incremento e ictericia de 6 semanas de evolución. La EF revela arañas vasculares, venas ingurgitadas alrededor del ombligo y ascitis. Lla causa más probable de esta ictericia es:

a) Hepatitis b) Coledocolitiasis c) Cirrosis d) Cáncer de páncreas

1.- CIRROSIS HEPÁTICA DEFINICIÓN: Se trata de una alteración crónica e irreversible que representa la etapa final de diversos procesos agudos o crónicos que afectan al hígado, si bien el concepto es fundamentalmente morfológico donde se define como una alteración difusa de la arquitectura del hígado por fibrosis y nódulos de regeneración, clínicamente se puede sospechar por los antecedentes y los diversos datos clínicos que se pueden documentar. Se sabe que los cambios histológicos condicionan una alteración vascular intrahepática y una reducción de la masa funcional hepática y como consecuencia se desarrolla hipertensión portal e insuficiencia hepática, dando como resultado final sus complicaciones mayores: ascitis, hemorragia digestiva, ictericia y encefalopatía hepática.

• •

• •

Cirrosis: Proceso hepático difuso con fibrosis y formación de nódulos Muchas causas; resultado final similar – Necrosis – Fibrosis I III – Regeneración Disminución síntesis Alteraciones vasculares: Hipertensión portal

ALCOHOL • Es una de las principales causas de cirrosis • El consumo per capita está correlacionado a la mortalidad global por cirrosis • Dependencia de alcohol en 5% de la población • Umbral de riesgo: 60 g/d en hombres, 20 g/d en mujeres

ETIOLOGÍA: Consumo excesivo de alcohol. Infección crónica por los virus de la hepatitis B y C. Hemocromatosis hereditaria Enfermedad de Wilson Hepatitis autoinmune Esteatohepatitis no alcohólica. Colestasis obstructivas crónicas Obstrucción crónica del drenaje venoso Toxicidad por fármacos (Metotrexato, metildopa, vitamina A, etc.). En los niños la causa más frecuente es la anormalidad congénita de la vía biliar (atresia de vía biliar, enfermedad de Alagille, enfermedad de Byler). Cuando no es posible determinar el factor etiológico (5%) se denomina cirrosis criptogenética. CLÍNICA: La cirrosis compensada puede ser clínicamente latente, asintomática y ser descubierta fortuitamente en un examen clínico efectuado por otro motivo. Otros pacientes se diagnostican cuando son explorados por síntomas inespecíficos, o en el seguimiento de una hepatitis crónica viral, en algunos otros casos los datos son muy floridos aunados a los antecedentes de relevancia (ingesta intensa de alcohol). Los signos y síntomas que pueden acompañar a la cirrosis son múltiples y muy variados, si bien no todos se presentan en todos los casos ni tiene un carácter patognomónico se pueden observar en la gran mayoría de estos enfermos; los podemos clasificar por aparatos y sistemas: SIGNOS Y SÍNTOMAS DE LA CIRROSIS HEPÁTICA Síntomas generales: Astenia Adinamia Signos cutáneos-ungueales Arañas vasculares Telangiectasias Eritema palmar Hipertrofia parotídea Contractura de Dupuytren Acropaquia Uñas en vidrio de reloj Desaparición de la lúnula (leuconiquia) Exploración abdominal Hepatomegalia Esplenomegalia

28.- Masculino de 47 años de edad con cirrosis hepática tuvo dolor abdominal generalizado durante 24 h sin náuseas ni vómitos. Su temperatura es de 38.3°C y ha tenido distensión abdominal con onda de líquido claro. Hay hipersensibilidad difusa en la palpación abdominal. En la paracentesis se obtuvo líquido transparente con 816 leucocitos/mm3 (85%

polimorfonucleares, 15% linfocitos). La tinción de Gram no muestra bacterias. ¿Cuál de los siguientes diagnósticos es más probable? a) b) c) d)

Enfermedad ulcerosa péptica Pancreatitis Colecistitis Peritonitis primaria

El diagnóstico más probable es peritonitis primaria. Aunque es difícil diferenciar la peritonitis primaria (espontánea) por rotura de víscera hueca y contaminación peritoneal, la presencia de fiebre y la leucocitosis la ascitis sugiere alguna clase de infección peritoneal. La pancreatitis se caracteriza por dolor localizado intenso (mesoepigástrico), que se irradia a la espalda. En general, las náuseas y vómitos no se relacionan con pancreatitis aguda. El dolor abdominal en caso de colecistitis se sitúa en el cuadrante superior derecho y suele haber náuseas y vómitos. El absceso hepático tiende a ser un trastorno subagudo sin datos peritoneales prominentes. En caso de ascitis crónica infectada, las enfermedades ulcerosas pépticas son causa poco probable. Allen R. M. MMS Medicina Interna. 5ª. Edición. National Medical Series. Mc. Graw Hill. 2006. (capítulo 8 V E 1 a).

29.- Femenino de 56 años que actualmente se diagnóstica con DM tipo 2, se desconoce el tiempo de evolución de la enfermedad, se envía a valoración por oftalmología, la primera lesión que se observa en la retinopatía diabética temprana es:

a) b) c) d)

Exudados blandos Exudados duros Microaneurismas Hemorragias intrarretinianas

La retinopatía diabética tiene cuatro etapas: 1. Retinopatía no proliferativa ligera. Esta es la etapa más temprana de la enfermedad en la que aparecen los microaneurismas. Estas son pequeñas áreas de inflamación, que parecen ampollas, en los pequeños vasos sanguíneos de la retina. 2. Retinopatía no proliferativa moderada. Según avanza la enfermedad, algunos vasos sanguíneos que alimentan la retina se obstruyen. 3. Retinopatía no proliferativa severa. En esta etapa muchos más vasos sanguíneos se bloquean, haciendo que varias partes de la retina dejen de recibir sangre. Entonces estas áreas de la retina envían señales al cuerpo para que haga crecer nuevos vasos sanguíneos. 4. Retinopatía proliferativa. En esta etapa avanzada, las señales enviadas por la retina para alimentarse causan el crecimiento de nuevos vasos sanguíneos. Esto se llama la retinopatía proliferativa. Estos nuevos vasos sanguíneos son anormales y frágiles. Crecen a

lo largo de la retina y de la superficie del gel vítreo, el gel incoloro que llena el interior del ojo.

Retinopatía diabética. Forma leve de retinopatía diabética, en la que destacan microaneurismas /microhemorragias de predominio en arcada temporal superior, junto con exudados duros que se acercan a la mácula.

1. The Diabetes Control and Compications Trial Research Grop. The Effect of Intensive treatment of Diabetes on the development and progression of long term complications in insulin dependent Diabetes Mellitus N Engl J. Med; 1993 ; 329 : 977-986. 2. Early treatment Diabetic Retinopathy Study Research Group Early photocoagulation for diabetic Retinopathy ETDRS Report 9. Ophthalmology, 1991 ; 98 : 1316-26. 3. Kahn HA Hiller R. Blindness caused by diabetic retinopathy Am. J. Ophthalmol, 1974; 78 ; 58-67. 4. Ladas ID. Theossiadis GA Long term efectiveness of modified grid photocoagulation for diffuse Macular edema Acta Ophthalmol, 1993 ; 71(3) ; 393-7. 5. Klein R, Klein BEK , Moss SE , et al The Eisconsin Epidemiology Group Ophthalmology, 1984 ; 91 : 1464-74.

30.- Masculino de 50 años con dolor precordial relacionado al esfuerzo, de corta duración, de 4 meses de evolución y con ECG en reposo normal, el siguiente estudio de elección es: a) b) c) d)

Ecocardiograma de reposo Prueba de Talio- Dipiridamol Ecocardiograma con estrés farmacológico Prueba de esfuerzo con protocolo de Bruce

El propósito fundamental de la prueba es el de demostrar la existencia de isquemia miocárdica en los subgrupos de población con mayor prevalencia de cardiopatía isquémica o bien en los subgrupos en donde la prueba se efectúa a manera de evaluación del tratamiento en pacientes ya conocidos con cardiopatía isquémica. Existe otro subgrupo de población en quienes este estudio ayuda a determinar la clase funcional en la que se encuentran e incluye a pacientes con valvulopatías o con insuficiencia cardiaca de cualquier etiología.

Protocolos de esfuerzo. El protocolo más empleado es el de Bruce sobre treadmill, aunque existen otros protocolos y su elección dependerá de las condiciones del individuo. Los protocolos discontinuos son los que alternan periodos de esfuerzo que se intercalan con periodos de reposo de duración similar, se emplean en escasas circunstancias. Los protocolos continuos son los que no interrumpen el esfuerzo una vez iniciado hasta finalizada la prueba, permiten mejor adaptación física y psicológica y es posible adaptar la intensidad de forma individualizada para que la prueba tenga una duración de 6 a 12 minutos. Los protocolos máximos son los que se suspenden debido a la sintomatología del paciente, a los signos registrados durante la prueba o se alcanzan valores máximos de FC y VO2. Los protocolos submáximos son los que se suspenden cuando el sujeto alcanza un nivel determinado de carga, habitualmente el 85% de la FC máxima teórica (que se encuentra entorno a los 170 lpm). En la práctica diaria, el nivel de carga (VO2) se expresa en forma de trabajo externo (MET ó equivalentes metabólicos) que corresponden a 3,5ml/kg/min de VO2, lo que permite comparar protocolos entre sí (cada protocolo dispone de fórmulas para realizar el cálculo de los METS), el error que cometen en el cálculo de los METS es mayor en protocolos discontinuos. Emplear la FC como único criterio para determinar el esfuerzo máximo es erróneo, por lo que deberían tenerse en cuenta otros criterios, como es la percepción subjetiva por parte del paciente mediante la escala de Borg (tabla I). Esta dificultad en la predicción del esfuerzo máximo es lo que limita la realización de pruebas submáximas a la determinación de la condición física de sujetos aparentemente sanos. Tabla I. Escala de Percepción del esfuerzo de Borg. Escala de 15 grados

Escala de 10 grados

Valor Percepción

Valor Percepción

6

No se siente nada

0

Nada

7

Muy muy leve

0,5

Muy muy leve

8 9 10 11 12 13 14 15 16 17 18 19 20

Muy leve

1

Muy leve

2

Leve

Considerablemente leve 3 Moderadamente dura Dura Muy dura Muy muy dura

Moderada

4

Algo fuerte

5

Fuerte o intensa

6 7

Muy fuerte

8 9 10

Muy muy fuerte (submáxima)

Esfuerzo máximo

* A la izquierda la escala original de esfuerzo percibido en 15 grados (de 6 a 20) y a la derecha la más nueva de 10 categorías. Tabla II. Indicaciones clásicas de la ergometría. I. Fines diagnósticos A. Pacientes sintomáticos. 1. Dolor torácico: a) Típico b) Atípico 2. Clínica de equivalentes isquémicos. A. Pacientes asintomáticos. 1. Con alteraciones en el ECG sugestivas de isquemia. 2. Con alta probabilidad de padecer Cardiopatía Isquémica (paciente con múltiples factores de riesgo) 3. Cuando convenga descartar con cierta seguridad CI. 4. Con sospecha de CI silente. 5. Sedentarios que inician programa de actividad física. 6. Para estudio funcional de ciertas arritmias. II. Con fines valorativos y pronósticos. 1. Seguimiento de paciente con CI conocida. 2. Tras IAM. 3. En exámenes prelaborales o laborales. 4. De la eficacia del tratamiento: Médico. Cateterismo y angioplastia. Quirúrgico.

5. Respuesta de la Tensión Arterial. 6. En valvulopatías o miocardiopatías. 7. Estudio de arritmias y trastornos de la conducción aurículo-ventricular. 8. En cardiopatías congénitas. Bibliografía 1.

2. 3.

4.

5.

6.

7. 8.

9. 10. 11. 12.

13. 14.

Gibbons RJ (Edit.). ACC/AHA 2002 Guideline Update of Exercise Testing. 2002 American College of Cardiology Foundation and American Heart Association ACC/AHA; 2002 [Acceso 1-4-06]. Disponible en: Guidelines for cardiac exercise testing. ESC Working Group on Exercise Physiology, Physiopathology and Electrocardiography Eur Heart J 1993; 14: 969-988. Fernando Arós Aros F, Boraita A, Alegria E, Alonso AM, Bardaji A, Lamiel R el al. Guías de práctica clínica de la Sociedad Española de Cardiología en pruebas de esfuerzo. Rev Esp Cardiol 2000; 53 (8): 1063-94 Chaitman B. Las pruebas de esfuerzo. En: Braunwald E, editor. Tratado de Cardiología. Medicina Cardiovascular. 4ª ed. Madrid Mc-Graw-Hill-Interamericana de España; 1993. p. 177-197. Schlant RC, Friesinger GC 2nd, Leonard JJ. Clinical competence in exercise testing: a statement for physicians from the ACP/ACC/AHA Task Force on Clinical Privileges in Cardiology. J Am Coll Cardiol 1990; 16: 1061-5 Reyes Lopez de los M, Iñiguez Romo A, Goicolea de Oro A, Funes Lopez B, Castro Beiras A. El consentimiento informado en cardiología. Rev Esp Cardiol 1998; 51: 782796. Fletcher GF, Flipse T, Malouf J, Kligfield P. Current status of ECG stress testing. Curr Probl Cardiol. 1998 Jul; 23(7): 353-423. Alegría Ezquerra E, Alijarde Guimerá M, Cordo Mollar JC, Chorro Gascó FJ, Pajarón López A. Utilidad de la prueba de esfuerzo y de otros métodos basados en el electrocardiograma en la cardiopatía isquémica crónica. Rev Esp Cardiol 1997; 50: 6-14 Wasserman K, Hansen JE, Sue DY, Whipp BJ, Casaburi R. Principles of exercise testing and interpretation . 2ª ed. Philadelphia: Lea & Febiger; 1994. p. 95-111. American college of Sports Medicine. Guideliness for exercise testing and prescription. 5ª ed. Baltimore: Williams & Wilkins; 1995. Borg GA. Psychophysical bases of perceived exertion. Med Sci Sports Exerc 1982; 14: 377-381 Froelicher VF, Umann TM. Exercise testing: clinical applications. En: Pollock ML, Schmidt DH, editors. Heart disease and rehabilitation. 3ª ed. Champaign, IL: Human Kinetics, 1995; p.57-79. Myers J, Froelicher VF. Exercise testing. Procedures and implementation. Cardiol Clin. 1993; 11(2): 199-213. Weiner DA, McCabe C, Hueter DC, Ryan TJ, Hood WB Jr. The predictive value of anginal chest pain as an indicator of coronary disease during exercise testing. Am Heart J 1978; 96: 458-462.

31.- Preescolar de 4 años de edad que acude al servicio de consulta externa diagnosticado con impétigo, el tratamiento fundamental primario de este padecimiento es:

a) b) c) d)

Es suficiente descostrar y usar antibiótico tópico Penicilina por 10 dias Dicloxacilina por 7 días Aciclovir por 5 días a dosis habituales

IMPETIGO Frecuente en la infancia. Más raro en el adulto, donde hay que buscar una dermatosis subyacente (a menudo parasitaria). Favorecida por falta de higiene. Muy contagiosa. No deja inmunidad por lo que frecuentemente recidiva. Definición Dermo-epidermitis superficial microbiana producida por Streptococcus pyogenes y/o Staphylococcus aureus. Etiología Actualmente es más frecuente la etiología estafilocócica que la estreptocócica, aunque muchas veces ambos gérmenes se asocian. Pilares diagnósticos a) clínico b) paraclínica. De poca utilidad práctica. Tratamiento primario: Mejorar las condiciones de higiene de la piel (generales y locales). Evitar maceración, humedad y rascado de las lesiones. Ducha diaria. Lavado repetido de manos, cepillado de uñas que deben mantenerse cortas. Cambio frecuente de ropa interior. a) local, único tratamiento en las formas muy localizadas. Realizar varias veces al día en forma sucesiva: Lavado y descostrado de las lesiones. Reblandecer las costras con compresas húmedas o aplicación de vaselina. Antiséptico local: solución de chlorhexidine o permanganato de potasio o sulfato de cobre. Crema con ácido fucídico o bacitracina o mupirocina. En lo posible tapar la lesión para evitar auto y heteroinoculación Ropa interior de algodón y amplia para evitar el roce.

BIBLIOGRAFIA

- Gantz NM, Brown RB, Berk SL, Espósito AL, Gleckman RA. Human infections following animal bites. In Manual of Clinical Problems in Infectious Disease. Nelson M Gantz et al. - Finegold SM, Baron EJ. Microorganisms encountred in wounds, abscesses, skin and soft tissue lesions. In Bailey & Scott´s Diagnostic Microbiology - Gradon J & Adamson C. Infections of Pressure Ulcers: Manegement and Controversies. Inf Dis Clin Practice 1:11-16 - Hirschman JV, Finegold DS. Cutaneaus Abcsesses and Ulcers in Infectious Diseases Ch 154. Gorbach SL, Bartlett JG and Blacklownr editors. WB Saunders co. 199 - Swartz MN. Enfermedades Infecciosas. Mandell GL, Bennett JE y Dolin R. 4ta Edición. T 1Cap 72:1010-1032.

32.- Thrombocytopenia that is caused by increased platelet destruction is most closely associated with which of the following conditions?

a) b) c) d)

Combination chemotherapy Systemic lupus erythematosus Acute leucemia Excessive ethanol intake

El diagnóstico de LEG se hace mediante la identificación de manifestaciones clínicas de la enfermedad acompañadas de uno o más autoanticuerpos típicos (Tablas 2 y 3).

Tabla 3 Criterios para la Clasificación del Lupus Eritematoso Generalizado Eritema malar Eritema discoide Fotosensibilidad Ulceras orales Artritis Serositis a. pleuritis b. pericarditis Renal:

c. proteinuria persistente > 0,5 g/24 h o +++ d. cilin

Neurológicos: e. convulsiones

f.

psicosis

Hematológicos: g. Anemia hemolítica con reticulocitosis h. Leucocitopenia < 4.000 por dos veces i. Linfocitopenia < 1.500 por dos veces j. Trombocitopenia < 100.000 10. Inmunológicos: a) Anti-DNA elevado b) Anti-Sm c) Antifosfolípidos: Anticardiolipinas (IgG o IgM) o Anticoagulante lúpico o VDRL falso positivo por 6 meses. 11. Anticuerpos antinucleares (en ausencia de lupus por drogas)

33.- Hombre de 44 años, refiere malos hábitos alimenticios comenta que con mucha frecuencia consume alimentos en la vía pública, actualmente se presenta con fiebre, edema facial, fotofobia y mioartralgias, estos datos son sugestivos de: a) b) c) d)

Larva migrans visceral Fasciolosis Criptosporidiosis Trichinellosis

DEFINICIÓN Se denomina triquinosis a la Infección parasitaria producida por nemátodos del género Trichinella, transmitida por carnivorismo, y caracterizada por un síndrome febril, signos oculopalpebrales, mialgias y eosinofilia elevada. Sinonimia: Trichinellosis. AGENTE Y CLASIFICACIÓN La Trichinella es un pequeño nemátodo blanquecino y filiforme, con su extremidad anterior más adelgazado que la posterior. La hembra mide entre tres y cuatro milímetros, en tanto que el macho es de menor tamaño. Originalmente, se reconocía como única especie a la Trichinella spiralis (Owens, 1835), pero en diversas áreas geográficas se han descrito recientemente triquinas que, aunque morfológicamente similares, presentan sutiles diferencias en sus características biológicas. Así, en la actualidad se distinguen:

1.

Trichinella spiralis, propia de las zonas geográficas templadas.

2.

Trichinella pseudospiralis, la cual, aunque no es frecuentemente observada afecta más a las aves que a los mamíferos y se caracteriza por ser de menor tamaño y por no formar quistes en la musculatura del hospedador.

3.

Trichinella nelsoni del Africa tropical, la cual se encuentra en los grandes carnívoros de la región, presenta un bajo grado de infectividad para los cerdos domésticos y ratas de laboratorio, y en el hombre provoca intensas infecciones con un gran número de larvas por gramo de músculo; aunque ha sido fatal en ocasiones, es muy bien tolerada a pesar de la masividad de la infección.

4.

Trichinella nativa de las zonas árticas que, distintivamente, presenta una considerable resistencia a la congelación, tiene bajo grado de infectividad para el cerdo doméstico, y en el hombre provoca importantes síntomas digestivos, principalmente diarreas prolongadas. Si bien T. spiralis, T. nelsoni y T. nativa son morfológicamente similares, se ha establecido la diferenciación entre ellas por sus características isoenzimáticas y mediante el uso de anticuerpos monoclonales. En cambio, existen diferencias estructurales entre esas tres "especies" y la T. pseudospiralis.

Ciclo doméstico: El cerdo adquiere la infección, principalmente, por la ingestión de ratas infectadas, lo que es posible cuando es criado en malas condiciones higiénicas o, simplemente, cuando debe buscar su propia fuente de alimentación en sitios eriazos o basurales; además, el cerdo se infecta con carnes de otros animales que encuentra en los criaderos o en los basurales. Las ratas, debido principalmente a sus hábitos de canibalismo, mantienen y propagan la infección en la naturaleza.

Los principales huéspedes domésticos de la T. spiralis son la rata, el cerdo y el hombre. El hombre adquiere la infección a través de la ingestión de carne de cerdo cruda o insuficientemente cocida, con larvas de triquina. Los jugos digestivos digieren la carne y las larvas quedan en libertad en el intestino, donde rápidamente, ya a las cuarenta y ocho horas, se diferencian en hembras y en machos adultos. Copulan en el lumen intestinal y, mientras los machos son eliminados con las deposiciones del huésped luego de cumplida su función genésica, las hembras grávidas - que son vivíparas- se localizan en el interior de la mucosa del duodeno y del yeyuno. Entre el tercero y el quinto día, comienza la postura de larvas. Cada hembra coloca alrededor de 1.500. Estas larvas miden entre 80 y 120 micrones, se profundizan en la mucosa intestinal, penetran a través de los capilares linfáticos y venosos y llegan a la circulación general, diseminándose por todo el organismo, pero enquistándose sólo en la musculatura, esquelética. Las larvas se localizan en el interior de las fibras musculares, destruyéndolas parcialmente; al cabo de unos quince días, quedan rodeadas por una envoltura constituida por el sarcolema. Así se origina el quiste larval, que mide entre 250 a 400 micrones y que, en consecuencia, no es visible a simple vista, aunque puede observarse con una lupa o con un microscopio de poco aumento. Tiene

un aspecto fusiforme o alargado, que recuerda la forma de un limón, y contiene, enrollado en su interior, una o varias larvas de triquina. La invasión de la musculatura esquelética por las larvas, comienza alrededor del séptimo día de ocurrida la infección y continúa mientras existan hembras grávidas en el intestino. Al cabo de un mes, las larvas completan su encapsulamiento y a los seis meses, se inicia el depósito de calcio en las paredes del quiste. La calcificación total se alcanza en un plazo aproximado de un año. En consecuencia, un mismo individuo es, sucesivamente, huésped definitivo e intermediario del parásito. Es hospedero definitivo cuando alberga en su intestino las formas adultas, y es intermediario cuando las larvas se localizan en su musculatura. Sin embargo, para completar todo su desarrollo, la Trichinella requiere siempre de dos huéspedes Referencias: 1. Leo X. Liu Peter F. Weller. Helminthic Infections: Trichinella and other tissue Nematodes. In Baunwald E, Faucy AS, Kasper DL, Hauser SL, Longo DL, Jameson JL, editors. Harrison´s Principles of Internal Medicine. 15 Th ed. New York . McGraww-Hill; 2001; 1231-3. 2. Pozio E. New patterns of Trichinella infections. Vet Parasitol 2001; 98: 133-48. 3. Murrel KD. Trichinellosis: now and forevermore?. Parasite 2001; 8 (2 supl.): 11-3. 4. Sánchez Rodríguez A, Martínez López de Letona J, Arias paciencia M, Sánchez García AM, Paz Bouza J, Jarrin J, et al. Triquinosis. Estudio de 21 casos en un mismo brote. Rev Clin Esp 1982; 165: 79-84. 5. Perucha González M, Lezaun Larrumbe ME, Torres Baile JL, Campo Hernández JM, Bernal Martínez A. Brote de Triquinosis en varias localidades de la Rioja Baja. Rev Sanid Hig Pública (Madrid) 1987; 61: 1035-47. 6. De la Torre Cecilia C, Espino Aguilar R, Cárdenas Talaverón C, Canuelo Ruiz O, Garrido Palomo R, Baena Sáez J, et al. Triquinosis: presentación de 2 casos. An Esp Pediatr 1989; 30: 227-8.

34.- Ante un paciente con diagnóstico de trastorno obsesivo-compulsivo, usted decide indicar el siguiente fármaco por ser el principal tratamiento de elección: a) b) c) d)

Antidepresivos inhibidores de la recaptación de serotonina. Antipsicóticos. Antidepresivos inhibidores de la recaptación de noradrenalina. Benzodiacepinas.

El abordaje farmacológico del TOC se basa en los inhibidores selectivos de la recaptura de serotonina (ISRS), medicamentos que han resultado ser efectivos y seguros. Como grupo, los ISRS son igual de eficaces que la clorimipramina, pero producen menos efectos secundarios y, por lo tanto, mejor tolerancia y mejor apego al tratamiento. La efectividad antiobsesiva parece ser independiente de su actividad antidepresiva. Entre los factores predictores de respuesta al tratameinto, se ha propuesto que los altos puntajes en las compulsiones predicen una mala respuesta al tratamiento. Para los casos

refractarios y resistentes o cuando hay síntomas de comorbilidad, se han utilizado, con relativo éxito, combinaciones con diferentes IRS, con benzodiacepinas, o bien, con potenciadores como el litio, o antipsicóticos, como la risperidona y el haloperidol. El uso del carbonato de litio es controvertido, aunque parece ser útil como potenciador a largo plazo entre 15 y 30% de los pacientes. La combinación de ISRS con antipsicóticos comenzó a utilizarse en los pacientes con síntomas psicóticos, aunque ahora se combinan también en los pacientes resistentes.

Tratamiento farmacológico del TOC Cristina Lóyzaga* Humberto Nicolini* *División de Investigaciones Clínicas. Instituto Nacional de Psiquiatría. Ramón de la Fuente. Calzada México-Xochimilco, 101, San Lorenzo Huipulco, 14370, México D.F. Primera versión: 14 de septiembre de 2000. Aceptado: 26 de septiembre de 2000.

35.- Femenino de 66 años que acude al servicio de neurología para valoración, al realizar exploración física, usted determina que éste, comprende pero no puede articular palabra, El tipo de afasia que presenta éste paciente es:

a) Wernike b) Broca c) Global d) Anómica

• • • • • • • •

Afasia De broca, motriz o de expresión (no fluente) De Wernicke (no comprende) De conducción (no denomina – no repite) Anómica (no denomina) Transcortical motora (no fluente) Transcortical sensitiva (no denomina – no comprende) Global (no fluente-no denomina-no repite-no comprende) Talámicas y putaminales (lenguaje espontaneo incomprensible)

Referencias: 1.Aronson A y cols. Examen clínico neurológico, 3ª Edición. La Prensa Médica Mexicana, México, 1995. 2. Uribe CS, Arana A, Pombo PL. Neurología, 5ª Edición. Corporación para investigaciones biológicas. Colombia, 1996. 3.-Adams R, Víctor M. Principles of Neurology. Mc Graw Hill. 7th ed. USA 2001.

4.-Bradley W.G. Neurology in clinical practice. Butterworth Heinemann. 4th ed. Philadelphia USA, 2004. 5.-Silberstein SD, Lipton RB, Goadsby PJ. Headache in clinical practice. Isis Medical Media. Oxford University Press, UK, 1998. 6.-Barinagarrementería F. Cantú C. Enfermedad vascular cerebral. McGrawHill Interamericana. México D.F. 1997.

36.- Masculino de 39 años, limpia vidrios, con antecedente de fractura vertebral de C6 y fractura del extremo distal del radio derecho tras caída de un tercer piso se presenta a consulta, refiere dolor, limitación de la movilidad y hormigueo en los dedos 2º y 3º de la mano derecha, de predominio nocturno, que mejora durante el día. ¿Cuál de los siguientes diagnósticos es el más probable?: a) b) c) d)

Radiculalgia C5-C6. Distrofia simpático refleja. Síndrome del túnel del carpo. Artrosis postraumática del carpo.

El Síndrome del Túnel Carpiano (STC) es una patología que afecta a la mano, provocada por una presión sobre el nervio mediano a nivel de la muñeca. Esto provoca síntomas como adormecimiento y hormigueos en la mano (especialmente en los dedos pulgar, índice, corazón y mitad del anular). Puede existir dolor, que puede estar limitado a la mano y muñeca, pero que en algunas ocasiones se irradia hacia el antebrazo. El STC con frecuencia despierta al paciente por la noche, y los síntomas pueden aparecer con actividades como conducir un vehículo, escribir, u otros ejercicios que suponen una utilización significativa de la mano. En el síndrome del túnel del carpo avanzado, puede producirse una pérdida de fuerza y una disminución de la masa muscular en la base del pulgar. CAUSA El túnel del carpo es un canal formado por los huesos de la muñeca y un ligamento (el ligamento transverso del carpo) situado en la cara palmar de la muñeca. Por este túnel transcurren todos los tendones que flexionan la muñeca y los dedos, y el nervio (nervio mediano) que recoge la sensibilidad del pulgar, índice, corazón y parte del anular y moviliza los músculos de la base del pulgar. Algunas personas nacen con túneles estrechos y por tanto están predispuestas a problemas de presión sobre el nervio. La utilización vigorosa de la mano, que conduce a una tendinitis de los tendones que flexionan el pulgar y los demás dedos, también puede conducir a un síndrome del túnel del carpo a través del engrosamiento de las vainas tendinosas. Las vainas engrosadas "rellenan" el túnel presionando sobre el nervio. Las personas con artritis reumatoide, hipotiroidismo, diabetes, amiloidosis, insuficiencia renal y algunos otros problemas médicos están más predispuestas a padecer este síndrome.

1.

Arthroshi I, Gummenson C, Johonsson R, Ornstein E, Ranstam J, Rossen I. Prevalence of carpal tunnel syndrome in a general population. JAMA. 1999;282:1538

2. Periódico El Público, 17 de octubre del 2007,pag 26 3. Durkan, JA. The carpal compression test: an instrumental device for diagnostic carpal tunnel syndrome. Lancet. 1990;335:393-5. 4. Marshall S, Tardif G. Injection local of steroids in the carpal syndrome. Cochrane Data Base of Systematic Reviews. 2005. Issue 5.

5. Verdugo RJ, Salinas RS, Castillo J, Cea JG. Tratamiento quirúrgico versus tratamiento no quirúrgico para el síndrome del túnel carpiano. Cochrane Data Base. 2005

37.- Se trata de paciente femenino de 43 años de edad con diagnóstico de polimiosistis al realizar la exploración física, la disminución de la fuerza muscular se caracteriza por:

a) b) c) d)

Ser de predominio distal Acompañarse de dolor intenso Ser de predominio proximal Afectar únicamente a la musculatura distal

POLIMIOSITIS CUADRO CLÍNICO • Inicio insidioso: 3-6 meses • Síntomas constitucionales • Debilidad muscular proximal y simétrica • Musculatura de la cintura escapular y pélvica (suele empezar en la pélvica); musculatura cervical • Respeta músculos oculares y faciales • Mialgias / Artralgias / Raynaud

La polimiositis (PD) y dematomiositis (DM), son enfermedades inflamatorias en las cuales el compromiso principal es la debilidad muscular, generalmente proximal y simétrica con atrofia subsecuente, generalmente indolora. Su etiología y patogenia siguen siendo desconocidas, siendo considerada dentro de las enfermedades difusas del tejido conectivo.

Numerosos estudios sugieren que una exposición previa a diversos virus como Influenzae A y B, Coxsackie virus tipo B y Picornavirus pueden desencadenar PM-DM; además se cree que los fenómenos inmunológicos juegan un papel preponderante, por la presencia de autoanticuerpos, depósito de inmunoglobulinas y complemento en las paredes vasculares. A nivel anatomo-patológico las principales alteraciones se encuentran en los músculos esqueléticos y en los vasos sanguíneos. Siendo la vasculitis un signo común en la dermatomiositis infantil y su severidad indicará una peor evolución. Clínicamente los síntomas iniciales son la debilidad muscular proximal, el rash típico, frecuente el edema en cara y a veces en miembros, fiebre moderada, posteriormente dificultad para la deglución y debilidad en la voz; asimismo manifestaciones viscerales que no se presentan siempre, pero son de gran importancia en cuanto al pronóstico, asimismo transtornos ventilatorios (pulmonares): por afectación primaria (neumonía intersticial) o secundaria a la disfunción faríngea (aspiración) o a la debilidad de los músculos respiratorios (insuficiencia ventilatoria), o como complicación del tratamiento (infecciones oportunistas).

Wortmann RL. Idiopathic inflammatory myopathies. A. Clinical features. In: Primer on the rheumatic diseases. New York: Springer-Arthritis Foundation; 2008. p. 363-7.

38.- Se trata de masculino de 19 años, que presenta hemartrosis. Tiene TPT alargado, TP y TT normales, por lo que tiene una alteración de la vía: a) Del Complemento b) Intrínseca c) Extrínseca d) Colinérgica

La vía intrínseca está constituida por la activación secuencial de los factores XII, XI, IX,. VIII y V. el tiempo de tromboplastina parcial activada mide la actividad de la coagulación intrínseca y sirve para monitorizar el tratamiento con la heparina no fraccionada.

39.- Masculino de 12 años que inicia con dolor intraescrotal intenso de aparición brusca al encontarse realizando actividad física, se tiene la sospecha de una torsión de testículo, ¿Cuál es, entre las siguientes, la prueba diagnóstica de elección?:

a) b) c) d)

Ecografía. Ecografía-doppler. Radiografía escrotal. Palpación bimanual.

Ecografía eco-doppler color: Es la prueba diagnóstica de elección al permitir la visualización de la vascularización testicular pudiendo distinguir si el flujo circulatorio es normal, ausente o aumentado. En la torsión lo normal es que esté reducido o ausente. Tiene una especificidad del 100% y una sensibilidad del 80%, aunque tiene sus limitaciones en las subtorsiones. Gammagrafía isotópica testicular con TC 99: Es también un método sensible y específico, pero del que no siempre se puede disponer en urgencias. Se verá un área de captación disminuida o ausente, rodeado de un área de captación aumentada por la hiperemia reactiva.

Bibliografía: 1) Sesions AE et al: Testicular torsión: direction, cadem, duration and desinformation. J. Urol 2003, 169: 663-665. 2) Cummings JM el al: Adult testicular torsión. J.Urol. 202, 167: 2109, 167: 2109-2110 3) Bedos F et al: Manual de Urología. Ed Masson SA. Barcelona 1996. 297-335.

40.- A través de un estudio de casos y controles se quiere conocer si existe asociación entre tabaquismo y cáncer de lengua. La variable tabaquismo se medirá como Leve (uno a 10 cigarrillo en 24 hrs), moderado (11 a 20 cigarrillos en 24hrs) severo (más de 21 cigarrillos en 24 hrs). Como podemos clasificar esta variable:

a) b) c) d)

Cuantitativa continua Cuantitativa discreta Cuantitativa discontinua Cualitativa ordinal

En las variables de tipo ordinal las observaciones se clasifican y ordenan por categorías según el grado en que los objetos o eventos poseen una determinada característica. Por ejemplo, se puede clasificar a las personas con respecto al grado de una enfermedad en leve, moderado o severo.

Moreno A. Principales medidas en epidemiología. Rev Salud Pública Mex, 2000;42( 4): 338

41.- Femenino de 27 años. Acude al servicio de urgencias por presentar salida de líquido vaginal. Antecedentes: G2, P1, cursa embarazo de 36 semanas de gestación exploración física: cervix con 10% de borramiento, 1 cm de dilatación y Tarnier positivo. ¿La complicación más frecuente en esta paciente es?

a)

Parto pre término.

b)

Sepsis neonatal.

c)

Corioamnioitis.

d)

Endometritis.

Corioamnioitis: El diagnóstico de la infección intraamniótica (IIA) es básicamente clínico. La corioamnionitis se debe descartar en toda gestante que presente fiebre sin foco aparente, sobre todo si se sospecha o se ha confirmado una rotura de membranas. Los criterios más empleados para el diagnóstico son: fiebre materna y, al menos, 2 de los siguientes signos: taquicardia materna, taquicardia fetal, irritabilidad uterina, leucocitosis materna o líquido amniótico purulento o maloliente. Progresos de obstetricia y ginecología: ISSN 0304-5013, Vol. 48, Nº. 6, 2005 , pags. 316317.

42.- Mujer de 26 años, con embarazo de término, sin antecedentes de control prenatal. G 3. C-1. Se ingresa al servicio de obstetricia por presentar actividad uterina regular y dolorosa. Ef.: Deambulante, tranquila, adecuada coloración de tegumentos, abdomen con fondo uterino a 32 cm. con producto único vivo en situación transversa dorso inferior FCF 144, al tacto vaginal cérvix dilatado a 3 cm. y membranas íntegras. Se realiza cesárea con retención de placenta e invasión a vejiga. La alteración placentaria que presenta esta paciente es: a) b) c) d)

Placenta increta Placenta acreta Placenta marginal Placenta percreta



Es la penetración y adherencia anormal de la placenta en la pared uterina.



Se divide en:  Placenta acreta.  Placenta increta.  Placenta percreta. 

ACRETA: Las vellosidades se adhieren al miometrio.



INCRETA: Penetran más de la mitad del espesor del miometrio.



PERCRETA: Atraviesa todo el espesor del miometrio, llegando a la serosa, incluso atravesándola y adhiriéndose a órganos vecinos.

Factores:  Endometriósis previa.

     

Tumores submucosos. (Miomas) Cicatríz uterina previa. (Cesárea, miomectomía) Implantación baja. (Placenta previa) Malformaciones placentarias. (Placenta extracorial) Legrado enérgico previo. Extracción manual previa de una placenta.

Diagnóstico transparto:  Placenta retenida por más de 20 minutos.  Imposibilidad para encontrar un plano de separación placentaria cuando se intenta su extracción manual.  Hemorragia incontrolable después de la pseudoextracción.  El diagnóstico histopatológico corrobora el diagnóstico clínico.  Escenario menos deseable. Tratamiento:  Histerectomía Obstétrica.  Constituye una cirugía no planeada y secundaria al hallazgo del acretismo placentario con sangrado incohercible. 

Cesárea-Histerectomía. (Con diagnóstico previo)  Cirugía planificada ante un correcto diagnóstico prenatal.



Recomendación ACOG:  Maduración pulmonar intrauterina.  Inyectar al cordón umbilical 50 mg de metrotexate.  Ligar el cordón en el nacimiento placentario y dejar la placenta in-situ.  Embolización inmediata de arterias uterinas bilaterales, así como de ramas de la división anterior de la arteria iliaca interna con alcohol polivinílico.  Continuar con 5 dosis I.M. de 50 mg de metrotexate y cuantificar niveles de βhCG.  Programar Histerectomía Total Radical Abdominal y/o Cistectomía parcial y/o resección pared anterior recto.

Lee et al. Conservative Management of Placenta Percreta. Obstet Gynecol, 112(2):421-424

43.- Lactante de un año que inició con un cuadro diarréico prolongado, recibió tratamiento con antibióticos, se reportó con mejoría clínica, posteriormente reinició con el cuadro enteral ¿El agente causal más probable en la recaída es? a) b) c) d)

Escherichia Coli Shigela Flexneri Clostridium Perfirgens. Clostridium Difficile

Clostrirum difficile es un bacilo anaerobio gram positivo, responsable de la colitis asociada a antibióticos. Coloniza el tracto digestivo después de que su flora habitual haya sido alterada por el tratamiento antibiótico. Tras la colonización, produce dos exotoxinas (toxina A y toxina B) que se unen al los receptores de las células epiteliales intestinales, conduciendo a la producción de una diarrea secretora. Es una de las infecciones nosocomiales más comunes, con una elevada morbimortalidad. La transmisión es fecal-oral y ha producido importantes epidemias dentro del ambiente hospitalario. No todas la cepas de Clostridium son productoras de enterotoxinas, en ocasiones se comportan como colonizantes del tracto digestivo sin producir enfermedad. Factores de riesgo: • Uso de antibioterapia: prácticamente todos las antibióticos pueden predisponer a la colonización por Clostridium difficile. • Asociación frecuente: Ampicilina, Amoxicilina, Cefalosporinas y Clindamicina • Asociación ocasional: otras Penicilinas, Sulfamidas, Eritromicina, Trimetropim y Quinolonas. • Raramente o nunca asociados: Aminoglucósidos, Tetraciclinas, Cloramfenicol, Metronidazol y Vancomicina. • Pacientes con edad avanzada y debilitados. • Intervención quirúrgica sobre tracto gastrointestinal. • Contacto previo con paciente colonizado. • Alimentación enteral. Manifestaciones clínicas: El espectro clínico abarca desde portadores asintomáticos hasta enfermedad fulminante con megacolon tóxico. Los síntomas más frecuentes son diarrea acuosa, dolor abdominal y fiebre. Las manifestaciones clínicas se inician entre 5-10 días después de la administración de antibióticos, aunque pueden aparecer el primer día o hasta 10 días después de haber cesado el tratamiento antibiótico. Colitis: • Diarrea profusa (más de 10 deposiciones al día) • Presencia de leucocitos en heces. • Fiebre, nauseas, anorexia, leucocitosis con desviación a la izquierda. • Dolor y distensión abdominal. • Colitis parcheada difusa en la sigmoidoscopia. Colitis pseudomembranosa: • Mayor número de deposiciones. • Presencia de leucocitosis y sangre en heces. • Fiebre, nauseas, anorexia, leucocitosis con desviación a la izquierda. • Más marcada distensión abdominal. • Placas adheridas, amarillentas de 2-10 mm en la sigmoidoscopia

BIBLIOGRAFIA: • Bartlett JG, Moon N, Chang TW, et al. The role of Clostridium difficile in antibioticassociated pseudomembranous colitis. Gastroenterology ; 75:778.

• Thomas C, Stevenson M, Riley TV. Antibiotics and hospital-acquired Clostridium difficileassociated diarrhea: a systematic review. J Antimicrob Chemother 2003; 51:1339. • Tedesco FJ. Pseudomembranous colitis: pathogenesis and therapy. Med Clin North Am ; 66:655. • Fekety R, Shah AB. Diagnosis and treatment of Clostridium difficile colitis. JAMA; 269:71.

44.- Lactante masculino de 8 meses de edad, con 8 Kg. de peso, antecedentes de ser sano, de manera súbita inicia con cólico intenso acompañado de sudoración y palidez, rechazo de alimentos, vómitos y evacuaciones mucosanguinolentas, a la E.F. Se palpa masa en colon transverso, la primera posibilidad diagnóstica es: a) b) c) d)

Apendicitis aguda. Invaginación intestinal. Divertículo de Meckel. Malrotación intestinal.

Invaginación intestinal Obstrucción intestinal aguda provocada por la introducción del tubo digestivo dentro de sí mismo. Clínica: lactante sano y bien nutrido, que de forma súbita, cada 10-15 minutos, durante unos minutos, presenta crisis de llanto, acompañadas de síntomas vagales (palidez, sudoración, decaimiento), irritabilidad y rechazo del alimento. Inicialmente permanece asintomático entre los episodios de llanto. De forma progresiva aparecen vómitos, anorexia, decaimiento, sangre roja en heces, e incluso colapso vascular y shock. Exploración física: •

Fosa iliaca derecha vacía. Signo de Dance (13%).



Palpacion de masa en hipondrio derecho y colon transverso “signo de morcilla” (24 a 90%)



Tacto rectal, “hocico de tenca” (0.5 al 3%)

Puede presentar sangre en el tacto rectal, excepcionalmente prolapso rectal de la invaginación. Rx. Simple:

aumento

de

ruidos

hidroaéreos

y

• • •

Patrón anormal de aire Opacidad en CID 25-60% Datos de oclusión



Distensión de asas



Niveles hidroaéreos



Ausencia de aire en recto



Tratamiento:



Ayuno.



Terapia hídrica.



Esquema Antimicrobiano. Ampicilina-Amikacina.



Sonda a derivación



Catéter Venoso.



Sonda Urinaria.

O’NEILL PEDIATRIC SURGERY 1998 URG. PED. HIM 5 ED 2002 OLDHAM SURGERY OF INFANTS AND CHILDREN. 1997

45.- Hombre de 72 años, diagnosticado de carcinoma de próstata, en este momento acude al servicio de urgencias por presentar confusión mental, náuseas, vómitos y estreñimiento. Se realizan pruebas de laboratorio y destaca una calcemia de 16mg/Dl. La primera medida terapéutica que debe tomarse ante éste paciente es: a) b) c) d)

Administración de Solución salina y furosemida por vía intravenosa. Hormonoterapia (leuprolide y estrógenos). Difosfonatos por vía oral. Glucocorticoides por vía intravenosa.

Los síntomas que provoca la hipercalcemia están en relación con sus niveles en sangre. Valores comprendidos entre 10,5 y 12 gr/dl no suelen provocar síntomas; a partir de estos valores la sintomatología es progresiva, afectando con más intensidad y gravedad a todos

los sistemas del organismo. No está establecido un orden de aparición de la sintomatología, ni su correspondencia con los valores de calcemia. Síntomas de la hipercalcemia Sistema nervioso central Desórdenes mentales Dificultades cognitivas Ansiedad Depresión Confusión, estupor y coma Calcificación corneal Suicidios (descritos aisladamente) Sistema neuro-muscular Fatiga o cansancio muscular Mialgias Descenso de la función de músculos respiratorios Laxitud articular Sistema renal Nefrolitiasis Diabetes insípida nefrogénica (poliuria y polidipsia) Deshidratación Nefrocalcinosis Sistema gastrointestinal Náuseas y vómitos Anorexia Estreñimiento Dolor abdominal Pancreatitis Úlcera péptica Sistema esquelético Dolor óseo Artritis Osteoporosis Osteítis fibrosa quística Resorción subperióstica Quistes óseos Embarazo Hipoparatiroidismo neonatal Tetania neonatal Bajo peso al nacer Retraso crecimiento intrauterino Hiperemesis gravídica Alta morbilidad neonatal y materna Partos pretérmino Sistema cardiovascular Hipertensión arterial Calcificación vascular Calcificación miocárdica

Otros

Hipertrofia miocárdica Acortamiento intervalo QT Arritmias cardíacas Queratitis Conjuntivitis Anemia normocítica normocrómica Gota o pseudogota

El tratamiento de la hipercalcemia está orientado fundamentalmente a eliminar la causa que la produce. En los casos sintomáticos se requiere de un tratamiento inicial específico. Debe ser tratada la que presenta síntomas o supera los 14 mg/dl. Los mecanismos para actuar sobre la hipercalcemia son: Aumento de la eliminación renal de calcio. Disminución de la absorción intestinal de calcio. Disminución de la resorción ósea de calcio. El tratamiento siempre debe iniciarse con la rehidratación del paciente, que produce un aumento del volumen extracelular, alcanzando una ganancia de volumen de 1,5- 2,5 litros en las primeras 24 horas. A continuación, se utilizará un diurético de asa, tipo furosemida, que aumenta la excreción renal de sodio y calcio. La dosis de diurético se ajustará en función de las cifras de calcio a lo largo del tratamiento. Cuando la función renal está comprometida puede recurrirse a la hemodiálisis con calcio bajo en el líquido de diálisis. En los casos en los que esté implicado un aumento de la resorción ósea como causa de hipercalcemia, deberemos controlarla con: Bisfosfonatos: disponemos de clodronato y pamidronato. El más empleado es el primero, porque la dosificación es más fácil. Su administración en el caso de la hipercalcemia es intravenosa, inicia el efecto a las 72 horas y alcanza el máximo a la semana. Calcitonina: actúa más rápido que los bisfosfonatos, pero sólo mantiene el efecto durante 72 horas.

Otros fármacos antirresortivos: Mitramicina y el nitrato de galio, con menor experiencia, pueden usarse en caso de fracaso de los anteriores. En aquellos casos en los que está aumentada la absorción intestinal de calcio, como en la producción endógena de vitamina D (enfermedades granulomatosas o linfomas) deben tratarse con glucocorticoides. LECTURAS RECOMENDADAS 1. Achogue HJ, Madias EN. Changes in plasma potassium concentration during acute acid base disturbances. Am J Med 71:456, 2. Humphreys MH. Urgencias por trastornos en los líquidos, electrolitos y equilibrio ácido básico. En: Diagnóstico y Tratamiento de Urgencias. Editado por MT Ho y CE Saunders. Editorial El Manual Moderno. México DF, 3. Mora JM, Delgado VA, D'Achiardi R. Trastornos del potasio. En: Manual de Urgencias en Medicina Interna. Capítulo Central. .

46.- Se trata de mujer de 44 años, acude a consulta por presentar cefalea frontal de predominio matutino. Antecedentes ginecoobstétricos: ciclos menstruales irregulares. E.f.: estatura 1.56 m, peso 82 kg, T/a 140/100, glucemia en ayunas de 120 mg/dl, triglicéridos 245 mg/dl, Hdl 37, colesterol total 320 mg/dl. El diagnóstico es: a) b) c) d)

Obesidad mórbida. Síndrome metabólico. Hipertensión arterial esencial. Síndrome premenopáusico.

Referencia:

OMS DM o glucosa de ayuno anormal o intolerancia a la glucosa o resistencia a la insulina y 2 más:

Sx metabólico Predominio de adipocitos viscerales Adipocitos metabólicamente activos

Glucocorticoides inactivos

Glucocorticoides activos

Menos leptina

lipotoxicidad

Resistencia a la insulina hiperglucemia

47.- Masculino de 20 años de edad, acude a su clínica un lunes por la mañana, el motivo de consulta es la presencia de más de 8 evacuaciones líquidas con sangre, moco y pujo, las últimas 24 horas, precedidas de dolor tipo cólico que cede posterior a la defecación, además de tenesmo rectal. Llama la atención que este cuadro inicio hace dos semanas, al principio solo existió disminución de la consistencia de la materia fecal, hasta que, alrededor del cuarto día, solo había evacuaciones líquidas. Afebril y sin vómitos. La exploración física demuestra mucosas orales humectadas. Dolor a la palpación de todo el abdomen, sin rebote, con ruidos intestinales intensos y abundantes. Su presión arterial es de 100/70. El diagnóstico más probable es:

a) b) c) d)

Diarrea crónica. Colitis por parásitos. Enfermedad intestinal inflamatoria. Diarrea aguda

Definición:

Se considera diarrea aguda a la presencia de heces líquidas o acuosas, generalmente en número mayor de tres en 24 horas y que duran menos de 14 días; la disminución de la consistencia es más importante que la frecuencia.1 El número de evacuaciones intestinales hechas en un día varía según la dieta y la edad de la persona. Los lactantes alimentados al seno materno tienen evacuaciones intestinales blandas frecuentes; ésto no es diarrea.

Epidemiología

La Organización Mundial de la Salud estima que cada año se presentan 1,300 millones de episodios de diarrea en niños menores de cinco años en países en desarrollo (África, Asia, excluida China, y América Latina), que ocasionan 4 millones de muertes, relacionadas en el 50-70% con deshidratación, lo que las ubica dentro de las principales causas de defunción en estos países. La mayoría de los niños que sobreviven quedan con algún grado de desnutrición y los desnutridos, no sólo padecen con mayor frecuencia de diarrea, sino que los episodios son más graves.3 El tercer gran problema asociado a las diarreas, en niños mayores, es el ausentismo escolar o laboral.

Etiopatogénia

Los agentes etiológicos más frecuentes son, en orden decreciente, virus, bacterias y parásitos. Los virus son la causa principal de las diarreas deshidratantes en niños menores de dos años, siendo los rotavirus del grupo A, serotipos G1 y G3, los responsables de la mayoría de los episodios.8 La diarrea osmótica que ocasionan se debe a que lesionan en forma focal las células de las vellosidades del intestino delgado, disminuyendo la producción de las enzimas encargadas de la absorción de la lactosa, entre otros disacáridos, lo que aumenta la osmolaridad en la luz intestinal y produce mayor secreción de agua que se pierde a través de las heces. Sin embargo, las células de las criptas encargadas de reparar las vellosidades lesionadas, migran para substituirlas en un periodo de 24 a 72 horas, con lo que desaparece la diarrea.9 Las enterobacterias, como Escherichia coli, Salmonella sp, Shigella, Campylobacter jejuni y Vibrio cholerae 01, producen diarrea a través de diversos mecanismos: 1. Liberación de enterotoxinas (V. cholerae 01, E. coli enterotoxigénica) que estimulan la adenilciclasa y aumentan la secreción intestinal de agua, sodio y cloro; 2. Enteroinvasión (E. coli enterohemorrágica) con disolución de la mucosa y del borde en cepillo y 3. Proliferación intracelular, previa invasión de la mucosa ( Shigella) con aparición de sangre en las evacuaciones, paso de microorganismos a la circulación sanguínea (bacteremia) y algunas veces sepsis. Shigella dysenteriae, produce además enterotoxinas que estimulan la secreción de agua y sodio en el intestino delgado y neurotoxinas que causan manifestaciones neurológicas, desde convulsiones hasta estado de coma. En los dos últimos decenios ha sido posible documentar el papel de otros patógenos como Campylobacter y Yersinia. Asimismo, se ha avanzado en los mecanismos fisio-patogénicos de las infecciones producidas por algunos tipos de Escherichia coli, Clostridium difficile, Salmonella y

Aeromonas hydrophila.9

Algunos parásitos producen diarrea sanguinolenta (Entamoeba histolytica) o diarrea prolongada (Giardia lamblia). El mecanismo de producción de diarrea es a través de enteroinvasión (E. histolytica) o enteroadhesión (G. lamblia), en donde el daño más grave es de carácter nutricional. El reservorio de los agentes de las diarreas infecciosas es el hombre, exceptuando el caso de Salmonella que es de origen animal. La fuente de infección es la materia fecal del hombre infectado, sintomático o asintomático, siendo este último más peligroso ya que no

presenta datos clínicos que permitan identificarlo; en el caso de algunos virus, las secreciones nasofaríngeas pueden ser el origen. La infección genera inmunidad específica, la cual es de duración prolongada en la etiología viral y más corta en la bacteriana. El mecanismo de transmisión clásico es el ciclo ano-boca, entendido éste como la salida del agente infeccioso con la materia fecal y su ingestión, casi siempre a través de las manos contaminadas o de fomites. Otro mecanismo de transmisión frecuente es la ingestión de alimentos contaminados o de agua, otras bebidas o hielo, no sometidos a algún proceso de purificación. Algunos factores de riesgo en el huésped son: el estado nutricional (círculo vicioso diarrea-desnutrición-diarrea); enfermedades previas de tipo anergizante (sarampión); ablactación temprana o ausencia de alimentación al pecho materno; saneamiento deficiente (carencia de agua potable, fecalismo); falta de educación y hábitos higiénicos; ignorancia o patrones culturales adversos (suspensión de alimentos, no aceptación de medidas sanitarias y de manejo oportuno del paciente), y económicos (limitantes a la incorporación de obras de saneamiento, adquisición de nutrientes y búsqueda de atención médica).

Referencias 1. World Health Organization. A manual for the treatment of acute diarrhoea for use by physicians and other senior health workers. Geneve: Program for Control of Diarrhoeal Diseases, World Health Organization, WHO/CDD/SER/80.2 Rev,2:1990. 2. World Health Organization. Division of Diarrhoeal and Acute Respiratory Disease Control. 25 years of ORS: Joint VMO/ ICDDR,B Consultative Meeting on ORS Formulation. Dhaka, Bangladesh, 10-12 December 1994. CDR/CDD/95.2 3. Sepúlveda J, Willet W, Muñoz A. Malnutrition and diarrhea. Alongitudinal study among urban mexican children. Am J Epidemiol 1988; 127: 365-376. 4. Mota HF, Tapia CR, Welti C, Franco A, Gómez UJ, Garrido MT. Manejo de la enfermedad diarreica en el hogar, en algunas regiones de México. Bol Med Hosp Infant Mex 1993; 50: 367-75. 5. Encuesta de manejo efectivo de casos de diarrea en el hogar. Dirección General de Epidemiología, SSA. Consejo Nacional para el Control de las Enfermedades Diarreicas. México, 1993 (Documento interno). 6. Glass RI, Lew JF, Gangarosa RE, Lebaron CW, Ho MS. Estimates of morbidity and mortality rates for diarrheal disease in American children. J Pediatr 1991; 118: 527-33

48.- Masculino de 35 años con datos clínicos altamente sugestivos para diagnóstico de íleo mecánico simple, los datos radiográficos de esta patología son: a) Retención de contenido en colon, sin niveles hidroaereos b) Liquido y aire libres en cavidad peritoneal

c) Niveles hidroaéreos a diferentes alturas, con dilatación de asas e) Asas fijas y lisas, con niveles largos

ILEO MECANICO A/ Extraluminal Adherencias postquirúrgicas (Causa más frecuente 35-40%)  Hernias externas (inguinales, crurales, umbilicales, laparotómicas, etc.)  Hernias internas Torsiones Vólvulos Invaginaciones  Efecto masa extraluminal (tumoración, masa inflamatoria ó abceso) B/ Parietal Neoplasias Alteraciones congénitas (atresias, estenosis, duplicaciones, etc.) Procesos inflamatorios (Crohn, postradiación, etc.) C/ Intraluminal Ileo biliar Bezoar Parasitosis Cuerpo extraño Impactación fecal Tumoraciones Íleo mecánico Los signos radiológicos de una obstrucción intestinal simple son: • • •

Asas distendidas proximales por retención de líquido y gas. Niveles hidroaéreos. Reducción o ausencia de gas y materia fecal en colon. Ante el hallazgo de gas en colon se descarta la existencia de una obstrucción de intestino delgado.

BIBLIOGRAFÍA 1) Di Lorenzo C. Pseudo-obstrucction. Current approaches. Gastroenterology 1999;116:980987. 2) Stanghellini V, Cammilleri M, Malagelada JR.Chronic idiopatic intestinal pseudobstruction: clinical and intestinal manometric findings. Gut 1987; 28: 5-12. 3) Scolapio JS, Ukleja A, Bouras EP et al. Nutritional management of chronic intestinal pseudo-obstruction. J Clin Gastroenterol 1991; 28:306-312. 4) Malagelada JR, Distrutti E. Managementof gastrointestinal motility disorders. A practical guide to drug selection and appropiate ancillary measures. Drugs 1996;52:494506.

49.- Recién nacido que presenta una conjuntivitis neonatal, para indicar el tratamiento usted relaciona los siguientes gérmenes por ser los de mayor frecuencia causantes de esta enfermedad: a) b) c) d)

Clamidia, treponema, gonococo Estreptococo grupo A, listeria Clamidia, gonococo, estafilococo aureus Listeria, pseudomonas

Las conjuntivitis neonatales (CN) siguen siendo uno de los motivos más frecuentes de consulta, pudiendo llegar a comprometer seriamente la capacidad visual del bebé, conduciendo incluso a la ceguera. La incidencia de esta enfermedad oscila entre 1,6%-12% en el primer mes de vida. Las CN pueden ser de origen infeccioso o químico. Las CN de origen infeccioso pueden ser adquiridas durante la gestación, durante el parto por contacto con la secreción uretrovaginal, o bien en el post-parto y el ámbito de convivencia diaria. Las CN que se manifiestan entre las 24 y las 48 horas del nacimiento son de origen químico, debidas a la profilaxis efectuada por instilación de solución de nitrato de plata en el saco conjuntival del neonato, para prevenir la proliferación de Neisseria gonorrhoeae.

Se debe tener presente la adquisición de la infección perinatal a partir del ambiente en que se halla el bebé. La etiología más frecuente en este caso corresponde a Staphylococcus aureus, Haemophilus influenzae y Streptococcus pneumoniae). En las últimas décadas, Chlamydia trachomatis ha resultado ser el microorganismo prevalente entre los agentes causales de enfermedades de transmisión sexual, con una prevalencia del 2 al 35% en embarazadas. La transmisión vertical se da entre el 60 y 70% en los hijos de madres infectadas .El riesgo de adquirir conjuntivitis en estos bebés es del 18 al 60%, mientras que el de neumonía es del 10 al 25%. En el caso de N. gonorrhoeae , la infección ocular puede complicar el cuadro con ceguera y artritis. Desde el año 1995, en nuestro hospital se lleva a cabo en forma rutinaria la búsqueda de gérmenes comunes (GC) y C. trachomatis en todo recién nacido con conjuntivitis.

Bibliografía: Krachmer, Manis, Holland. Cornea and External Disease: Clinical Diagnosis and Management, Vol II. Mosby, 1997. Yanoff, Ophtalmology. Mosby, 1999.

50.- Mujer de 74 años acude a consulta externa por referir debilidad general y apatía, añadiéndose en la última semana disnea progresiva hasta ser de pequeños esfuerzos.

Antecedentes: cardiopatía hipertensiva con función sistólica conservada, en los últimos 2 años ha tenido 3 episodios de fibrilación paroxística cardiovertidos eléctricamente. Durante este tiempo ha recibido diversos tratamientos que incluían algunos de los siguientes fármacos: propafenona, amiodarona, digoxina, diltiacem y captopril. El ECG muestra fibrilación auricular con frecuencia ventricular a 130 lpm, RX de tórax cardiomegalia signos de congestión pulmonar y el estudio de función tiroidea una T4 libre elevada con una TSH indetectable. El medicamento responsable de la sintomatología de esta paciente es: a) Propafenona. b) Digoxina. c) Amiodarona. d) Diltiacem.

Alteraciones de lab por amiodarona: Hepáticos (1,2%) Elevación del ASAT y ALAT 2-3 veces valor normal Elevaciones persistentes de LDH y Fosfatasa alcalina · Gástricos (4,2%) Naúseas. Vómitos. Estreñimiento — · Pulmonares (1,9%) * Síntomas respiratorios y nuevas alteraciones en Rx de tórax > 30% de reducción en difusión pulmonar. (PFR) · Tiroides (3,7%) Hipertiroidismo o hipotiroidismo clínico Cambios en la función tiroidea (TSH, T4, T3) que requiere medicación · Neurológicos (4,6%) Temblor. Ataxia. Disestesias Insomnio · Piel (2,3%) Fotosensibilidad; coloración azul/grisácea (piel) · Ojos (1,5%) Alteraciones visuales. Halos visuales Visión nocturna borrosa. Depósitos corneales en la lámpara de hendidura. · Bradicardia/Conducción (3,3%) Bradicardia sintomática. Bloqueo A-V de 3er grado Bradicardia < 50 lpm asintomática. Bloqueo A-V de 1er y 2º grado · Supresión del fármaco (23%) ® Por efectos adversos no tolerados, decisión del paciente o no cumplimentación de prescripción farmacológica (*) Se relaciona con la dosis (más frecuente en dosis >400 mg/día) y duración del tratamiento.

Volumen 13, Número 4 Octubre - Diciembre 2002 pp 149 – 152 Cardiología Trabajo de investigación * Cardiólogo, Ex-Residente Hospital de Especialidades, Centro Médico Nacional “La Raza”, IMSS. Hospital de la Fe, San Miguel Allende, Gto. ** Jefe del Servicio de Cardiología, Hospital de Especialidades Centro Médico Nacional “La Raza”, IMSS.

51.- Masculino de 40 años, acude a consulta por presentar de manera espontánea una erupción de lesiones pruriginosas, habonosas, diseminadas por toda la superficie corporal, de evolución fugaz así como una amplia placa edematosa en hemicara izquierda. El diagnóstico más probable es: a) b) c) d)

Angioedema hereditario. Urticaria aguda. Erisipela. Vasculitis.

La urticaria se define como un síndrome reaccional de piel y mucosas caracterizado por edema y ronchas pruriginosas ocasionadas por edema vasomotor transitorio y circunscrito de la dermis que dura algunas horas; puede ser recidivante y de origen inmunológico, no inmunológico o desconocido.1 La lesión elemental es una pápula edematosa dérmica (roncha).2 El angioedema se manifiesta típicamente como un edema asimétrico causado por la presencia de plasma dentro de tejido celular subcutáneo y mucosas.1 Clasificación La urticaria se puede clasificar de acuerdo con diferentes parámetros: 1) según la evolución: en aguda o crónica; 2) según el cuadro clínico: en urticaria ordinaria (urticaria propiamente dicha), urticaria física (por estímulo detonador), urticaria por contacto (inducida por un contacto químico o biológico) y angioedema (sin ronchas), en el cual el espectro de las manifestaciones clínicas de los diferentes tipos es muy amplio; y 3) según el mecanismo potencial de su desarrollo: inmunológico, no inmunológico, mediada por el complemento, o bien urticaria autoinmune.3 Clasificación por evolución La urticaria aguda se define tradicionalmente por la presencia de ronchas de forma espontánea, casi la mayoría de los días, por menos de seis semanas.4 La urticaria crónica se define como la presencia de ronchas de forma espontánea por más de seis semanas, diariamente o casi la mayoría de los días de la semana. BIBLIOGRAFIA 1- Yadav S, Upadhyay A, Bajaj A. Chronic urticaria: An overview. IJD [en línea] 2006 [fecha de acceso 4 de marzo de 2007];51-3:171-177. Disponible en:

2- Woscoff A, Kaminsky A, Marini M, Allevato M. Dermatología en Medicina Interna. Buenos Aires, Edición de los Autores, 2003: 24-27. 3- Criado PR, Fachini Jardim Criado R, Maruta C, Costa Martins JE, Rivitti E. Urticaria. An.Bras.Dermatol 2005,v.80 n.6: 183-185.

52.- Female patient 23, who will have an operation with a history of hepatitis. Which of the following anesthetics are contraindicated in your case?

a) b) c) d)

Lidocaine Fentanyl Halothane Ketamine

El halotano reduce el flujo sanguíneo esplácnico y hepático como consecuencia de una reducción de la presión de riego. Puede producir necrosis hepática fulminate, que se caracteriza por fiebre, anorexia, náusea y vómito que pueden durar varios días después de la anestesia y acompañarse por un exantema y eosinofilia periférica. Este síndrome recibe el nombre de hepatitis por halotano. Hardman J, Limbird L. Goodman and Gilman Las bases farmacológicas de la terapéutica. Mc Graw Hill 2002. 361.

53.- Se trata de paciente femenino que cursa con diagnóstico de Salmonella Typhy. ¿Cuál de los siguientes cultivos permite aislar con mayor frecuencia éste agente causal?

a) b) c) d)

Hemocultivo Coprocultivo Mielocultivo Urocultivo

DIAGNOSTICO DE LABORATORIO Dadas las variadas manifestaciones clínicas de las salmonelosis, la confirmación del diagnóstico de estas infecciones, requiere de métodos microbiológicos que permitan el aislamiento o identificación del agente causal o de pruebas serológicas que facilitan reconocer anticuerpos específicos presentes en el suero de los pacientes. Hemocultivo: es el procedimiento de elección, cuando se realiza apropiadamente y en medios selectivos a base de bilis. Coincidiendo con la fisiopatología de la infección, son positivos especialmente durante la primera semana de la infección; se calcula que al final de la tercera semana de positividad solamente alcanza un 50%.

Mielocultivo: el cultivo del aspirado de médula ósea se considera como el mejor método para el aislamiento de salmonella en los pacientes con fiebre tifoidea y paratifoidea. Aunque el procedimiento produce una molestia transitoria, en general es bien tolerado y los cultivos son más rápidamente positivos. Se recomienda sea practicado por personal con experiencia. Pueden ser positivos aún cuando los hemocultivos sean negativos. Coprocultivo: puede ser positivo desde el comienzo de la infección, aunque su máxima positividad en la infección aguda, se observa durante la tercera semana. Es particularmente útil para el control postratamiento de los pacientes y para detectar los portadores crónicos. Cultivo de bilis duodenal: obtenido por aspiración o utilizando la técnica que lleva un dispositivo en cápsulas de gelatina. No es superior al hemocultivo y con certeza no supera a la asociación del hemocultivo con el coprocultivo. Urocultivo: su valor diagnóstico es muy limitado pues la bacteriuria no es continua. Su máxima positividad está en la tercera semana. La Salmonella también puede ser aislada de otros productos como las manchas rosadas o reoseolas tíficas, de la secreción bronquial, del líquido articular, etc. DIAGNOSTICO SEROLOGICO Reacción de seroaglutinación (Widal): es de poco valor como prueba diagnóstica. En la infección no tratada sólo cerca del 50% de los pacientes pueden tener un aumento significativo de las aglutininas contra el antígeno "O", en algún momento de la enfermedad. Las aglutininas contra el antígeno "H" no tienen valor diagnóstico aunque puedan observarse títulos elevados de ellas. En muchos casos de fiebre tifoidea no hay elevación de los títulos de aglutininas durante el curso de la infección y en ocasiones se pueden observar elevaciones no específicas, debido a reacciones cruzadas. Diagnóstico inmunoenzimático: la detección de anticuerpos IgM e IgG contra el lipopolisacarido por técnica ELISA aún no está disponible para uso rutinario. Con fines de investigación se han utilizado otras pruebas dentro de las cuales están la reacción de polimerasa en cadena (PCR), las pruebas de fagotipificación, las de susceptibilidad antimicrobiana y la investigación del perfil plasmídico de algunas cepas. En los estudios epidemiológicos se usan las pruebas de fagotipificación, de susceptibilidad contra los antimicrobianos y el perfil plasmídico, las cuales han demostrado ser útiles y complementarias para el estudio de cepas aisladas de alimentos, o de aguas contaminadas, y en brotes de salmonelosis en los cuales se requiere establecer una fuente común de infección.

LECTURAS RECOMENDADAS

Borrego JJ, Castro D, Jiménez Notorio M, Luque A, Rodríguez Avial C, Picazo JJ. Comparison of epidemiological markers of salmonella strains isolated from differents sources in Spain. J Clin Microbiol 30: 3058, 1992. De Los Rios O, Restrepo J, Carvajal CD. Salmonelosis: Revición de conceptos. Comportamiento epidemiológico en Antioquia. Bol Epidenmol Antioquia 7:19 Edelman R, Levine MM. Sumary of an international workshop on typhoid fever. Rev Inf Dis 8:329, 1996. Prada G. Infecciones por especies de Salmonella. En: Medicina Interna. Chalem, Escandón, Campos, Esguerra editores. Fundación Instituto de Reumatología e Inmunología. Editorial Presencia. Santafé de Bogotá. Tacket C. Molecular epidemiology of salmonella. Epidemiologic Reviews 11:99.

54.- Mujer de 30 años de edad, que acude a consulta por presentar dolor en el recto, sin encontrarse una causa orgánica después de múltiples exploraciones médicas. Además refiere que desde hace 4 años ha presentado vómitos, dolor abdominal generalizado, palpitaciones, mareos, disfagia, visión borrosa, dolor en los miembros inferiores, dismenorrea y dispareunia. Se observa triste y ansiosa. El diagnóstico más probable es:

a) b) c) d)

Trastorno de somatización Trastorno de ansiedad crónico Depresión crónica con somatización Trastorno de conversión

López- Ibor J J, Valdés M M. Manual diagnóstico y estadístico de los trastornos mentales (DSM- IV). Masson 2005. 545 – 574. El Trastorno de somatización se caracteriza por la presencia de muchos síntomas somáticos que no pueden explicarse por los hallazgos físicos o de laboratorio. Comienza antes de los 30 años, puede perdurar durante años, es crónico y va asociado a malestar psicológico, a un deterioro del funcionamiento social y laboral y a la búsqueda excesiva de

ayuda médica. Para hacer el diagnóstico deben presentarse dolor en cuatro zonas del cuerpo, dos síntomas gastrointestinales, un síntoma sexual y un síntoma pseudoneurológico. La ansiedad y la depresión son las patologías psiquiátricas más prevalentes.

55.- Hombre de 67 años, el cual ingresa al servicio de urgencias quejándose de dolor torácico de 3 horas de evolución, con diaforesis, palidez. El EKG muestra elevación del segmento ST en la cara inferior. Cual de las siguientes es una contraindicación absoluta para trombolisis:

a) Antecedente de evento vascular cerebral hemorrágico b) Enfermedad convulsiva crónica c) Enfermedad de Parkinson temprana sin demencia d) Historia de traumatismo hace 1 año en accidente vehicular

CONTRAINDICACIONES A LA TROMBOLISIS Absolutas

Relativas

Antecedente de AVE hemorrágico - Aneurisma disecante. - Diátesis hemorrágica. - Hemorragia digestiva en el mes precedente. - Cirugía o traumatismo reciente (últimas 3 semanas).

- Tratamiento anticoagulante. - Hipertensión arterial refractaria > 180 /110 mmHg. - Maniobras de resucitación cardiopulmonar prolongadas. - Embarazo. - Punción de vaso en sitio no compresible (vena subclavia). - AVE isquémico en los últimos 6 meses.

Como se explicó previamente, a mayor precocidad de terapia, se logra una mayor reducción de la mortalidad. Por ejemplo, si logramos reperfundir a un paciente con menos de 3 horas de evolución, la mortalidad cae en un 50% en comparación con la terapia realizada en un paciente con 12 horas de evolución, en el cual la mortalidad cae sólo en un 10%. Otro factor que influye en la disminución de la mortalidad en los pacientes tratados con trombolisis es la extensión del infarto, ya que a mayor isquemia hay mayor disminución de mortalidad.

BIBLIOGRAFIA 1. Overview of the management of suspected myocardial infarction. Reeder G.S., Kennedy H.S., Rosenson R.S. UpToDate v9.1, 2001 2. Diagnosis of acute myocardial infarction with biomarkers of cardiac injury. Jaffe Allan S. UpToDate v9.1, 2001 3. Braunwald: Heart Disease: A Textbook of Cardiovascular Medicine, 6th ed., 1114-1207. W. B. Saunders Company, 2001 4. Rakel: Conn's Current Therapy 2001, 53rd ed., 335-348, W. B. Saunders Company, 2001 5. Rosen: Emergency Medicine: Concepts and Clinical Practice, 4th ed., 1655-1979. MosbyYear Book, Inc., 1998 6. Futterman L.G., Lemberg L. Update on management of acute myocardial infarction: facilitated percutaneous coronary intervention. American journal of critical, 2000;9:70-6. 7. Wright R.S., Kopecky S.L., Reeder G.S. Update on intravenous fibrinolytic therapy for acute myocardial infarction. Mayo Clinic proceedings, 2000;75:1185-91.

56.- El signo clínico que indica irritación del nervio ciático como componente de una lumbalgia se denomina:

a) b) c) d)

Signo Signo Signo Signo

de Galeazzi de McMurray de Lassegue de Filkestein

La lumbalgia es una patología muy frecuente en nuestro medio, con una enorme repercusión sanitaria y sociolaboral. Con la edad, se inicia una cascada degenerativa en la columna lumbar, que comienza en el disco intervertebral, continuando por las facetas articulares y demás elementos vertebrales. Esta degeneración del raquis forma parte del envejecimiento normal del individuo, aunque en ocasiones puede causar dolor y/o alteraciones neurológicas. Para comprender la fisiopatología del dolor lumbar, será preciso conocer que la inervación de la columna lumbar se hace fundamentalmente por tres ramos nerviosos: ramo dorsal de los nervios espinales lumbares (o rami dorsal), nervio sinuvertebral de Luschka y ramos ventrales de la cadena simpática. Existen dos tipos de patrones de dolor en la columna lumbar: el dolor irradiado y el dolor referido. La cascada de la degeneración consta en tres estadios. El primer estadio sería la disfunción. El anillo fibroso se fisura y pierde la capacidad de contener al núcleo pulposo. Esto ocasiona primero el síndrome de disrupción discal y, si el núcleo supera el contorno del annulus, las hernias discales. El segundo estadio de Kirkaldy-Willis es el de inestabilidad. En este estadio la movilidad en el segmento móvil aumenta de forma patológica.

Maniobra de Lassègue: Es la más importante, es casi patognomónica. Con el paciente en decúbito supino se levanta la pierna extendida. Se considera positiva cuando aparece un dolor intenso a nivel lumbar, en la pierna o en ambos niveles, entre los 30º-75º de flexión en la cadera, debido al estiramiento del nervio ciático. Pasados los 70º

puede aparecer un falso positivo (dolor también en sujetos sanos) por la distensión de los isquiotibales. Para descartarlo realizaremos otras maniobras diagnósticas.

Referencias: 1. Andersson GBJ. Epidemiologic features of chronic low-back pain. Lancet. 1999;354:5815. 2. Estudio EPISER. Sociedad Española de Reumatología. 2000. 3. Herrera Rodríguez A, Rodríguez Vela J. Estenosis de canal lumbar. Rev Ortop Traumatol. 2002;4:351-72. 4. MacGregor AJ, Andrew T, Sambrook PN, Spector TD. Structural, psychological, and genetic influences on low back and neck pain: a study of adult female twins. Arthritis Rheum. 2004;51:160-7. 5. Kim KS, Yoon ST, Park JS, Li J, Park MS, Hutton WC. Inhibition of proteoglycan and type II collagen synthesis of disc nucleus cells by nicotine. J Neurosurg Spine. 2003;99:291-7. 6. Fujiwara A, Tamai K, Yamato M, An HS, Yoshida H, Saotome K, et al. The relationship between facet joint osteoarthritis and disc degeneration of the lumbar spine: an MRI study. Eur Spine J. 1999;8:396-401. 7. Eyre DR, Muir H. Types I and II collagens in intervertebral disc. Interchanging radialdistributions in annulus fibrosus. Biochem J. 1976;157:267-70.56.-5

57.- Hombre de 52 años que presenta súbitamente dolor intenso en primer ortejo de pie derecho durante la noche posterior, tras ingesta de alcohol. Con los datos clínicos anteriores usted pensaría que el paciente presenta:

a) b) c) d)

Hiperuricemia crónica sintomática. Síndrome de reiter. Monoartritis infecciosa. Artritis gotosa.

Las causas que generan hiperuricemia son múltiples, pero en general podemos dividirlo en tres grupos:1° Hiperuricemia idiopática (10-15%) que representa los errores innatos del metabolismo, padecimientos caracterizados por un incremento en la síntesis de purinas y sobreproducción de ácido úrico; 2° Por incremento del recambio metabólico y que genera; un exceso de ácido úrico sérico, observado en los síndromes mieloproliferativos, neoplasias, etcétera, y 3° grupo conformado por padecimientos o condiciones que producen baja excreción renal de ácido úrico. La acidosis metabólica, insuficiencia renal, enfermedades

metabólicas endocrinológicas y el uso de fármacos, son las causas más comunes de esta situación. Factores de Riesgo: Todas las causas de hiperuricemia son un factor de riesgo para la gota. 1) La obesidad o el aumento o pérdida repentinos de peso; 2) mayores de 40 años; 3) sexo masculino; 4) Miembros de la familia con gota;5) Diuréticos, tales como el hidroclorotiazido; 6) Algunas medicinas, como la aspirina; 7) Una diera rica en cristales de nitrógeno; 8) Consumo de alcohol; 9) Algunos tipos de cáncer o tratamientos contra el cáncer (por ejemplo, medicinas citotóxicas); 10) Medicinas (tales como los que combaten la apoplejía y otros); 11) Deshidratación;12) Hipercolesterolemia;13) Enfermedad renal; 12) Desordenes endocrinos, como el hipotiroidismo y el hiperparatiroidismo.

Como es una enfermedad inflamatoria tiene un comportamiento episódico, de distribución universal, predomina en el hombre con el 90% de los casos y 10% en mujeres, de preferencia posmenopáusicas. De inicio mas frecuente entra la 4° y 6° década. Con diferentes estadios conocidos como hiperuricemia sintomática, artritis gotosa aguda, periodo intercrítico y gota crónica tofácea. Entonces, de acuerdo a los estadios de la gota podemos mencionar: A Hiperuricemia asintomática B Artritis Gotosa: Es la mas común y se caracteriza por dolor agudo, intenso, que afecta a una ó más articulaciones de predominio monoarticularlas, de acuerdo a orden de frecuencia son; el dedo gordo del pie tobillo, tarso, rodilla y muñeca. C Periodo Intercrítico: intervalo entre un ataque y otro. El paciente queda libre de síntomas hasta la presentación del siguiente ataque de artritis, la duración del periodo asintomático es variable. Posteriormente los periodos asintomáticos se van acortando y el paciente evoluciona hacia la cronicidad. Gota Tofácea Crónica: Caracterizada por periodos recidivantes de artritis con depósitos de uratos conocidos como tofos, tumoraciones sobre la articulación que pueden estar excretando un material calcáreo.

Rev Paceña Med Fam 2006; 3(3): 2-5 Dra. Nilsa Selaya C. Dr. Cesar Rabaza M. Dr. Rafael Castillo R.

58.- Masculino de 53 años de edad presenta una historia de 2 días de hemoptisis. Reporta un inicio agudo de 8 episodios de tos con sangre fresca (aprox una cucharadita de sangre por cada episodio). No reporta otros síntomas, excepto por una tos productiva de 5-10ml de esputo cada mañana. Tiene una historia de EPOC, para lo cual toma broncodilatadores. Ha fumado 30 cigarrillos diariamente por los últimos 30 años. El examen físico es normal, y una RX limpia. De los siguientes cual es la causa más probable de la hemoptisis?

a) Bronquiectasias b) Carcinoma broncogénico

c) Tb pulmonar d) Bronquitis crónica

La EPOC puede presentar episodios de hemoptisis con placa radiográfica normal, aunque no es muy frecuente; las otras causas como TB pulmonar y Cáncer pulmonar así como las bronquiectasias presentan lesiones en la radiografía que hace sospechar en estos diagnósticos y en el presente caso no se mencionan lesiones en la Rx, la deficiencia de alfa 1 provoca la presencia de enfisema pulmonar la cual igualmente debería tener una placa con alteraciones. 1.- Fishman AP, , Fishman JA, Grippi MA, Kaisser LR, Señor RM. Pulmonary Diseases and disorder. 3a. Edición McGraw-Hill, EUA, 2006. 2.- Fraser, R ; Neil, C; Parè, P; Diseases of the Chest, Third Edition, Editorial Elsevier, 2005. 3.- Murray and Nadels; Textbook Respiratory Medicine, Vol 1-2, Elsevier editorial, 2005.

59.- La frecuencia es de 1/20,000, bajo peso al nacimiento, LPH (Labio paladar hendido) microftalmia, holoprosencefalia, aplasia cutis vertex, riñones poliquísticos, malrotación del colon, y en niñas puede existir útero bicorne, los datos anteriores son compatibles con la siguiente alteración cromosómica:

a) b) c) d)

Trisomía 13 Trisomía 21 Trisomía 18 Trisomía 11

La trisomía 13 representa la tercera aneuploidía autosómica viable más frecuente en la especie humana, sólo superada en frecuencia por la trisomía 21 y trisomía 181. Aunque su etiología es aún desconocida, estudios citogenéticos en individuos afectados han podido determinar que aproximadamente un 75% de los casos corresponden a no-disyunción meiótica, un 20% a translocaciones y un 5% a mosaicismo debido a nodisyunción postcigótica. La expresión fenotípica de la trisomía 13 es característica y consiste en anomalías faciales, esqueléticas y del sistema nervioso central, siendo también frecuentes las malformaciones estructurales del aparato cardiovascular, genitourinario y gastrointestinal (Tabla I).1 La mayoría de los defectos estructurales asociados a trisomía 13 pueden ser identificados prenatalmente por ultrasonografía.2-6 Esto permite ofrecer estudio citogenético prenatal en casos seleccionados, establecer el diagnóstico definitivo y manejar el embarazo afectado en forma racional, ya que esta anomalía se asocia a una mortalidad perinatal cercana al 100%.

60.- Se trata de masculino de 22 años, sin antecedentes patológicos de importancia, inicia un cuadro de febrícula, dolor articular, tos seca persistente y astenia de dos semanas de evolución. En el último mes, sus dos hermanos menores que él han presentado consecutivamente un cuadro similar, que se ha autolimitado de forma progresiva. Tras practicársele una radiografía de tórax, el médico le ha diagnosticado de neumonía atípica. El agente etiológico más probable en este caso es: a) b) c) d)

Legionella pneumophila. Coxiella burnetti (fiebre Q). Haemophilus influenzae Mycoplasma pneumoniae

La neumonía por micoplasma es un tipo de neumonía atípica y es causada por la bacteria M. pneumoniae. Este tipo de neumonía generalmente afecta a personas menores de 40 años.

Diversos estudios sugieren que esta enfermedad comprende entre el 15 y el 50% de todos los casos de neumonías en adultos e incluso más en los niños en edad escolar.

Las personas que se encuentran en mayor riesgo de adquirir neumonía por micoplasma incluyen aquellos que viven o trabajan en áreas de hacinamiento como escuelas y hogares de personas abandonadas, aunque muchas personas que la contraen no presentan ningún factor de riesgo que se pueda identificar. Cuadro clínico

Mycoplasma pneumoniae produce infecciones del aparato respiratorio, principalmente en

forma de neumonía que, por sus peculiares características de presentación clínico radiológica se denomina neumonía atípica primaria. Los síntomas se presentan de manera gradual en varios días, y consisten en fiebre, tos no productiva, cefalea y mialgias. A menudo, se acompaña de faringitis, rinitis, otitis y traqueobronquitis. La exploración física se caracteriza por la parquedad de síntomas, auscultándose ligeros subcrepitantes, aunque los pacientes pueden presentar crepitantes francos, roncus y sibilantes. En la radiografía de tórax se observan infiltrados retículonodulillares parahiliares o peribronquiales que pueden ser uni o bilaterales. Puede observarse la presencia de un pequeño derrame pleural en uno de cada cuatro o cinco pacientes. En los análisis complementarios suele encontrarse una discreta leucocitosis en un 30% de los pacientes. Los niños con alteraciones inmunológicas como la anemia de células falciformes, con anesplenia funcional o con síndrome de Down, pueden desarrollar una infección respiratoria grave y de evolución fulminante. La hipogammaglobulinemia es también un factor de riesgo para las infecciones del tracto respiratorio y de sus complicaciones a nivel articular. Se han descrito otro tipo de infecciones, aunque en menor frecuencia. Generalmente, acompañan a un cuadro respiratorio, pero pueden aparecer en ausencia absoluta de síntomas de esta localización. En la tabla 1 se relacionan las principales manifestaciones y complicaciones extrapulmonares.

Tabla 1. Infecciones extrapulmonares por Mycoplasma pneumoniae.

Sistema nervioso central

Meningoencefalitis, neuritis óptica, parálisis nervios craneales, parálisis ascendente (Síndrome Guillain-Barré), ataxia y psicosis Piel Erupción eritematosa papular o vesicular. Síndrome de Stevens-Johnson Articular Mialgias, artralgias y poliartropatias Artritis séptica (especialmente en caso de hipogammaglobulinemia) Cardiaca Pericarditis, miocarditis y derrame pericárdico Sistema hematopoyético Anemia hemolítica asociada con aglutininas frias Púrpura trombótica trombocitopénica Renal Glomerulonefritis, nefritis tubulointersticial, nefropatía IgA Gastrointestinal Vómitos, diarreas y hepatitis colestásica. Pancreatitis Otros Otitis externa, otitis media y miringitis Rabdomiolisis Conjuntivitis, uveítis anterior, retinitis y neuritis óptica Abscesos tubo-ováricos BIBLIOGRAFÍA:

WAITES KB, TALKINGTON DF. Mycoplasma pneumoniae and its role as a human patogen. Clin Microbiol Rev 2004; 17:697-728. AUSINA V, RODRIGO C. Infecciones causadas por micoplasmas. En: Farreras-Rozman (eds). Medicina Interna, 15ª ed. Madrid: Elsevier España SA, 2004; pp 2362-2365. TALKINGTON DF, SHOTT S, FALLON MT, SCHWARTZ SB, THACKER WL. Analysis of eight commercial enzyme immunoassay tests for detection of antibodies to Mycoplasma pneumoniae in human serum. Clin Diagn Lab Immunol 2004; 11:862-867. MICHELOW IC, OLSEN K, LOZANO J, DUFFY LB, MCCRACKEN GH, HARDY RD. Diagnostic utility and clinical significance of naso- and oropharyngeal samples used in a PCR assay to diagnose Mycoplasma pneumoniae infection in children with community-acquired pneumonia. J Clin Microbiol 2004; 42:3339-3341. TEMPLETON KE, SCHELTINGA SA, GRAFFELMAN AW, VAN SCHIE JM, CRIELAARD JW, SILLEKENS P, ET AL. Comparison and evaluation of real-time PCR, real-time nucleic acid sequencebased amplification, conventional PCR, and serology for diagnosis of Mycoplasma pneumoniae. J Clin Microbiol 2003; 41:4366-4371.

61.- Se trata de paciente femenino de 21 años que cursa con amenorrea secundaria, la cual presenta menstruación posterior a la administración de progestágenos, lo más probable es que ésta paciente presente:

a) b) c) d)

Estrógenos bajos Progesterona elevada Gonadotropinas altas Estrógenos normales

La presencia de estrógenos se puede establecer de dos maneras: con la prueba de desafío con acetato de medroxiprogesterona (AMP) y el índice de maduración vaginal. Si hay estrógenos, luego de la administración de 10 mg de AMP diarios por 10 días debería presentarse un sangrado por vagina. Y de esta forma corroborar los niveles de estrógenos serian normales. La ausencia de esta metrorragia sugiere una obstrucción o un hipogonadismo. La otra alternativa es realizar el índice de madurez vaginal que es un procedimiento sencillo que puede obtenerse en el consultorio. Usando una espátula como las de Papanicolaou el ginecólogo extrae una muestra de la vagina y la extiende en un portaobjeto. Este portaobjeto se procesa y se estudia en el microscopio. Un extendido maduro tendrá células epiteliales superficiales grandes y numerosas con un núcleo rodeado por una gran cantidad de citoplasma. Por el contrario, una muestra no estrogénica tendrá un mayor número de células parabasales y basales con núcleo grande rodeado por escaso citoplasma. Speroff L, Glass RH, Kase NG, eds. Clinical Gynecologic Endocrinology and Infertility, 5th ed. Baltimore: Williams & Wilkins; 1994:334-335.

62.- Se trata de femenino de 29 años de edad, la cual inicia con hiperemesis gravídica de difícil control, así como sangrado trasvaginal. Se realiza el diagnóstico de mola hidatiforme. ¿El tratamiento inicial indicado en esta patología es? a) b) c) d)

Legrado por aspiración. Histerectomía total abdominal. Metotrexate y seguimientos radiográficos. Legrado uterino instrumental

La enfermedad trofoblástica gestacional agrupa a diferentes entidades interrelacionadas: mola completa, generalmente diploide con origen cromosómico paterno, mola parcial generalmente triploide, tumor trofoblástico del lecho placentario y coriocarcinoma, con tendencias variables a la invasión local y a las metástasis, cuyo denominador común es la hipersecreción de hCG. El coriocarcinoma es diploide y proviene de ambos progenitores, excluyendo probablemente su origen directo en la mola completa. El tumor trofoblástico del lecho placentario está constituido por trofoblasto mononuclear intermedio no conteniendo vellosidades coriónicas e inmunohistoquimicamente caracterizado por expresar muchas de sus células hPL y unas pocas hCG. Tratamiento

Hay que tratar las complicaciones como la hiperémesis, anemia, hipertensión, y alteraciones electrolíticas, coagulopatías, alteraciones cardio-respiratorias y preeclampsia, procediendo a evacuar la mola lo antes posible, con lo que se producirán menos malignizaciones. La evacuación del contenido uterino se realiza mediante dilatación, y legrado por aspiración. Además se pauta profilaxis antibiótica y oxitócicos.3 La histerectomía, con la mola en su interior, está indicada en pacientes de edad superior a 40 años o en mujeres con más de tres hijos, ya que en ambos grupos se ha demostrado una mayor incidencia de malignización. . Tras la cirugía, se mide la concentración de gonadotropina coriónica humana para determinar si la extirpación ha sido completa. Si es así, el valor de esta hormona vuelve a la normalidad, en unas 8 semanas, y se mantiene en esos valores. Si una mujer a la que se le ha extirpado una mola queda embarazada, es difícil interpretar un valor alto de gonadotropina coriónica humana, porque podría estar causado tanto por el embarazo como por una parte de la mola que no se ha extirpado. En consecuencia, a las mujeres a las que se les ha extirpado una mola se les recomienda no quedar embarazadas durante un año. Las molas hidatiformes benignas no necesitan quimioterapia, pero las malignas sí. Los fármacos que se usan para este tratamiento son el metotrexato, la dactinomicina o una combinación de ambos. Referencias: Silverman L, Romero Zambrano F, Saldaño S. Enfermedad molar. Diagnóstico, tratamiento y seguimiento, 4- Puertas A, López Fernandez J et al. Enfermedad trofoblástica. Casuística del Hospital Virgen de las Nieves de Granada. Clín Invest Gin Obs 20: 98-103. 5- Enfermedad trofoblástica gestacional. Propuesta Normativa Perinatológica y Ginecológica de Alto Riesgo. Ministerio de Asuntos Sociales. Tucumán. 1996-1997; 19:171-183. 6- Jones. Enfermedad Trofoblástica Gestacional: qué hemos aprendido en la última década. Am J Gynecol Obstet 1990;162: 1286-1292. 7- Resúmen del simposio Enfermedad Troblástica Gestacional del Segundo Congreso Nacional de AGORA, 1990. 8- Gonzalez Merlo et al. Protocolos de diagnóstico y tratamiento en Obstetricia y Ginecología, Barcelona: editorial Salvat, 7:35-45.

63.- Lactante de un año de edad, es atendida en consulta por erupción inflamatoria aguda del área cubierta por el pañal de aproximados 4 días de evolución. Antecedentes: gastroenteritis hace 4 días. Exploración fisica: se aprecian pápulas y pústulas sobre una base eritematosa, intensa, brillante, asimismo lesiones satélites que se presentan en la periferia. El cuadro clínico de éste paciente está generado por:

a) b) c) d)

Infeccion bacteriana. Contacto prolongado con orina o heces. Alergia al material del pañal. Dermatitis atópica

DERMATITIS DE LA ZONA DEL PAÑAL. SINONIMIA. Dermatitis del pañal, dermatitis amoniacal,“rozadura”. DEFINICION. Erupción inflamatoria aguda del área cubierta por el pañal. EPIDEMIOLOGIA. Afecta con mayor frecuencia recién nacidos y lactantes, con un pico de incidencia a los 9 a 12 meses, aparentemente en relación con el cambio en la dieta. Se presenta además en niños mayores y adultos con incontinencia urinaria y/o fecal secundaria a alteraciones urológicas o neurológicas. La prevalencia es del 7 al 35% en población pediátrica, ocupando el 3ª a 4ª causa de consulta dermatológica en niños. No se ha descrito diferencia racial ni en cuanto al sexo. ETIOPATOGENIA. En 1973 Koblenzer clasificó la dermatitis de esta zona en tres grupos: Grupo 1. Enfermedades que se producen en forma independiente del uso de pañales (epidermolisis bulosa, histiocitosis de células de Langerhans, etc.). Grupo 2. Enfermedades agravadas por el uso de pañales (dermatitis atópica, dermatitis seborreica, psoriasis, etc.). Grupo 3. Enfermedades provocadas por el uso de pañales, en niños no predispuetos y como consecuencia directa de su uso, es el más frecuente y ocupará nuestra revisión. Lo mas característico es la dermatitis de contacto por irritante primario en la cual la humedad persistente provoca maceración de la piel; lo anterior aunado a la fricción generada por los movimientos del bebé altera la función de barrera y permite la acción de irritantes como son: heces (proteasas, lipasas y sales biliares), orina (urea y amoniaco), jabones, lociones, cremas y productos empleados para el lavado de los pañales; por lo tanto, los irritantes actúan en forma secundaria en piel ya dañada previamente. Por otro lado, la piel así dañada, es susceptible de infección por Candida albicans con mayor frecuencia (50%) y mas raramente, agentes bacterianos (S aureus, estreptococos y enterobacterias). La dermatitis por contacto alérgica no es común el área del pañal, sin embargo algunas sustancias como gomas, plásticos, neomicina y mercurio pueden actuar como sensibilizantes. CUADRO CLINICO. Existen diferentes patrones de afección clínica: Dermatitis eritematosa simple. Es el mas frecuente. Afecta áreas convexas (en W), respetando los pliegues. Las formas mas leves se caracterizan por eritema y descamación; el eritema puede desarrollar una apariencia vidriosa y aparecer pápulas (fig1). Variedad sifiloide pápulo-erosiva o poserosiva de Sevestre-Jaquet. Es una forma moderada de la dermatitis eritematosa simple. Se presenta sobre las superficies convexas en que se observan pápulas del color de la piel o eritematosas, generalmente menores de 5 mm de diámetro, que por acción de la fricción o maceración se erosionan en la cúpula. Pueden volverse violáceas y liquenificadas (fig.2). Ulcerosa. Se caracteriza por pérdida cutánea epidérmica o mas profunda, siendo la presentación mas grave de las anteriores; surge como consecuencia de la potencia y el tiempo de exposición a la agresión y sensibilidad individual de la piel. Dermatitis perianal. Limitada a esta área, se manifiesta como eritema, erosiones o pústulas; es mas frecuente en el periodo neonatal. Puede deberse a cambios químicos

(alteración del pH en procesos diarreicos o enteropatías) o infecciosos (dermatitis estreptocóccica perianal, parasitosis). COMPLICACIONES: Candidosis. Es la complicación mas frecuente. Compromete los pliegues inguinales e interglúteos y se caracteriza por pápulas y pústulas sobre una base eritematosa, intensa, brillante, con bordes netos, elevados, geográficos y escama blanquecina; asimismo lesiones satélites constituidas por pápulas y pústulas que se presentan en la periferia. Se ha demostrado colonización por C albicans en la dermatitis de la zona del pañal con mas de 72 horas de evolución (45-75%), aún en ausencia del cuadro clínico característico. Impétigo secundario. Agregándose pústulas y costras melicéricas. Granuloma glúteo infantil. Pápulas y nódulos rojizos o purpúricos de 0.5 a 4 cm, en región perianal y glútea; se ha asociado a irritación y fricción crónica, C. albicans y uso de esteroides fluorinados. Fig. 1. Dermatitis Fig. 2. Variedad sifiloide Fig. 3. Candidosis. eritematosa simple pápuloerosiva. DIAGNOSTICO. Clínico; puede solicitarse examen directo con KOH y cultivo para corroborar infección micótica. La biopsia se reserva a casos resistente al tratamiento, en que se sospecha otra enfermedad de base. DIAGNOSTICO DIFERENCIAL. 1.Dermatitis seborreica. Se caracteriza por placas eritematoescamosas con aspecto graso curso crónico y falta de respuesta al tratamiento convencional, que puede afectar el área del pañal, pero afectando también piel cabelluda, áreas centrofacial y pliegues proximales de los miembros (axilas e ingles). 2.Dermatitis atópica. Cuenta con ntecedentes personales o familiares de atopia; se relaciona con afección simultánea o previa en cara y pliegues de flexión (poplìteos y antecubitales), prurito, historia de cronicidad y recurrencia y respuesta inadecuada al tratamiento habitual. 3.Acrodermatitis enteropática. Alteración autonómica recesiva del transporte y absorción de zinc; se caracteriza por placas eritematoescamosas y erosionadas, de distribución periorificial, comprometiendo el área perianal pero también perioral. El cuadro se presenta con mayor frecuencia después del destete. Se asocia con manifestaciones extracutáneas como irritabilidad, diarrea, fotofobia y conjuntivitis. 4.Psoriasis. Puede encontrarse historia familiar de psoriasis y se caracteriza por un áreas eritematosas, que pueden comprometer toda el área del pañal, generalmente sin escama (por efecto queratolítico de la urea), sin mejorar con los tratamientos mencionados. Puede presentar además afección en piel cabelluda y uñas. 5.Histiocitosis de cèlulas de Langerhans. En la zona del pañal se presenta como dermatitis hemorrágica o erosiva grave, generalmente perianal, que no responde al tratamiento, sin embargo, generalmente se asocian a eritema y escamas adherentes en piel cabelluda, pápulas eritematosas o parduscas en tronco y lesiones purpúricas en palmas y plantas, así como manifestaciones extracutáneas. TRATAMIENTO. Medidas preventivas, como son el cambio frecuente de pañal, recomendándose pañales desechables con geles absorbentes (alginatos), de mayor tamaño, para evitar el roce y facilitar la absorción; realizar el aseo del área con agua, dejando el área sin pañal el mayor tiempo posible. Las pastas con talco y óxido de zinc son útiles como aislantes y para reducir la fricción y se aplican posterior a cada cambio de pañal. En las lesiones con mas de 72 horas de evolución o datos de infección por candida debe aplicarse algún agente antimicótico, 2 veces al día, por 3 semanas.

En caso de impétigo segundario se recomienda el uso de antibióticos tópicos o sistémicos. La utilización de corticoesteroides deberá ser racional, restringiéndose a casos resistentes a las terapias convencionales, utilizando esteroides de baja potencia y nunca más de 2 semanas.

BIBLIOGRAFIA: HOSPITAL FEDERICO GOMEZ. GUÍAS DIAGNÓSTICAS Y TERAPÉUTICAS DE LAS 10 PATOLOGÍAS MÁS FRECUENTES DR CARLOS ALFREDO MENA CEDILLOS, JEFE DEL SERVICIO DRA ADRIANA MARÍA VALENCIA HERRERA 1. Kazaks EL, Lane AT. Diaper dermatitis. Pediatr Clin North Am 2000 Aug; 47 (4): 90919. 2. Mena-Cedillos CA. Dermatitis de la zona del pañal. Bol Med Hosp Infant Mex 1997; 54: 386-91. 3. Ruiz-Maldonado R, Parish LCH, Beare LM. Tratado de dermatología pediátrica. McGraw Hill-Interamericana editores. México, 1993. 4. Arenas R. Dermatitis de la zona del pañal en: Dermatología atlas. Diagnóstico y tratamiento. McGraw Hill-Interamericana Editores. Segunda edición. México D.F. 1996; 41-2. 5. Pueyo de CS. Patología reaccional en: Dermatología infantil en la clínica pediátrica. Artes Gráficas Buschi. Argentina. 1999 Agosto; 183-212. 64.-Se trata de masculino en edad preescolar. Acude al servicio de consulta externa. Antecedentes: historia de geofagia. Hace 2 semanas presenta tos húmeda y "silbido del pecho". E.f.: temp. :37.2 ºc., mala higiene personal, tórax con sibilancias espiratorias bilaterales. Se reportan labs. BH con anemia y eosinofilia. En la radiografía de tórax de éste paciente espera encontrar datos de:

a) b) c) d)

Neumonía lobar. Infiltrados migratorios. Infiltrado intersticial unilateral. Infiltrado miliar.

En 1932 Loeffler describe un síndrome caracterizado por sintomatología respiratoria compatible con "pulmón sibilante",

Figura 1: Placa PA de tórax del paciente., infiltrado intersticial Difuso abigarrado bilateral compatible con neumonitis.

Los hallazgos radiológicos en los síndromes PIE generalmente son inespecíficos, los infiltrados en la radiografía (Rx) tórax pueden ser intersticiales, alveolares o mixtos, habitualmente son bilaterales y difusos. Radiológicamente la eosinofilia inducida por parásitos se manifiesta por infiltrados fugaces y migratorios. En la NEC se observa la clásica imagen de “fotografía negativa de edema pulmonar”, caracterizada por compromiso de las zonas apicales y periféricas, con indemnidad de las zonas centrales e inferiores. La Rx tórax en la ABPA muestra infiltrados fugaces, mediante la tomografía computada (TC) de tórax se pueden evidenciar bronquiectasias centrales. En la NEA puede encontrarse un derrame pleural de escasa cuantía, que contiene un alto porcentaje de eosinófilos. La imagenología además es útil en determinar la extensión del compromiso pulmonar, elegir los sitios óptimos para obtener la biopsia pulmonar y la respuesta al tratamiento. REFERENCIAS 1.- Del Giudice P, Desalvador F, Bernard E, Caumes E, Vandenbos F, et al. 2002. Löffler's syndrome and cutaneous larva migrans: a rare association. British J Dermatol 2002; 147: 385- 7 2. Botero D, Restrepo M. Parasitosis Humanas. 3° Ed., Medellín Colombia: Corporación para Investigaciones Biológicas 1998. 3.- Noemí I, Atias A. Eosinofilia y parasitosis. En Atias A. Parasitologia Médica. Mediterraneo, Santiago-Chile. 2000. 4.- Fujimura J, Murakami Y, Tsuda A, Chiba T, Migita M, Fukunaga Y. 2001.A neonate with Loeffler syndrome.Journal of Perinatology 2001; 21: 207-8 5.- Hunninghake GW, RichersonHB. Neumonitis por hipersensibilidad y neumonias eosinófilas. En Harrison TR. Principios de Medicina Interna. 15°Ed. McGraw-Hill -Interamericana de España, MadridEspaña. 2001 6.- Chusid MJ. Eosinophilia in childhood. Immunol and Allergy Clinics North America 1999; 19: 327-46 7.- Rothenberg ME. Eosinophilia N Engl J Med 1998; 338: 1592 8.- Noemi IH. Eosinofilia y parasitosis. Rev. Chil. Pediatr. 1999; 70: 1-7

9.- Hotez PJ, Broker S, Bethony JM, Bottazzi ME, Loukas A. Hookworm Infection. N Engl. J Med 2004; 351(8): 799-808 10.- Yilderan A, Ikinciogullari A. In the light of recent advances: eosinophil, eosinophilia and idiopathic hypereosinophilic syndrome. Turk Haematol 2005; 22(3): 107-16

65.- Se trata de paciente femenino de 42 años diagnosticado con síndrome de Cushing, se realizan exámenes de laboratorio que demuestran una ACTH muy baja casi suprimida, la primera posibilidad de diagnóstico es:

a) b) c) d)

Enfermedad hipotálamo-hipofisiaria Hiperplasia suprarrenal congénita Adenoma suprarreanal Adenoma hipofisiario

Aproximadamente 20 a 25% de los pacientes con síndrome de Cushing tienen una neoplasia suprarrenal. En la mayoría de estos casos existe una producción autónoma de cortisol por la neoplasia, lo que conduce a descenso en los niveles de ACTH a rangos indetectables o menores a 2 pmol/L o 10 pg/ml, por retroalimentación negativa de la secreción de ACTH. Williams GH, Dluhy RG. Enfermedades de la corteza suprarrenal. En Jameson JL (ed): Harrison. Endocrinología. 1a ed. Madrid. MacGraw-Hill España, 2006: 126-127.

66.- Se trata de masculino de 24 años que acude por esterilidad; es azoospérmico, tiene fenotipo eunucoide, ginecomastia, distribución feminoide del vello púbico, testículos pequeños y cromatina sexual positiva en 8%. ¿El diagnóstico más probable es ? a) b) c) d)

Síndrome de Klinefelter Síndrome de Noonan Síndrome de Turner Varón XX

SINDROME DE KLINEFELTER (SK) Es la cromosomopatía más frecuente y la causa más habitual de hipogonadismo hipergonadotrópico en el varón. Descrito en 1942 como un síndrome caracterizado por hipogonadismo, testes pequeños y duros, azoospermia y ginecomastia. Se comprobó posteriormente que el cuadro corresponde a una patología genética, cuya alteración cromosómica más habitual es la presencia de un cromosoma X adicional, reflejando un

cariotipo 47 XXY, que representa el 80% de los casos de SK, pero se han descrito otras variantes como mosaicismos: 47XXY/46XY, 47XXY/46XX, 47XXY/46XY/45X, etc. y formas con más de un cromosoma X ó Y (48XXYY ó 47XXY/46XX/poliX). Esto hace que estos sujetos presenten una cromatina de Barr con masa presente, siendo esto propio de las mujeres por la presencia de 2 cromosomas X. La aparición de más de 2 cromosomas X, ocasiona una patología que se diferencia del cuadro clásico de SK y se denomiona polisomía X del varón: 48XXY, 49XXXXY. La fórmula XXY se debe a una no disyunción del cromosoma X en la primera ó segunda división meiótica, siendo más frecuente la aparición de SK en relación a la edad materna más avanzada. Su frecuencia se estima en 1/1000 recién nacidos varones, pero posiblemente sea más elevada: en torno a 1/500 concepciones masculinas, y en varones con retraso mental moderado la frecuencia aumenta a 8/1000 aproximadamente. CLINICA: en general el diagnóstico es tardío, debido a la pobre expresividad clínica de este síndrome en la infancia. 1) Talla alta: es propia del síndrome de Klinefelter en la edad de la adolescencia, pero no lo es en la época prepuberal de forma habitual. Las proporciones corporales son eunucoides, con aumento desproporcionado de las extremidades inferiores y con inversión del cociente: segmento superior/inferior. Cuando este cociente es inferior a 1 en la edad puberal, sugiere fuertemente la existencia de un SK si se asocia a un cuadro de hipogonadismo aunque sea leve. La velocidad de crecimiento suele aumentar a partir de los 4 – 5 años, y se adopta ya en la infancia un cierto hábito eunucoide. La edad ósea es normal ó poco retrasada, coincidiendo con el marcado retraso de otras variantes benignas de retraso puberal. 2) Alteraciones genitales: Los testes pequeños y duros en la edad prepuberal, pueden presentarse en ocasiones, siendo por el contrario un hallazgo típico pasada la adolescencia. La presencia de criptorquídea, micropene, hipospadias etc y a veces un fenotipo peculiar, puede orientar al diagnóstico. En la edad puberal, la atrofia testicular es un signo constante, los testes son pequeños y su tamaño va involucionando con el tiempo, contrastando con un desarrollo normal del pene y del relativamente normal desarrollo puberal. La histología del testículo demuestra atrofia testicular con hialinización de los túbulos seminíferos. 3) Ginecomastia: Está presente en un elevado nº de casos (más del 60%), siendo el riesgo de neoplasia mamaria 20 veces superior al de la población normal. 4) Desarrollo intelectual: la inteligencia en general suele ser normal, ó “bordeline”, especialmente es el área verbal la más precoz y afectada, lo que condiciona dificultades de aprendizaje y en general peor rendimiento escolar. 5) Alteraciones hormonales: En la edad prepuberal, tanto el nivel de gonadotrofinas como de testosterona se encuentran en rango normal, tanto basal como tras estímulo. En la pubertad se va estableciendo ya respuesta propia del hipogonadismo hipergonadotrópico, especialmente el nivel de FSH que se eleva de forma llamativa.

6) Otras alteraciones asociadas: anomalías del tracto urinario, cúbito valgo etc. (se recogen en la tabla adjunta). Especial relevancia por la ayuda que puede suponer para el pediatra, es la relativamente aumentada incidencia de incontinentia pigmenti. Son frecuentes las asociaciones del SK con neoplasias especialmente las de células germinales mediastínicas, y con otros tumores en la edad adulta. Rossodivita A, Colabucci F. Short stature in a patient with Klinefelter syndrome and growth hormone deficiency.Am J of Med Genet 1994; 49:244-6.

67.- Masculino de 47 años con datos positivos de alcoholismo crónico, al cual se le diagnóstica pancreatitis aguda ¿Cuál es la primera medida terapéutica a adoptar? a) Iniciar antibióticos de amplio espectro. b) Administración de inhibidores de la bomba de protones. c) Suspensión de la vía oral. d) Aspiración nasogástrica.

El 80 % de los pacientes con pancreatitis aguda se tratan mediante medidas de sostén, suspensión de la vía oral, hidratación intravenosa, analgésicos y alimentación parenteral cuando la vía oral se restringe más allá de la semana.

La indicación de la suspensión de la vía oral se basa en el dolor y la intolerancia digestiva. No debe ser prolongada y se debe restablecer secuencialmente luego de 48 horas sin dolor. La alimentación parenteral no tiene ningún sentido si el restablecimiento de la vía oral se realiza dentro de los primeros 7 días. De no ser así se sabe que esta patología grave provoca una agresión severa que determina un estado hípercatabólico por lo tanto debe implementarse soporte nutricional para evitar la desnutrición y las complicaciones que ella trae consigo (alteración de la modulación de la respuesta inflamatoria, translocación bacteriana, inmuno supresión, etc.) El 60 % de esta enfermedad presenta hipermetabolismo (1,5 veces el metabolismo basal), el 40 % son normo o hipometabólicos. Según la Conferencia de Consenso de Nutrición de la Agresión (Francia 1998), los triglicéridos no son contraindicados a menos que el paciente presente una hipertrigliceridemia importante. La necesidad de aporte debe ser de 0,25 a 0,30 g/Kg. La suplementación de micronutrientes fundamentalmente antioxidantes (vitaminas A, C, E y selenio) y zinc están indicadas. La suplementación en base a glutamina, inmunomoduladores, nuevas emulsiones lipídicas en base a aceite de oliva necesitan aun ser confirmadas para la pancreatitis severa. La vía enteral se debe privilegiar ya que no solo es mas fisiológica sino que también presenta menor costo y complicaciones habiendo actualmente estudios que muestran un menor índice de morbimortalidad en estos pacientes ya que la integridad de la barrera intestinal limitaría la sobreinfección bacteriana pero fundamentalmente la fúngica ( Kalfarentzos 97, Windsor 98, Pupelis 2000)

Referencias Banks PA, Freeman ML; Practice Parameters Committee of the American College of Gastroenterology. Practice guidelines in acute pancreatitis. Am J Gastroenterol. 2006 Oct;101(10):2379-400. Frossard JL, Steer ML, Pastor CM. Acute pancreatitis. Lancet. 2008;371:143-152.

68.- Se trata de masculino de 57 años de edad acude a consulta refiriendo sensación de cuerpo extraño en epigastrio e hipocondrio derecho, antecedente de pancreatitis crónica, afebril, presenta una masa abdominal palpable lisa y dura con datos de laboratorio de amilasa sérica persistentemente elevada. El diagnóstico de mayor probabilidad es:

a) b) c) d)

Cistadenoma pancreático. Pseudoquiste pancreático. Carcinoma pancreático. Colección pancreática aguda.

El pseudoquiste pancreático es una colección de jugo pancreático localizada, por lo general, en el interior o alrededor del parénquima pancreático. El pseudoquiste pancreático está confinado por una capa no epitelializada de tejido necrótico, fibrótico y de granulación, que se desarrolla tras una lesión pancreática. Para su formación requiere un mínimo de cuatro semanas desde que ese daño se produce. El pseudoquiste pancreático es una complicación tanto de la pancreatitis aguda como de la crónica. Si bien la mayoría de los pseudoquistes pancreáticos se localizan en la cabeza y el cuerpo del páncreas, hasta un 20% de los mismos son extrapancreáticos (1). Se han descrito pseudoquistes pancreáticos en múltiples localizaciones, como cavidad pleural, mediastino y pelvis (2). Se presenta el caso de un pseudoquiste pancreático de localización hepática que apareció en el curso de una agudización de una pancreatitis crónica, y que se resolvió sin necesidad de drenaje. Diagnóstico: Manifestaciones Clínicas: 1. 2.

Pacientes en la cuarta o quinta década de vida, antecedentes etiológicos. Pacientes con pancreatitis aguda que no resuelve luego de 5 a 7 días de tratamiento o luego de mejoría recae. 3. Sensación de cuerpo extraño y pesadez en la mitad superior del abdomen. 4. Si pancreatitis crónica, dolor abdominal o síntomas por compresión de víscera. 5. Nauseas, vómitos y pérdida de peso por obstrucción duodenal. 6. Ictero, si compresión del colédoco. 7. Masa en abdomen superior, lisa y dura, muchas veces insensible. 8. Más raramente ascitis y derrame pleural. Complementarios Diagnósticos:

1. 2.

Ultrasonografía Abdominal: Muchas veces diagnóstico, preferido para vigilancia. TAC: Ideal para diagnóstico.

Bibliografía 1. Hamm VB, Franzen N. Atypically located pancreatic pseudocyst in liver, spleen, stomach wall and mediastinum: their CT diagnosis. Rofo 1993; 159 (6): 522-7 2. Vitas GJ, Sarr MG. Selected management of pancreatic pseudocyst: Operative versus expectant management. Surgery 1992; 111 (2): 123-30. 3. Mofredj A, Cadranel JF, Dautreaux M, Kazerouni F, Hadj-Nacer K, Deplaix P, et al. Pancreatic pseudocyst located in the liver: a case report and literature review. J Clin Gastroenterol 2000; 30 (1): 81-3 4. Balzan S, Kianmanesh R, Farges O, Sauvanet A, O'toole D, Levy P, et al. Right intrahepatic pseudocyst following acute pancreatitis: an unusual location after acute pancreatitis. J Hepatobiliary Pancreat Surg 2005; 12 (2): 135-7.

69.- Debemos sospechar altamente de un retinoblastoma en un niño que presenta los siguientes síntomas:

a) Dolor, fotofobia y lagrimeo. d) Estrabismo y leucoria. b) Lagrimeo, fotofobia y aumento del diámetro corneal. c) Fotofobia y quemosis conjuntival.

RETINOBLASTOMA TUMOR OCULAR MÁS FRECUENTE EN INFANCIA. 1/20.000 RN 80% en < 3 años Uni o bilaterales. Herencia 1. AD alta penetrancia (90-95%) 2. Esporádicos

Diagnóstico •leucocoria •estrabismo •mala AV •ojo rojo y doloroso •celulitis orbitaria •examen de rutina

Annals d.Oftalmologia 2001;9(2):74-92 N. Martín, MD. Coll, J. García, J. Sánchez de Toledo, E. Triviño, M. Guitart, JJ. Gil 1Unidad Oftalmología Pediátrica. Hospital Maternoinfantil Vall d.Hebron 2Departamento de biología-celular, fisiología e inmunología de la Universidad Autónoma de Barcelona 3Servicio oftalmología Hospital General Vall d.Hebron

70.- Se trata de masculiono de 24 años de edad, que acude al servicio de cardiología, sin antecedentes patológicos y sin hábitos tóxicos inicia con cuadro de 8 días de evolución acompañado de fiebre y dolor centrotorácico intenso que aumenta con la inspiración y los movimientos respiratorios. En el ecocardiograma se objetiva un derrame pericárdico importante, sin signos de compromiso hemodinámico. El diagnóstico más probable es:

a)

Pericarditis tuberculosa.

b)

Taponamiento cardíaco.

c)

Pericarditis aguda idiopática.

d)

Pericarditis de origen autoinmune.

La pericarditis aguda (PA) es un síndrome clínico debido a una inflamación del pericardio que se caracteriza por dolor torácico, roce pericárdico y alteraciones electrocardiográficas evolutivas. Su incidencia y prevalencia son difíciles de determinar. Estudios realizados en autopsias muestran una prevalencia de alrededor de 1% en la población general, lo que sugiere que la presentación en ocasiones se hace de manera subclínica. Representa un 5% de los dolores torácicos no isquémicos que acuden a urgencias 1. La causa más frecuente de PA es la idiopática y/o viral, que corresponden a casi el 80% de los casos, de hecho, los términos idiopático y viral suelen utilizarse de forma indistinta 2-5. Otras posibles causas son: Infecciosa (7%): Bacteriana: neumococo, estreptococo, estafilococo, neisseria, legionella. Tuberculosa. Vírica: coxsackie, influenzae, VIH, hepatitis, adenovirus, echovirus. Fúngica: histoplasmosis, coccidiomicosis, blastomicosis, candidiasis. Otras: sífilis, protozoos, parásitos (entamoeba histolytica, toxoplasma). Neoplasias (7%): primarias: mesotelioma, sarcoma, fibroma, lipoma secundarias (metástasis): pulmón, mama, linfoma, leucemia, carcinoides… La triada diagnóstica clásica es: dolor torácico, roce pericárdico y alteraciones electrocardiográficas que suele ser precedida de fiebre, mal estado general y mialgias (aunque en pacientes ancianos no suele presentarse la fiebre). Los síntomas principales son: Dolor torácico: dolor retroesternal o localizado en hemitórax izquierdo en la zona precordial, de características pleuríticas (aunque en ocasiones puede simular un cuadro isquémico), que se puede irradiar al cuello o al trapecio izquierdo. Puede acompañarse de sensación de falta de aire. Ocasionalmente se localiza en región epigástrica, simulando un abdomen agudo. Suele aumentar con la inspiración profunda, la tos, la deglución y la posición supina y mejorar con la incorporación a la posición de sentado. Roce pericárdico: es el hallazgo patognomónico de la exploración física en la PA. Corresponde al movimiento del corazón dentro del pericardio y se asemeja al ruido que produce la fricción de cuero, por roce de las hojas pericárdicas inflamadas. Se da en aproximadamente el 60-85% de los casos 7,8. Puede ser transitorio, monofásico, bifásico o trifásico, según la relación que tenga con los movimientos cardiacos durante la sístole auricular, sístole ventricular y el llenado ventricular rápido. Se ausculta mejor a nivel de mesocardio y en parte baja de borde esternal izquierdo, sobretodo al final de la espiración con el paciente inclinado hacia delante. Es independiente de la existencia de derrame.

Cuando sólo tiene un componente se puede confundir con un soplo sistólico mitral o tricuspídeo. Es característico del roce pericárdico su evanescencia (por lo que las auscultaciones deben ser repetidas en varias ocasiones) y los cambios en sus características según la posición en que se realice la exploración.

Bibliografía LeWinter MM, Kabbani S. Pericardial diseases. En: Braunwald´s heart disease. Douglas P. Zipes editor. 7th. Ed. Philadelphia: Elsevier; 2005. p.1757-1780 Friman G, Fohlman J. The epidemiology of viral heart disease. Scand J Infect Dis Suppl 1993; 88: 7–10. [Medline] Braunwald E. Enfermedades del pericardio. En: Harrison Principios de Medicina Interna. Dennos L. Koper editores. 16ª ed. México: Mc-Graw-Hill; 2005. p. 1554-1571 Maisch B, Seferovic PM, Ristic AD, Erbel R, Rienmüller R, Adler Y, et al, Grupo de Trabajo para el Diagnóstico y Tratamiento de las Enfermedades del Pericardio de la Sociedad Europea de Cardiología. Guía de Práctica Clínica para el diagnóstico y tratamiento de las enfermedades del pericardio. Versión resumida. Rev Esp Cardiol. 2004; 57:1090-114. [Medline] [Texto completo] Zayas R, Anguita M, Torres F, Gimenez D, Bergillos F, Ruiz M, Ciudad M, Gallardo A, Valles F. Incidence of specific etiology and role of methods for specific etiologic diagnosis of primary acute pericarditis. Am J Cardiol. 1995 Feb 15; 75(5):378-82.

71.- Mujer de 23 años, que presenta lesiones eritematoescamosas, edema y alguna vesícula en la cara, escote, dorso de las manos y antebrazos. Las lesiones tienen 12 horas de evolución y han aparecido tras una escursión al campo. Entre los antecedentes personales destaca acné vulgar en tratamiento con retinoides tópicos y doxiciclicina oral. El diagnóstico más probable es:

a) Erupción lumínica poliforma. d) Reacción fototóxica. b) Eritrodermia por fármacos. c) Urticaria solar.

DEFINICIÓN Enfermedades cutáneas que se producen por el aumento de capacidad de reacción de la piel a las radiaciones lumínicas tras la administración de una sustancia fotosensibilizante. Se conocen como reacciones de fotosensibilidad y pueden desencadenarse tanto por contacto

como por la administración sistémica del agente fotosensibilizante. Si existe implicación inmunológica se denomina dermatitis fotoalérgica y si no dermatitis fototóxica.

Dermatitis fototóxica No existe un mecanismo inmunológico, puede afectar a muchas personas siempre que exista dosis elevada de irradiación y cantidad suficiente de sustancia química. Las lesiones aparecen tras la primera exposición, son monomorfas, y se caracterizan por eritema intenso, edema y vesiculación en áreas de piel fotoexpuestas, marcando claramente los bordes de las zonas descubiertas, y onicolisis ungueal . Formas particulares de fototoxia: fitofotodermatitis (dermatitis de los prados, apio) (Fig. 3), dermatitis de Berloque, fotosensibilidad en tatuajes (sulfuro de cadmio), fármacos (tetraciclinas, AINEs, amiodarona (color azulado), clorpromacina (color gris…).

Figura 3. Fitofotodermatitis 1. Litt Jz. Drug eruption reference manual 2001. New York: Parthenon, 2001. 2. Sullivan JR, Shear NH. Drug eruptions and other adverse drug effects in aged skin. Clinics in geriatric medicine 2002;18(1). 3. Lim HW, Gigli. Complement-derived peptides in phototoxic reaction. En: Daynes RA, Spikes JD, editors. Experimental and clinical photoimmunology. Boca Raton: CRC Press, 1983:81-93. 4. Torinuki W, Tagami H. Role of complement in chlorpromazine-induced phototoxicity. J Invest Dermatol 1986;86:142-4. 5. Hearst JE, Issacs ST, Kanne D, Rapoport H, Straub K. The reaction of the psoralens with deoxyribonucleic acid. Q Rev Biophys 1984;45:891-5. 6. Athar M, Elmets CA, Bickers DR, Mukhtar H,. A novel mechanism for the generation of superoxide anions in hematoporphyrin derivative-mediated cutaneous photosensitization. Activation of the xantine oxidase pathway. J Clin Invest 1989;83:1137-43. 7. Matsuo I, Inukai N, Fujita H, Ohkido M. Possible involvement of oxidationmof lipids in inducing griseofluvin photosensitivity. Photodermatol Photoimmunol Photomed 1990;7:213-

7. 8. Harber LC, Bickers DR. Photosensitivity diseadses. Principles of diagnosis and treatment. Ontario: BC Decker Inc, 1989:160-202. 9. Kockevar IE. Phototoxicity of nonsteroidal inflammatory drugs. Coincidence or specific mechanism?. Arch Dermatol 1989;125:824-6.

72.- Which one of the following conditions results in prologation of the partial thromboplastin time (PTT), but not the prothrombin time (PT)? a) b) c) d)

Varicela hemorrhage as a result of cirrosis Therapy with broad-spectrum antibiotics Therapy with coumarin for phlebitis Menorrhagia resulting from von Willebrand’s disease

Enf. Von Willebrand

Manifestaciones

     

Epistaxis

60 %

Hemorragia transvaginal

50 %

Equimosis

40 %

Gingivorragias

35 %

Hematomas

5%

Hemartrosis

3%

EvW Pruebas de escrutinio T. Hemorragia

Prolongado

C. Plaquetaria

Normales, excepto 2B

TTPa

Normal o prolongado

TP

Normal

Gpo AB0

25 % bajo en “0”



1.-Lee GR, Foerster J, Lukens J, Paraskevas F, Greer JP and Rodgers GM. Wintrobe’s clinical haematology; 10th Edition, Lippincott Williams & Wilkins, United States of America, 1999.

• •

2.-Williams WJ. Manual Williams de hematología 5a Edición McGraw-Hill Interamericana, México, 1997.



3.-Beutler E, Lichtman MA, Coller BS, Kipps T. Hematology. 5th International Edition. United Stated of America, 1995.



4.-Ruiz Argüelles GJ. Fundamentos de hematología ; 2ª. Edición, Editorial Médica Panamericana, México, 1998.

73.- .Se trata de masculino de 27 años acude a su consultorio con reporte de exámenes de laboratorio con los siguientes resultados. Hemolisinas bifásicas + (o anticuerpos de Donath-Landsteiner) productoras de hemoglobinuria paroxística a frigore. La Entidad responsable de los resultados de este paciente es: a) Leucemia linfática crónica. b) Sífilis. c) Mieloma múltiple. d) Lupus eritematoso diseminado. Razones por las que se realiza el examen Este examen se realiza algunas veces cuando el médico sospecha de un diagnóstico de criohemoglobinuria paroxística. Valores normales La ausencia de anticuerpos es lo normal.

Significado de los resultados anormales Los resultados anormales indican la presencia de criohemoglobinuria paroxística (PCH), un trastorno que ocurre cuando la exposición a bajas temperaturas hace que el sistema inmunitario produzca anticuerpos que destruyen los glóbulos rojos. Estos anticuerpos se denominan anticuerpos de Donath-Landsteiner. A medida que las células son destruidas, la parte de los glóbulos rojos (hemoglobina) que transporta el oxígeno es eliminada en la orina. La criohemoglobinuria paroxística es un síndrome adquirido. Algunas veces, los anticuerpos se presentan con una infección viral (por ejemplo, sarampión y paperas) o con sífilis; sin

embargo, en algunos casos, el trastorno no está relacionado con una enfermedad y la causa se desconoce. K. Holmes, P. Mardh, P. Sparling et al (eds). Sexually Transmitted Diseases, 3rd Edition. New York: McGraw-Hill, 1999, chapters 33-37.

74.- Masculino de 14 años de edad con antecedentes de convivencia con aves, gatos e ingestión muy frecuente de berros. Inicia su padecimiento actual hace dos semanas con fiebre de 39ºC sin predominio de horario, náusea y dolor en hipocondrio derecho. A la exploración física con palidez generalizada y hepatomegalia 3-3-5. Resto de la exploración sin datos patológicos. Laboratorio con BH, con Hb de 11.5, Hto de 40, leucocitos; 16,500, eosinófilos; 38%, linfocitos; 30%, formas inmaduras; 4, plaquetas; 270,000, Fosfatasa alcalina; 280 UI/L.

El diagnóstico más probable en éste paciente es:

a) Larva migrans visceral b) Toxoplasmosis generalizada c) Fasciolosis e) Estrongiloidosis

Definición La Fasciolosis es una zoonosis parasitaria causada por la Fasciola hepática que ocasiona patología y sintomatología hepato-biliar.

Clínica Se considera las siguientes formas de presentación clínica: a) Sintomática: Aguda o invasiva, crónica o de localización y extrahepática. 1. Aguda o invasiva: Hay tres elementos esenciales a identificar: hepatomegalia dolorosa, fiebre y eosinofilia con cifras que superan frecuentemente el 30-40%. 2. Crónica o de localización: La sintomatología y signología corresponden a padecimiento crónico hepato biliar incluyendo cólicos biliares y litiasis biliar. 3. Extrahepática: Incluye nódulos subcutáneos en el hipocondrio derecho, seno derecho, escápula derecha con poco dolor local y signos inflamatorios. Eosinofilia alta. b) Asintomática: En algunas personas los síntomas o signos suelen pasar desapercibidos.

Diagnóstico de Laboratorio

En la forma aguda, la búsqueda de huevos en las deposiciones es inútil, ya que las formas juveniles están en el tejido hepático, por lo tanto las pruebas inmunobiológicas son importantes. Son útiles la inmunoelectroforesis o inmunodifusión buscando el arco 2 de Caprón (4). El inmunoblot o westernblot tiene buena sensibilidad y especificidad (5,6). Se han identificado fracciones antigénicas en las cistenilproteasas de F. hepatica (7). En las formas crónicas, la búsqueda de huevos en heces es lo indicado. Son útiles la sedimentación rápida de Lumbreras (8). Recientemente se han preparado anticuerpos monoclonales contra el parásito y ello ha permitido elaborar la técnica de ELISA para detectar los coproantígenos (E/S) del parásito en heces. La ecografía de vías biliares (v.b.) puede detectar al parásito moviéndose en las v.b. o vesícula. En formas extrahepáticas, la eosinofilia alta es orientadora; F. hepatica en las biopsias confirma el diagnóstico.

75.- Masculino de 46 años de edad que inicia con dolor intenso en fosa renal izquierda ante la sospecha de litiasis renoureteral se realizan estudios de laboratorio y gabinetem, los Rx demuestran cálculos radiolúcidos, los cuales están relacionados a: a) b) c) d)

Calcio Estruvita Cistina Ácido úrico

-Bruce E. Jarrell, R. Anthony Carabasi, Nacional Medical Series for Independent Study. Wiliams & Wilkins, 3rd Edition: 451-475. Los cálculos que sepueden presentar en la vía urinaria son formados por calcio, ácido úrico y cistina, los únicos radiolucidos son los de ácido úrico que representan un reto diagnóstico.

76.- Masculino de 56 años con diagnóstico probable de sinusitis aguda, la proyección radiológica que mejor valora los senos maxilares y las estructuras intranasales, en ésta patología es:

a) b) c) d)

Lateral. Submentoniana. Anteroposterior. Waters.

La proyección Waters es la proyección que mejor permite valorar la neumatización, opacificación o engrosamiento de mucosa de los senos maxilares así como las estructuras intranasales.

Proyección de Waters u occipito-mentoniana para senos maxilares (Radiografía normal). González-Saldaña N, Infectología Clínica Pediátrica, 7ª edición, páginas 63-98.

77.- Masculino de 18 años que desde la infancia padece anemia, esplenomegalia e ictericia. El frotis de sangre periférica muestra eritrocitos pequeños con palidez central y una fragilidad osmótica muy incrementada. El diagnóstico más probable es:

a) Anemia perniciosa b) Talasemia mayor c) Esferocitosis hereditaria d) Leucemia linfoblástica

La esferocitosis hereditaria (EH) es una enfermedad caracterizada por anemia hemolítica de severidad variable, con presencia de esferocitos en sangre periférica y una respuesta clínica favorable a la esplenectomía. Con el desarrollo de nuevas técnicas se encontraron las primeras alteraciones bioquímicas de las proteínas de la membrana eritrocitaria, y posteriormente, se han podido precisar las alteraciones moleculares mediante las técnicas del ADN recombinante. La EH es una enfermedad muy heterogénea que se produce por un defecto intrínseco del glóbulo rojo, y existen otras alteraciones secundarias a esta afección. La prueba más utilizada para el diagnóstico de la EH es la fragilidad osmótica del glóbulo rojo. Se ha demostrado que esta enfermedad es producida por defectos de las proteínas que intervienen en las interacciones verticales entre el esqueleto de la membrana y la bicapa lipídica. El tratamiento de elección en la EH es la esplenectomía, ya que es el más efectivo en el control de la anemia, aunque la sobrevida de los glóbulos rojos permanece acortada y los esferocitos no desaparecen. Este proceder se indica en pacientes con anemia hemolítica severa o en individuos moderadamente asintomáticos pero que presentan litiasis vesicular.

Formas Clínicas La EH es una enfermedad muy heterogénea desde el punto de vista clínico. Se puede observar desde el portador asintomático hasta pacientes que presentan una anemia hemolítica crónica con grandes requerimientos transfusionales.24,25 Dependiendo de la severidad del cuadro clínico, de las cifras de hemoglobina, los niveles de bilirrubina y el conteo de reticulocitos, esta enfermedad se clasifica en 4 formas: portador asintomático, EH ligera, EH típica y EH severa.26,27

Portador asintomático. En algunas familias se ha señalado un patrón de herencia

autosómico recesivo. En estos casos, los padres de un paciente afectado no presentan ninguna alteración. En ocasiones la afectación es muy leve, como ligero incremento de las cifras de reticulocitos, escasos esferocitos en periferia o fragilidad osmótica incubada alterada y puede no ser detectada por los exámenes de rutina. Debe tenerse en cuenta también que pueden ocurrir nuevas mutaciones dentro de una familia aparentando un patrón de herencia autosómico recesivo, por lo que siempre es importante un estudio minucioso de todos los miembros de la familia.3,4.

EH ligera. Comprende entre el 20 y 30 % de todos los pacientes con EH autosómica

dominante, los que pueden presentar una hemólisis ligera compensada.3,27 Los individuos son frecuentemente asintomáticos y algunos casos son difíciles de diagnosticar, ya que la anemia y la esplenomegalia son muy ligeras y en ocasiones pueden estar ausentes.28 Muchos de estos pacientes se diagnostican durante estudios familiares o cuando en la etapa adulta aparece el íctero y la esplenomegalia. Episodios hemolíticos pueden presentarse en el curso de algunos procesos infecciosos como mononucleosis, parvovirus o citomegalovirus, así como durante el embarazo, por esfuerzos físicos intensos o por sangramientos.27-30.

EH típica. Entre el 50 y 60 % de los pacientes con EH autosómica dominante tienen esta forma clínica. Presentan una hemólisis compensada incompleta y una anemia de ligera a moderada. El íctero es común en niños, aunque se puede ver también en los adultos y está asociado con infecciones virales ligeras, debido a la estimulación reticuloendotelial y a un aumento de la hemólisis. Los requerimientos transfusionales son esporádicos. La esplenomegalia está presente en el 50 % de los niños y en el 75 % de los adultos.2,3,31,32. EH severa. Estos pacientes (5-10 %) evolucionan con una hemólisis severa y presentan

frecuentes requerimientos transfusionales. La mayoría de estos casos tienen una forma autosómica recesiva de la enfermedad. Pueden presentar crisis aplásticas, retardo del crecimiento y de la maduración sexual. La esplenectomía es el tratamiento de elección en esta forma clínica.33,34. Generalmente la enfermedad debuta al nacimiento con ictericia y hemólisis y se requiere, en muchas ocasiones, de exanguinotransfusión.30,32.

REFERENCIAS BIBLIOGRÁFICAS

1. 2.

3. 4. 5.

6.

7. 8.

9.

Delaunay J. Genetic disorders in the red cell membrane. Crit Rev Oncol Hematol;19:79-110. Iolascon A, Miraglia del Giudice E, Perrotta S, Alloisio N, Morle L, Delaunay J. Hereditary spherocytosis: from clinical to molecular defects. Haematologica 1; 83:240-57. Scriver SR, Beaudet AL, Sly WS, Valle D. The metabolic and molecular bases of inherited disease. Philadelphia: McGraw-Hill Company, CD-ROM, . Yu J, Steck TL. Isolation and characterization of band 3, the predominant polypeptide of the human erythrocyte membrane. J Biol Chem ;250:9170-6. Kay MMB, Folwers N, Goodman J. Alterations in membrane protein band 3 associated with accelerated erythrocyte ageing. Proc Nall Acad Sci USA 1989;86:5834-8. Marsh WL. Molecular defects associated with McLeod blood group phenotype. En. Salmon E, ed. Blood groups and other red cells surface markers in health and disease:17-82. Tse WT, Lux SE. Red blood cell membrane disorders. Br J Haematol 1999;104:2-13. Speicher DW, DeSilva TM, Speicher KD, Ursitti JA, Hembach P, Weglarz L. Location of human red cell spectrin tetramer binding site and detection of a related “closed” hairpin loop dimer using proteolytic footprinting. J Biol Chem 1993;268:4227-31. Jordan C, Puschel B, Koob GR. Identification of a binding motif for ankirin on the a subunit of Na+ K+ATPase. J Biol Chem 1995;270:29971-4

78.- Masculino de 62 años con antecedentes de insuficiencia cardiaca de 2 años de evolución, actualmente se presenta a consulta por presentar edema en MPS. Una de las siguientes puede ser la causa de un edema no inflamatorio:

a) Un aumento de la permeabilidad vascular. b) Una eliminación excesiva de sal y agua por el riñón. c) La disminución de la presión hidrostática intravascular. d) Un aumento de la presión hidrostática intravascular.

Edema Acumulo de un exceso de líquido en el espacio tisular intercelular (intersticial) o en las cavidades del organismo. El edema localizado: se puede producir en el caso de una obstrucción del flujo seroso (produciéndose la inflamación o hinchazón de una pierna por ejemplo). También puede ser causado por un proceso infeccioso (como los abscesos causados por Staphylococcus aureus). Edema generalizado: es de carácter sistémico como en el caso de insuficiencia cardiaca o de síndrome nefrótico (que se caracteriza por una intensa proteinuria secundaria a la permeabilidad glomerular anómala). Cuando el edema es intenso y generalizado de tal forma que provoca hinchazón difusa de todos los tejidos y órganos de la economía, especialmente a nivel del tejido subcutáneo, se denomina Anasarca. Exudado: líquido inflamatorio extravascular rico en proteínas, detritus celulares y leucocitos. Se trata de un acumulo de líquido que se encuentran rico en restos celulares, leucocitos, restos proteínicos y por lo general presenta una r>1.020. Se debe a un incremento en la permeabilidad endotelial con la salida de proteínas plasmáticas (principalmente albúmina). Se presenta por infecciones piógenes, TB, etc. (de origen inflamatorio). Trasudado: en el líquido de edema de origen no inflamatorio y está relacionado con los casos de insuficiencia cardiaca y nefropatías por lo general no hay (es pobre) proteínas, ni leucocitos, con una rÍ1.012. Únicamente es el acumulo de líquidos. El trastorno hemodinámico (trasudado) se presenta cuando se altera la Ley de Starling que es el equilibrio normal de los líquidos (intravascular y extravascular). Se mantiene por la acción de dos grupos de fuerzas opuestas, los que hacen que el líquido tienda a salir de la circulación son la presión osmótica del líquido intersticial y la presión hidrostática intravascular; los que hacen que el líquido pase a la circulación son la presión osmótica de las proteínas plasmáticas (principalmente albúmina) (presión coloidosmótica) y la presión hidrostática tisular. Por lo tanto el equilibrio entre estas fuerzas es tal que en los capilares musculares periféricos existe un movimiento neto de líquidos hacia fuera, pero éste líquido es drenado a los linfáticos, por lo que no se produce edema. Los factores que aumentan la presión hidrostática intravascular o disminuye la presión coloidosmótica intravascular dan lugar al aumento de la salida del líquido desde los capilares, con el consiguiente trastorno hemodinámico el edema.

Nota: en la lesión (por falta de la Angiotensina, que regula las sales) por pérdida de proteínas (principalmente albumina) (o sea la proteinuria) se produce el Edema generalizado (Anasarca).

Fisiopatología del edema •

Autores: José María Sillero Fernández de Cañete



Localización: Seminario médico, ISSN 0488-2571, Vol. 49, Nº. 1, 1997 , pags. 7486

Organización Panamericana de la Salud Programa de Publicaciones (DBI/E) 525 Twenty-third Street, NW

79.- Which of the following physical sings and syntoms is indicative of left ventricular failure? a) b) c) d)

Neck vein distensión Ascites Anorexia Orthopnea

INSUFICIENCIA CARDIACA IZQUIERDA Los síntomas más característicos son la disnea paroxística nocturna, la ortopnea y la tos. Al examen físico se constata la existencia de taquicardia, R3 y/o R4, pulso alternante, estertores inspiratorios de predominio en las bases. Algunas veces se auscultan sibilancias. Para confirmar su existencia se deben solicitar los siguientes estudios: a.

Radiografía de tórax. Para constatar la existencia de cardiomegalia, vasos sanguíneos prominentes, líneas B de Kerley, (patrón en "alas de mariposa" por edema pulmonar gravitacional) y derrame pleural.

b. Gases arteriales

c.

Cuadro hemático

d. Creatinina

e.

Electrocardiograma

f.

Ecocardiografía

LECTURAS RECOMENDADAS Bigger JT. Why patients with congestive heart failure die.

Circulation 75 (suppl, IV):28, 1997 Braunwald E. Heart Disease. En: Textbook of Cardiovascular Medicine. WB Saunders Co. Philadelphia, 1990 Matiz H. Insuficiencia cardiaca congestiva En: Diagnóstico y Tratamiento Integral en Medicina. Editado H Matiz. Colección Educación Médica Vol. 5. Fundación Escuela Colombiana de Medicina Santafé de Bogotá, 1991

80.- El dolor abdominal intermitente, la anemia, la proteinuria, la necrosis tubular, la esterilidad, los trastornos de la conducta, la polineuropatía periférica se encuentran en la intoxicación por: a) b) c) d)

Plomo Organoclorados. Sulfuro de hidrogeno. Etilenglicol

La intoxicación por plomo es la más común de las exposiciones a metales, el cual tiene muchos usos, las fuentes más frecuentes vienen de las minas y del reciclado de materiales conteniendo plomo. Este metal es absorbido por pulmones y del tracto gastrointestinal. El mecanismo de acción es por unión a los grupos sulfhidrilo y tóxico para las enzimas dependientes de zinc. Diagnóstico: La toxicidad aguda se presenta luego de una exposición respiratoria a altas concentraciones, con encefalopatía, insuficiencia renal y síntomas gastrointestinales. La toxicidad crónica es la más frecuente y se manifiesta con compromiso multisistémico: hematopoyético, del sistema nervioso, gastrointestinal, riñón y sistema reproductor. (Astenia, dolor abdominal, irritabilidad, náusea, vómitos, pérdida de peso, cefalea, anemia, neuropatía periférica, ribete de Burton, IRC, proteinuria, Nefritis intersticial, etc). En los exámenes auxiliares podemos encontrar anemia, punteado basófilo, aumento del ácido úrico, etc.

REFERENCIA BIBLIOGRÁFICAS 1. Keogh JP y Boyer LV. «Lead» en Sullivan y Krieger editores: Clinical Environmental Health and Toxic Exposures. Lippincott Williams & Wilkins, 2da edición, 2001

2. Decreto Supremo N° 019-98-MTC. Dispone eliminar del mercado la oferta de gasolina 95 RON con plomo y reducir el límite máximo de contenido de plomo en la gasolina 84 RON. (14/07/98) 3. Shannon Michael. «Lead» en Haddad, Shanon y Winchester editores: Clinical Management of Poisoning and Drug Overdose. WB Saunders, 3ra edición, 1998. 4. Krantz A, Dorevitch S. Metal exposure and common chronic diseases: A guide for the clinician. Dis Mon 2004; 50:215- 262. 5. Nogué S. Burton’ s Line. N Engl J Med 2006; 354:e21. 6. Rempel D. The lead-exposed worker. JAMA 1989; 262:532-4.

81.- Mujer de 26 años G-3, P-1, A-1 con 39 SDG por FUR. Reporta contracciones uterinas que han sido regulares las últimas tres horas. Al examen encuentras que las contracciones son cada tres minutos y duran 50 segundos y son firmes a la palpación. Tuvo ruptura de membranas hace una hora y lo demuestras con papel de nitrazina. El examen digital cervical demuestra una dilatación de 5 cm, con borramiento del 100% y presentación en vértex en estación 0. ¿Cual de los siguientes criterios es el más preciso para decir que se encuentra en la fase activa del trabajo de parto?

a) b) c) d)

Dilatación cervical mayor de tres centímetros Borramiento cervical más de 90% Duración de las contracciones de más de 30 seg Ruptura de membranas

FASES DEL TRABAJO DE PARTO El trabajo de parto se divide en tres fases: Fase 1 ó latente Es llamado así al periodo que sirve para la preparación uterina del parto, ocurre al final del embarazo y va hasta el inicio de las contracciones del trabajo de parto. Los aspectos a destacar en este lapso es el reblandecimiento cervical, el aumento importante en el número de receptores para oxitocina a nivel de las células endometriales, un aumento sustancial en los puentes de unión y el número de conexinas a nivel miometrial y por consiguiente una mayor sensibilidad a los agentes uterotónicos. Fase 2 ó activa Es el lapso que representa el trabajo de parto activo, y se acepta que se inicie cuando existen 3 cm de dilatación y las contracciones uterinas son aptas para producir un avance en el trabajo de parto; se divide en tres periodos: Primer periodo. Se inicia cuando las contracciones uterinas alcanzan la frecuencia, intensidad y duración suficientes para causar borramiento y dilatación del cuello uterino, y finaliza cuando éste se encuentra en completa dilatación. El lapso de tiempo que dura es variable, pero se acepta como normal hasta diez horas en primigrávidas y ocho horas en multigrávidas; pero independientemente de esto, se debe considerar como adecuado si el borramiento y la dilatación cervical son progresivos e ininterrumpidos.

Segundo periodo. Se inicia con una dilatación cervical completa y termina con la expulsión del feto; tiene una duración variable, pero se acepta como normal una hora en pacientes primíparas y 30 minutos en multíparas; y tiene como característica que debe de ser progresivo e ininterrumpido. Tercer periodo. Este comienza inmediatamente finalizada la expulsión fetal y termina con

la expulsión total de la placenta y las membranas corioamnióticas; a este periodo se le conoce también como de “alumbramiento” y es el más corto de los periodos del parto; como norma general se acepta que no debe de extenderse más allá de 10 minutos. Existen algunos autores que incluyen un “cuarto periodo” dentro del trabajo de parto, el cual abarca aproximadamente la hora posterior al alumbramiento, y comprende el lapso de tiempo cuando ocurre la contracción y retracción de las fibras miometriales, así como la trombosis de los vasos adyacentes, lo cual es un efectivo control de la hemorragia del sitio de implantación de la placenta. Fase 3 Este periodo es el que representa el regreso de la mujer a su estado previo al embarazo, y se caracteriza por la involución uterina, la eyección láctea y por último la restauración de la fertilidad; existen estudios que involucran en esta fase a la endotelina-1 y a la oxitocina como substancias responsables de estos cambios postparto.

PROGRAMA DE ACTUALIZACION CONTINUA PARA GINECOLOGÍA Y OBSTETRICIA PAC GO-1 Libro 3 Obstetricia 2005

82.- Se trata de femenino de 32 años de edad que cursa en éste momento con diagnóstico de preclampsia leve, el fármaco de elección que se administra en esta patología es: a) b) c) d)

• •    

Nifedipina. Alfametildopa. Inhibidores de la enzima convertidora de angiotensina. Clonidinas.

Prevenir complicaciones a corto plazo de las mujeres con PA elevada que comprometa el bienestar fetal Cuando la PAS es mayor o igual a 150 mmHg y la PAD mayor o igual a 100 mmHg. El propósito es alcanzar cifras de TA alrededor de 140/90. La medicación antihipertensiva se reserva para los casos en que la PAD ≥ 100 mmHg. Se recomienda continuar el tratamiento antihipertensivo previo al embarazo, exceptuando el uso de IECA. La alfametildopa y la hidralazina vía oral son los fármacos de elección dado su uso extensivo con seguridad y eficacia y sin efectos colaterales para el feto (excepto hidralazina en lupus).

• ALFA METILDOPA 500-2000 MG/DÍA • HIDRALAZINA 50-200 MG/DÍA • LABETALOL 100-400 MG/DIA • ATENOLOL 50-200 MG/DÍA • NIFEDIPINA 10-30 MG/DÍA 1. Aagard K, Belfort M. Eclampsia: Morbility, mortality, and management. Clin Obstet Gynecolol. 2005; 48: 12-23. 2. Oyarzún E. Síndrome hipertensivo del embarazo en Oyarzún E. Ed. Embarazo de alto riesgo. Ediciones Universidad Católica de Chile. Santiago. 1997: 157175. 3. Roberts J, Redman C. Pre-eclamsia: More than pregnancy induced hypertens

83.- Masculino de 7 meses de edad, presenta fiebre alta desde hace 3 días, acompañada de hiperemia faríngea. La fiebre cede al cuarto día de la enfermedad, momento en que aparece un exantema morbiliforme, que desaparece en un plazo de 3 días. El diagnóstico más probable es:

a) b) c) d)

Rubéola Infección por herpes virus humano 6 Reacción medicamentosa a antitérmicos Infección por enterovirus

Nos encontramos ante un caso de exantema súbito tópico (también llamado roséola infantil o 6ª enfermedad). Se trata de una enfermedad exantemática, de mayor incidencia entre los 6 y 12 meses (como el caso de la pregunta) y que se presenta con una frecuencia del 90% antes de los años de vida. No muestra preferencia por ningún sexo y es algo más frecuente al final de la primavera y principios del verano. En el 80-92% de los casos de exantema súbito el agente etiológico es el Herpes Virus Humano. Lo más característico de este período febril es la ausencia de hallazgos físicos suficientes para explicar la fiebre, así como el buen estado general del lactante a pesar de la misma. Al tercer o cuarto día cede la fiebre y aparece una erupción maculosa o maculopapulosa que comienza en tronco y se extiende a los brazos y al cuello. También afecta algo a piernas y cara. La erupción desaparece en unos 3 días, normalmente no dejando descamación ni despigmentación residual. El diagnóstico de este proceso es fundamentalmente clínico, por la edad y la secuencia de las manifestaciones clínicas.

Es característico una BHC realizada en 24 -36h puede mostrar leucocitosis con neutrofilia, pero más allá de 48h aparece el patrón típico d leucopenia con neutropenia absoluta y linfocitosis relativa.

Nelson, Tratado de Pediatría 15ª Ed., págs. 1120-21

84.- Varón de 4 semanas de vida acude a consulta inicialmente con una historia de 2 semanas de vómitos crecientes y escasa ganancia ponderal. Los vómitos, a menudo proyectados, han persistido pese a que se ha cambiado varias veces de fórmula. En las últimas 24 horas tiene vómitos en posos de café. Las heces han sido firmes y el niño defeca un día si y otro no. La exploración física revela un lactante delgado e irritable con abdomen plano. En el hipocondrio derecho se nota una masa firme en forma de aceituna. De los siguientes estudios de laboratorio, el que debe llevarse a cabo inicialmente es: a) b) c) d)

Radiografía de abdomen Valoración de electrólitos en suero. Ecografía abdominal Recuento sanguíneo completo.

La historia y la exploración física son típicas de un diagnostico de estenosis hipertrófica del piloro. En este caso, la consideración mas importante es el estado hidroelectrolítico del paciente, ya que en este trastorno pueden observarse anomalías graves. Es más común una alcalosis metabólica hipoclorémica, y la terapia inicial anterior a la corrección quirúrgica, debe incluir normalización de los electrolitos del suero. En muchos lactantes con estenosis Pilarica se observa hematemesis, normalmente como resultado de una gastritis superficial. Sin embargo, rara vez es de consecuencia hemodinámica. La confirmación del diagnóstico puede realizarse fácilmente en la mayoría de los casos mediante ecografía de abdomen, que revela un canal pilórico alargado y engrosado. La radiografía de abdomen puede mostrar un estomago distendido y lleno de aire. La radiografía con contraste revela un canal pilórico alargado con retraso del vaciamiento gástrico. Un recuento sanguíneo completo no tiene ningún valor en el diagnóstico de la estenosis Pilórica.

LECTURA RECOMENDADA Acta Pediátrica, Mex. 2010;31(2):50-54 INP Estenosis hipertrófica del píloro. Estudio clínico-epidemiológico Dr. Carlos Baeza-Herrera,* Dra. Alín Villalobos-Castillejos,** Dr. Arturo Arcos-Aponte,*** Dr. Javier LópezCastellanos,**** Dr. Luis Manuel García-Cabello*****

85.- Mujer de 39 años la cual presenta amenorrea secundaria de 2 años y medio de evolución. Los niveles reportados de prolactina son de 150ng/ml (normal hasta 20 ng/ml). La resonacia magnética detecta macrotumor de 2,8 cm. de diámetro con expansión lateral izquierda. Sin presentar alteraciones visuales. El tratamiento de elección es: a) Cirugía por tratarse de un macrotumor. b) Tratamiento médico con agonistas dopaminérgicos. c) Somatostatina previa a cirugía. d) Radioterapia hipofisaria previa a cirugía.

Tratamiento Los dopaminérgicos han revolucionado el tratamiento del prolactinoma y virtualmente han dejado fuera a la cirugía; así independientemente del tamaño del adenoma la primera opción terapéutica es la farmacológica.1,2 Con los dopaminérgicos se consigue en poco tiempo restaurar el funcionamiento ovárico y corregir la esterilidad, incluso antes de que se normalice la concentración de prolactina; asimismo se consigue reducir el tamaño del adenoma. Los dopaminérgicos actúan sobre los receptores localizados en las células mamotrópicas de la hipófisis anterior y suprimen la síntesis y secreción de prolactina con la consecuente normalización del eje hipotálamogonadotropico hipotálamogonadotropico- ovárico. La acción dopaminérgica puede ocasionar efectos colaterales indeseables como náusea, hipotensión arterial, constipación nasal, mareo y estreñimiento, los cuales no necesariamente corresponden con la dosis utilizada, pero sí se relacionan con el tipo de dopaminérgico. Está ampliamente documentada la superioridad de la farmacoterapia para el tratamiento de los prolactinomas; además la cirugía es raramente curativa, incluso en el caso de microadenoma.4,5 Cuadro I. Agentes dopaminérgicos que se usan hiperprolactinemia y el prolactinoma. Genérico Comercial Dosis (mg) Bromocriptina Parlodel 2.5-5 diaria Lisurida Dopergin 0.2 diaria Quinagolida Norprolac 25-50 diaria Cabergolina Dostinex 0.5 c/4 días

como

tratamiento

de

la

Los adenomas hipofisarios representan el 10% de todos los tumores intracraneales diagnosticados y 25% de los tumores cerebrales que son intervenidos quirúrgicamente. Los objetivos del tratamiento de un paciente con un adenoma de la pituitaria son: eliminar el efecto de la masa tumoral (compresión sobre estructuras vecinas) disminuir la producción excesiva de hormonas, restaurar la función normal de la pituitaria y evitar la recurrencia. El tratamiento de elección para todos los prolactinomas es con un agonista de la dopamina. La bromocriptina y la cabergolina son efectivas para reducir el tamaño del tumor y para restaurar la función gonadal. El tratamiento quirúrgico debe recomendarse sólo cuando falla el tratamiento médico. Los tumores de la pituitaria productores de hormona de crecimiento son tratados preferentemente mediante adenomectomía transesfenoidal, pero la normalización de los niveles de HC y de IGF-1 ocurre en menos de la mitad de los pacientes con macroadenomas;

por lo tanto, un importante número de pacientes acromegálicos requiere un tratamiento adicional. Los análogos de la somatostatina son en la actualidad los medicamentos que más usados para el control de la acromegalia. En grupos especiales de pacientes, el tratamiento con agonistas de la dopamina y somatostatina parece que suprimen mejor los niveles de HC que cuando se administran esos fármacos en forma separada

Figura 1. Paciente de 20 años quien consultó por amenorrea primaria. El estudio de RMN muestra un macroprolactinoma que invade el seno cavernoso izquierdo y envuelve la carótida del mismo lado.

RMN de control 10 meses después de tratamiento con un agonista dopaminérgico. Corte coronal en T1. No se observa tumor. Tallo hipofisario central y quiasma óptico libre. REFERENCIAS BIBLIOGRÁFICAS Gac Méd Méx Vol. 140 No. 5, 2004 Referencias 1. Schlechte JA. Prolactinoma. N Engl J Med 2003;349:2035-2041. 2. Zárate A, Canales ES, Jacobs LS, Soria J, Daughaday WH. Restoration of ovarian function in patients with the amenorrhea-galactorrhea syndrome after long-term therapy with L-Dopa. Fertil Steril 1973;24:340. 3. Tyson JE, Carter JN, Andreassen B, Huth J, Smith B. Nursing mediated

86.- Masculino de 36 años con poliuria se le practica una prueba de deshidratación. Después de la restricción de líquidos, su osmolalidad urinaria máxima es de 550 mosm/kg y la plasmática es de 295 mosm/kg. Luego de 1 h de aplicar la inyección subcutánea de 5 U de vasopresina acuosa, la osmolalidad urinaria es de 860 mosm/kg. ¿Cuál de los siguientes diagnósticos es probable? a) b) c) d)

Sano Diabetes mellitus Diabetes insípida parcial Diabetes insípida nefrógena

Allen R. M. MMS Medicina Interna. 5ª. Edición. National Medical Series. Mc. Graw Hill. 2006. (capítulo 9 I B 1 c; cuadro 9-2). El paciente tiene diabetes insípida parcial. La respuesta definitiva del enfermo a la inyección de hormona antidiurética (ADH) indica que no produce concentraciones máximas eficaces de ADH después de la restricción de líquidos, y por tanto tiene diabetes insípida parcial o completa. La capacidad para lograr una concentración urinaria normal o casi normal indica que el déficit de ADH sólo es parcial. La respuesta de ADH descarta diabetes insípida nefrógena. La diabetes mellitus, otra causa de poliuria, se diagnostica por las concentraciones de glucosa en orina y sangre más que por estudios del procesamiento renal de agua.

87.- Masculino de 66 años con antecedentes de constipación y dolor abdominal en hipogastrio y fosa ilesa izquierda de manera recurrente. El enema baritado muestra múltiples divertículos de pequeño calibre en el sigmoides. ¿Cuál es el tratamiento más apropiado?

a) b) c) d)

Colonoscopía con biopsia Inhibidores de la bomba de protones Enema con esteroides Dieta alta en residuos

Tratamiento El tratamiento de la diverticulosis intenta reducir el espasmo segmentario. Una dieta rica en residuos es de utilidad y puede suplementarse con preparados de semillas. Las dietas bajas en residuos están contraindicadas. Teóricamente deberían ser de ayuda los antiespasmódicos (p. ej., la belladona); su valor en la práctica es difícil de enjuiciar. Su utilización crónica, especialmente en los ancianos, suele causar efectos secundarios adversos. No está justificada la cirugía en caso de enfermedad diverticular sin complicaciones. La resección con anastomosis del área del intestino afectada en la colitis espástica (una combinación de divertículos, espasmo y diarrea) puede llevar a resultados equívocos.

Deben cateterizarse las arterias mesentéricas superior e inferior. La inyección selectiva de vasopresina controla la hemorragia en el 70% de los Pacientes. En algunos casos la hemorragia recurre a los pocos días y es necesaria la cirugía. Es posible practicar la resección segmentaria si se conoce el punto sangrante; en alrededor del 75% de los Pacientes este punto se encuentra en una zona proximal respecto a la flexura esplénica, aun cuando los divertículos predominen en el lado izquierdo. Si no se puede identificar el punto sangrante está indicada una colectomía subtotal. Los divertículos gigantes deben ser quirúrgicos. Estas lesiones pueden observarse en las radiografías abdominales simples o demostrarse mediante enema de bario. Dado que la posibilidad de infección o perforación es alta, se prefiere la resección del área afectada del colon. Blibliografía Shakelford’s. Surgery of the alimentary tract. 5a. Ed. 2002. Tomo 3. Feldman´s. Gastroeneterology. 2002. Perez. Anatomía y fisiología del hígado. Univ. Católica de Chile. 2005. Bratiz. Serum laboratory test in cirrhosis. Journal of Hepatology. Slovakia. 2005. Paradis. Glycomics. Journal of hepatology. Ireland. Agosto 2005.

88.- Mujer de 65 años que desde hace dos semanas presenta ictericia. Hace dos días se añade confusión mental. A la EF FR 20, FC 110, TA 90/60, Temperatura 39°, no responde a comandos verbales, pero se aleja del dolor que se provoca al palpar el hipocondrio derecho y epigastrio. Los datos clínicos en el caso anterior son sugestivos de:

a) b) c) d)

Colangitis Coledocolitiasis Cáncer de páncreas Cirrosis

Diagnóstico El diagnóstico de colangitis se basa en la asociación de signos y síntomas de infección con los propios de una obstrucción biliar. La presentación clásica es la aparición de dolor en hipocondrio derecho o epigastrio junto a fiebre, generalmente alta, con escalofríos, e ictericia (tríada de Charcot). Cuando se añade confusión mental y shock (sepsis) se denomina Pentada de Reynolds, que se observa con menor frecuencia, pero habitualmente se asocia con una colangitis supurada grave. Sin embargo, la correlación entre la clínica típica, las formas atípicas y la presencia de pus en la vía biliar es pobre y en muchos casos de colangitis faltan algunos de estos rasgos. Algunos enfermos, sobre todo de edad avanzada, pueden tener confusión mental o shock sin fiebre, o existir una leucocitiosis con desviación izquierda como única manifestación de la infección, por lo que se debe sospechar la existencia de una colangitis subyacente, sobre todo en enfermos de edad con algunas de estas manifestaciones. El laboratorio muestra hallazgos de obstrucción biliar con aumento variable de la bilirrubina y enzimas de colestasis. Es habitual encontrar leucocitosis con

desviación izquierda, siendo por lo general las cifras más altas, en torno a 20.000 por mm3 , reflejo de las formas más graves. La ecografía es la técnica de elección para detectar la existencia de obstrucción biliar por su elevada eficacia y versatilidad, pudiéndose detectar también complicaciones de la colangitis, como el absceso hepático. Dentro de la colangitis se pueden establecer unas formas leves, generalmente de buen pronóstico, y unas formas graves, donde se concentra la mayor parte de la mortalidad. Las primeras se manifiestan como cuadros febriles sin signos de afectación sistémica que se autolimitan espontáneamente o bajo tratamiento médico en 24-48 horas. Las formas inicialmente graves son las que asocian confusión mental, hipotensión, shock o fracaso renal. La edad es un factor constante de aumento de la morbimortalidad. Referencias bibliográficas: 1. Bilhartz LE, Horton JD. Gallstone disease and its complications. En: Gastrointestinal and liver diseases. Sleisenger and Fordtran. Filadelfia: WB Saunders Co., 1998; 948-972. 2. Chung-Mau L, Chi-Leung L, Lai ECS, Sheuns-Tat F, Wong J. Early versus delayed laparoscopic cholecystectomy for treatment of acute cholecystitis. Ann Surg 1996; 223: 37-42. 3. Hamy A, Visset J, Likholatnikov D, Lerta F, Gibaud H, Savigny B et al. Percutaneus cholecystostomy for acute cholecystitis in critically ill patients. Surgery 1997; 121: 398401. 4. Harris A, Chong Hen Chang A, Torres-Viera C, Hammett R, Carr-Locke D. Meta-analysis of antibiotic prophylaxis in endoscopic retrograde cholangiopancreatography (ERCP). Endoscopy 1999; 31: 718-724. 5. Hermann RE. Surgery for acute and chronic cholecystitis. Surg Clin North Am 1990; 70: 1.263-1.275. 6. Koo Kp, Thirlby RC. Laparoscopic cholecystectomy in acute cholecystitis. What is the optimal timing for operation? Arch Surg 1996; 131: 540-545. 7. Lai ECS, Mok FPT, Tan ESY, Lo CM, Fan ST, You KT et al. Endoscopic biliary drainage for severe acute cholangitis. N Engl J Med 1992; 326: 1.582-1.586. 8. Marton KI, Doubilet P. How to image de gallbladder in suspected cholecystitis. Ann Int Med 1988; 109: 722-727. 9. Van den Hazel SJ, Speelman P, Tytgat GNJ, Dankert J, Van Leeuwen DJ. Role of antibiotics in the treatment and prevention of acute and recurrent cholangitis. Clin Infect Dis 1994; 19: 279-286. 10. Westphal J-F, Brogard J-M. Biliary tract infections A guide to drug treatment. Drugs 1999; 57: 81-91.

89.- Mujer de 28 años manifiesta que inicialmente presentó un enrojecimiento, sensibilidad y dolor en el borde externo del párpado. Actualmente cursa con orzuelo, usted decide el siguiente tratamiento por ser el de elección:

a) b) c) d)

Compresas tibias y antibióticos tópicos Compresas frías Drenaje Resección amplia

Orzuelo Un orzuelo es una infección, en general provocada por un estafilococo, de una o más de las glándulas que se encuentran en el borde del párpado o por debajo de éste. Se forma un absceso que tiende a romperse y, en consecuencia, genera una pequeña cantidad de pus. El orzuelo a veces se forma al mismo tiempo que la blefaritis o bien como resultado de ésta. Una persona puede tener uno o dos orzuelos en toda su vida, pero otras los desarrollan repetidamente. El orzuelo en general se manifiesta primero con un enrojecimiento, sensibilidad y dolor en el borde externo del párpado. Luego, una pequeña área se torna redondeada y sensible y se hincha. El ojo puede lagrimear, volverse muy sensible a la luz intensa y provocar la sensación de que hay algo en su interior. Generalmente, sólo una parte muy pequeña del párpado se hincha, pero a veces se inflama en su totalidad. En general aparece un diminuto punto amarillento en el centro de la zona hinchada. A pesar de que se recurre a los antibióticos, no parecen ser demasiado útiles en estos casos. El mejor tratamiento consiste en aplicar compresas calientes durante 10 minutos varias veces al día. El calor ayuda a que el orzuelo madure, se rompa y drene. Cuando se forma un orzuelo en una de las glándulas más profundas del párpado, una afección llamada orzuelo interno, el dolor y los demás síntomas suelen ser más intensos. El dolor, el enrojecimiento y la hinchazón suelen aparecer sólo en un área muy pequeña, en general en el borde del párpado. Como esta clase de orzuelo rara vez se rompe por sí solo, el médico puede abrirlo para drenar el pus. Los orzuelos internos suelen ser recurrentes.

90.- Masculino de 66 años que cursa con antecedente de infarto agudo de miocardio, el siguiente grupo de medicamentos son un tratamiento generalmente indicado, por su alta disminución de mortalidad:

a)

Betabloqueadores.

b)

Nitritos.

c)

Anticoagulación oral.

d)

Antiarrítmicos.



Los BB son recomendados en todos los pacientes que han presentado un IAM,

siempre que no tengan contraindicaciones para su uso, y de modo permanente (indefinidamente): clase de recomendación I, nivel de evidencia A. •

Se ha puesto en evidencia que los BB son infrautilizados en esta indicación.



Más de 35.000 pacientes han sido incluidos en estudios postinfarto con BB.



Se ha demostrado una reducción de la mortalidad total, muerte súbita y reinfarto

del orden del 20-25%.

Clase de recomendación

Nivel de evidencia

Para mejorar supervivencia

I

A

Para prevenir reinfarto

I

A

Prevención primaria de la muerte súbita

I

A

Prevención/tratamiento de arritmias ventriculares tardías

IIb

B

Situación clínica/indicación Todos los pacientes sin contraindicaciones, indefinidamente



Los BB siguen estando indicados en el tratamiento de la HTA, aunque existen

críticas a su empleo en el primer escalón. Pese a ello, son sin duda de primera elección si el/la paciente presenta angina, cardiopatía isquémica en general, insuficiencia cardíaca, taquiarritmias, glaucoma o embarazo.



Los BB son fármacos de primera elección en la insuficiencia cardíaca con disfunción

sistólica. Sólo el carvedilol, el bisoprolol o el nebivolol son BB que puedan ser empleados a las dosis disponibles en nuestro medio para dicho tratamiento. •

Todos los pacientes sin contraindicaciones deben recibir BB si presentan

cardiopatía isquémica y, muy especialmente, si existe angina o han presentado un IAM.

ESC Expert consensus document on b-adrenergic receptor blockers. The Task Force on Beta-Blockers of the European Society of Cardiology. Eur Heart J 2004; 25: 1341–1362

91.- Masculino de 52 años que es derivado al servicio de dermatología por presentar prurito en manos y antebrazos, eritema y esfacelación con exacerbación desde hace más de 3 meses. Así mismo máculas en cuello y parte alta del tórax, agregándose edema facial. Antecedentes de trabajar durante 15 años en fábrica de cementos. Exploración física: Placa eritematosa con hiperqueratosis en miembro pélvico izquierdo con huellas de rascado y acompañado de edema. ¿Cual es el diagnóstico más probable? a) b) c) d)

Dermatitis por contacto Psoriasis Urticaria Dermatitis Atópica

Las dermatitis por contacto son consecuencia del contacto de la piel con una sustancia química y pueden producirse por varios mecanismos patogénicos. Con frecuencia se hacen sinónimos dermatitis por contacto y eccema por contacto, puesto que la mayoría de estas reacciones tienen una presentación clínica eccematosa. Sin embargo, sobre todo la dermatitis alérgica por contacto, también es posible que adopten otros patrones, como urticariforme, liquenoide, eritema exudativo multiforme, etc. Según el mecanismo patogénico, pueden distinguirse dermatitis por contacto irritativas, alérgicas, fototóxicas y fotoalérgicas. Debe tenerse en cuenta que una misma sustancia puede ser responsable de dermatitis por contacto por diversos mecanismos. Bibliografía: 1. Bielsa marsol I. Eccemas (I). En: Ferrándiz C. Dermatología Clínica. Madrid: Harcourt 2001: 117-125. 2. Fernández Vozmediano JM, Nieto Montesinos I. Dermatitis por contacto en la infancia. En: Fonseca Capdevila E, ed. Dermatología pediátrica. Madrid: Grupo Aula Médica 1997: 456567.

3. García Bravo B, Rodríguez Rey E. Dermatitis de contacto en la infancia. Piel 2000; 15: 316323. 4. Giménez Camarasa JM, Conde Salazar L, de la Cuadra Oyanguren J, Fernández Vozmediano JM, Fonseca Capdevila E, Giménez Arnau Am, Ortiz de Frutos FJ. Documento de Consenso en Eczema. Madrid: Grupo Aula Médica 1998. 5. Grupo Español para la Investigación de Dermatitis de Contacto. Batería estándar. (En línea). (15-01-2001). Disponible en: http:www.arrakis.es/%7Egeidc/index.htm Dermatitis por contacto La piel es la primera barrera de defensa del organismo y está expuesta a múltiples agresiones del medio ambiente, tales como frío, calor, humedad, jabones, cosméticos y otros múltiples agentes físico-químicos entre los que destacan los medicamentos de uso tópico.

92.- Infant, 33 weeks of gestation presenting in the first day of life, tachypnea, xiphoid retraction, intercostal retractions, nasal flaring, deep moan and cyanosis. On physical examination, crackles are audible bibasilar. The chest radiograph shows reticulonodular infiltrates with air bronchogram. What is the most likely diagnosis? a) b) c) d)

Hyaline membrane disease. Pneumonia Transient tachypnea of the newborn. Meconium aspiration syndrome.

La enfermedad de membrana hialina se debe a un déficit de surfactante, afecta a recién nacidos prematuros, su frecuencia aumenta en hijos de madres diabéticas y en embarazos múltiples. El cuadro clínico se caracteriza por datos de dificultad respiratoria de inicio precoz como: taquipnea, quejido intenso, aleteo nasal, retracciones xifoideas e inter y subcostales y cianosis parcialmente refractaria al oxígeno. Los síntomas progresan hasta alcanzar un máximo al tercer día. En la auscultación aparecen crepitantes en ambas bases. En la radiografía de tórax se presenta un infiltrado reticulonodular con broncograma aéreo en unos pulmones poco ventilados, se pueden presentar atelectasias. En la gasometría se observa hipoxemia, hipercapnia y acidosis respiratoria. El tratamiento consiste en soporte respiratorio, administración endotraqueal de surfactante y antibióticos. Por otro lado, la taquipnea transitoria del recién nacido se presenta en recién nacidos de término, que nacen por cesárea o parto vaginal rápido, debido a un retraso en la absoprción del líquido; manifestándose con un distrés respiratorio de inicio precoz, la auscultación es normal.

Manual pediatría, 7° edición p. 1302. Guía clínica SDR neonatalMarzo 2006

93.- Masculino de 14 años de edad con antecedentes de convivencia con aves, gatos e ingestión muy frecuente de berros. Inicia su padecimiento actual hace dos semanas con fiebre de 39ºC sin predominio de horario, náusea y dolor en hipocondrio derecho. A la exploración física con palidez generalizada y hepatomegalia 3-3-5. Resto de la exploración sin datos patológicos. Laboratorio con BH, con Hb de 11.5, Hto de 40, leucocitos; 16,500, eosinófilos; 38%, linfocitos; 30%, formas inmaduras; 4, plaquetas; 270,000, Fosfatasa alcalina; 280 UI/L. ¿Cuàl es la forma infectante en esta entidad clínica?

a) b) c) d)

Larva filariforma Metacercaria Ooquiste Huevo larvado

El parásito y su ciclo evolutivo

La Fasciola hepatica adulta, es un gusano aplanado en forma de hoja . Mide de 2-5 cm. Es hermafrodita. Los huevos en el exterior y en un ambiente acuático, desarrollan en su interior el miracidio, que al eclosionar, busca al caracol Lymneido (2)., lo penetra y se desarrollan los estadios larvarios de esporoquiste, redia madre, redia hija y cercarias, las que abandonan al caracol y se adosan a las hojas y tallos de las plantas acuáticas, transformándose en metacercaria (forma infectante). FIGURA 1

Esta es ingerida por el animal o el hombre, en el "berro", verduras de tallo corto o en el agua, deja en libertad la forma juvenil en el intestino, la que penetra la pared intestinal, cae a la cavidad peritoneal y se dirige al hígado, perfora la cápsula de Glisson y migra por el hígado hasta las vías biliares, donde finalmente se desarrolla el adulto. El tiempo entre la ingesta de la metacercaria y la localización del adulto es de 2 a 3 meses.

Ciclo biológico de Fasciola hepática (tomado de Atías-Parasitología Médica).

Bibliografía: 1. Leguía, G. 1991. Distomatosis hepática en el Perú. Epidemiología y Control. Segunda edición. U. Nacional Mayor de San Marcos, Lima. 2. Vivar, R. 1987. Aportes al conocimiento de algunos Tremátodes en el Perú. Tesis. UPRP, Lima. 3. López, M; Zerda, K; Náquira, C. y Guerra, H. Major Fasciola hepatica antigens are mainly localized within the digestive tube of the adult. Parasitología al Día. 1997; 21:104-108. 4. Caprón, A; Biguet, G; Tran Van Ky e Rose G. Posibilities nouvelles dans le diagnostic immunologie de la distomatose humaine a Fasciola hepatica. Press. Med. 1964;72:31033107. 5. Córdova, M; Herrera, P; Nopo, L; Bellatin, J; Náquira, C; Guerra, H. and Espinoza, J. Fasciola hepatica cysteine proteinases: immunodominant antigens in Human Fascioliasis. Am. J. Trop. Med. Hyg., 1997; 57(6):660-666. 6. Córdova, M; Reátegui, L. and Espinoza J.R. Immunodiagnosis of human fascioliasis with Fasciola hepatica cystine proteinases. Trans. Roy. Soc. Trop. Med. Hyg. 1999; 93:54-57.

94.- Femenino de 43 años portadora de DM tipo II, e HTAS, es ingresada al servicio de Medicina Interna por cetoacidosis. Posterior a su recuperación metabólica inicia con fiebre, cefalea, dolor facial, disminución del nivel de conciencia y enrojecimiento nasal con lesión negruzca en fosa nasal derecha. El diagnóstico más probable en ésta paciente es:

a) b) c) d)

Endocarditis por S. aureus. Carcinoma epidermoide. Infección por M. tuberculosis. Infección por Mucor.

Mucormicosis es el nombre común dado a varias diferentes enfermedades causadas por hongos de la orden de los Mucorales. Muchas diferentes especies han sido implicadas como agentes de síndromes clínicos similares.

Los Mucoraceos son hongos que se encuentran en todas partes y son comunes habitantes de materia en descomposición. Por ejemplo, Rhizopus sp. Frecuentemente puede ser recuperado de pan mohoso. Por su rápido crecimiento y prolífica capacidad de formar esporas, inhalación de conidias debe ser una experiencia cotidiana. La presencia de esporas Mucorales en cinta adhesiva no estéril se demostró fue la fuente de mucormicosis cutánea primaria. Aun cuando estos hongos crecen en muchos nichos ecológicos, la infrecuencia de enfermedad debida a estos organismos da fe de su baja potencial virulencia en el huésped humano. En contraste a la amplia distribución de este hongo, la enfermedad en humanos esta limitada, en muchos casos a población con severo inmunocompromiso, diabetes mellitus o trauma.

Manifestaciones clínicas En 1973 Meyer y Armstrong12 categorizaron las diferentes presentaciones clínicas de mucormicosis, considerando el órgano involucrado y haciendo una división en seis entidades: rinocerebral, cutánea, gastrointestinal, pulmonar, diseminada y formas misceláneas. Existe una fuerte asociación entre la entidad subyacente y la forma de presentación. La mucormicosis rinocerebral se presenta más frecuentemente en pacientes diabéticos con acidosis, y debido al advenimiento de quimioterapias cada vez más potentes, se aprecia con más frecuencia en pacientes leucémicos con neutropenia prolongada, en aquellos que reciben múltiples esquemas de antibióticos y corticoesteroides, así como en pacientes con trasplantes de órganos. En pocas ocasiones se ha documentado esta forma invasora en personas sin enfermedades subyacentes. La forma pulmonar puede presentarse en pacientes con leucemia y neutropenia; la mucormicosis gastrointestinal se observa más comúnmente en pacientes con desnutrición calórico-proteica y en prematuros, y la diseminada en pacientes con déficit inmunológico grave como trasplantados, leucémicos y nefrópatas tratados con deferoxamina.1,2 La infección rinocerebral es la presentación más frecuente y característica de mucormicosis, siendo la rinoorbitaria y la mucormicosis paranasal estadios tempranos de esta.

La infección generalmente inicia en senos paranasales o paladar duro, y se extende a senos adyacentes con diseminación a través de senos etmoidales y zona retroorbitaria; puede tener acceso al cerebro a través del ápex orbitario, lámina cribosa y por vía vascular. Una costra necrótica sangrante en paladar o en mucosa nasal y un drenaje ocular de pus negruzco orientan al diagnóstico. Puede haber una progresión rápida y presentarse el deceso en pocos días o ser indolente si la enfermedad subyacente se logra controlar. Inicialmente puede haber dolor facial, cefalea, fiebre y algún grado de celulitis orbitaria, conforme progresa la invasión de la órbita la pérdida de la función del II, III, IV y VI nervios craneales puede ocurrir, así como también puede haber pérdida de la función de músculos extraoculares, proptosis, quemosis progresiva, congestión nasal, epistaxis y letargia. La disfunción de nervios craneales, especialmente el V y VII, ocurre de manera tardía manifestando ptosis y midriasis, lo cual es un factor pronóstico grave, en caso de trombosis retiniana, hay pérdida de la visión y puede haber afección intraocular. El compromiso cerebral es la complicación más seria y puede manifestarse como: infarto, absceso, trombosis del seno cavernoso, hematoma subdural y necrosis del lóbulo frontal.2 Otras complicaciones son trombosis de arteria carótida interna y vena yugular, e incluso se ha reportado infarto de miocardio por oclusión de coronaria y aborto séptico. Puede haber presentaciones crónicas y secuelas tardías a pesar del tratamiento aparentemente satisfactorio por lo cual siempre debe realizarse un seguimiento del paciente a largo plazo

BIBLIOGRAFIA:

• Mayo Clinic. Pulmonary Diseases • Mandele?Douglas. Infections Diseases • Pennington. Respiratory infections, Diagnosis and Management • Ronald B. George., Richard W. Light, Michael A. Matthay. Chest Medicine 3era. Edición. • Scott E Davis. Neumonía Micótica. Clínicas médicas de Norteamérica 1997; 5: 1092 ,1094.

95.- Al existir la sospecha de líquido libre en la cavidad peritoneal y después de analizar los hallazgos de manera conjunta de las placas de abdomen y en el ultrasonido abdominopélvico, ¿Dónde se deberán de buscar los cambios de densidad de manera inicial? a) b) c) d)

Infradiafragmático Transcavidad de los epiplones Correderas parieto-cólicas Fondos de saco de Douglas y de Morrison

Pedrosa C, Casanova R. Diagnóstico por imagen. Mc Graw Hill 2001 p. 209, 210.

La diseminación del líquido intraperitoneal está producida por la gravedad y la presión hidrostática secundaria al movimiento diafragmático.la pelvis es la parte más dependiente de la cavidad peritoneal, su capacidad es de 300 cc, a partir de lo cual el líquido asciende por las correderas parietocólicas. El líquido pélvico desplaza las asas intestinales que rellenan los espacios pélvicos, el íleon del lado derecho y el colon sigmoides en el izquierdo, lo que resulta de la parición de una densidad homogénea en al pelvis menor, en contra del patrón poco homogéneo de las asas intestinales que tienen gas y líquido. En la pelvis femenina el líquido se acumula en los recesos laterales de la vejiga y al útero, y en el hombre puede verse en la fosa vesicorrectal y los recesos pélvicos laterales. En el ultrasonido el diagnóstico de pequeñas cantidades de líquido debe hacerse fundamentalmente en el saco de Douglas, en la bolsa de Morrison y en el receso yuxtaesplénico.

96.- Al realizar la exploración clínica y colocar un diapasón que está vibrando frente al conducto auditivo del oído que queremos explorar (conducción aérea) y apoyando después sobre la mastoides (conducción ósea), podemos de modo sencillo y en la consulta, distinguir entre sordera nerviosa (alteración en la cóclea o nervio auditivo) y sordera de conducción (trastorno en el sistema de transmisión tímpano-osicular). ¿Cuál de estas afirmaciones es correcta para un paciente que presenta una sordera de conducción? a) b) c) d)

La percepción del sonido es mejor por vía ósea que por vía aérea. La percepción del sonido es igual por vía aérea que por vía ósea. La percepción del sonido es mejor por vía aérea que por vía ósea. La percepción del sonido es indistinguible tanto por vía aérea como ósea.

MEDICIÓN CLÍNICA DE LA AUDICIÓN La valoración audiológica mínima debe incluir la determinación de los umbrales de conducción aérea y ósea, el umbral de recepción y la discriminación del lenguaje, una timpanometría y pruebas de reflejos acústicos, que incluyan la prueba de deterioro de los reflejos. La información obtenida por medio de estas técnicas permite determinar si hace falta una mayor diferenciación entre la sordera neural y la sensorial. La audición por conducción aérea se valora presentando un estímulo acústico mediante auricular o altavoces. Una sordera o elevación del umbral de audición detectada por esta prueba se puede deber a defectos en cualquier parte del aparato auditivo: pabellón auricular, conducto auditivo, oído medio o interno, VIII par craneal o vías auditivas centrales.

La audición por conducción ósea se valora colocando una fuente sonora (el vibrador de un audiómetro o un diapasón) en contacto con la cabeza. El sonido produce una vibración a través del cráneo, que alcanza las paredes óseas de la cóclea y estimula directamente el oído interno. La audición por conducción ósea no atraviesa los oídos externos y medio y permite valorar la integridad del oído interno, del octavo nervio craneal y de las vías auditivas centrales. Si aumenta el umbral de conducción del aire y el umbral de conducción ósea es normal, la sordera es de conducción, mientras que si ambos umbrales aumentan por igual es de tipo neurosensorial. Existen algunas formas de sordera mixtas con componente neurosensorial y de conducción, en las que aumentan ambos umbrales, aunque el de la conducción aérea es más significativo. Manual Merck 10. Edición en Español

Editors of The Merck Manual

Robert S. Porter, MD, Editor-in-chief Justin L. Kaplan, MD, Senior Assistant Editor Editorial Board of The Merck Manual

97.- Paciente masculino de 3 años de edad que acude a urgencias con una historia de secreción nasal purulenta y de mal olor unilateral desde hace 5 días. El diagnóstico más frecuente es: a) b) c) d)

Atresia de coanas unilateral. Rinitis crónica por Rinovirus. Fibrosarcoma del correte nasal. Cuerpo extraño intranasal.

Una de las patologías más frecuentes que enfrenta el otorrinolaringólogo en su práctica médica, es la presencia de cuerpos extraños animados o inanimados en vías aéreodigestivas superiores. La mayoría de las veces su tratamiento (extracción) no reviste mayor importancia sobre todo cuando no han sido manipulados por médicos de primer contacto, pero en otras, se convierte en un procedimiento muy difícil de realizar, ya sea por las complicaciones que se produjeron o por la presencia del mismo. En la población infantil es más frecuente por la curiosidad propia de esa edad y la tendencia a colocarse una variedad de elementos en las fosas nasales. A veces existen ciertas dificultades en el diagnóstico, pero la tríada sintomática de obstrucción nasal, rinorrea unilateral y halitosis, nos sugiere la posibilidad de un cuerpo extraño hasta que no se demuestre lo contrario. Independientemente de su naturaleza animada o inanimada, éstos pueden alojarse y/o impactarse en distintos niveles.

El tratamiento electivo es la extracción por vías naturales de los mismos; en los niños que no colaboran es necesario recurrir a la anestesia general. 1. “Manual de Urgencias en Otorrinolaringología.” Manuel Tomás Barberán, Julio GarcíaPolo Alguacil, Guillermo Til Pérez

98.- Se reporta ultrasonografía renal donde usted observa una masa, con presencia de una pared delgada bien delimitada, en el que se interrumpe el contorno renal, sin presencia de ecos en su interior y refuerzo posterior. El diagnóstico más probable es:

a) Absceso renal. b) Hipertrofia de columna de Bertin. c) Carcinoma renal. d) Quiste renal simple. Los criterios para el diagnóstico ultrasonográfico de un quiste simple renal son los siguientes: 1.- Forma ovoide o esférica 2.-Ausencia de ecos internos (quiste anecoico) 3.-Presencia de una pared delgada y lisa bien definida que lo separa del parénquima. 4.-.El refuerzo acústico más allá de la pared posterior del quiste es proporcional a su contenido líquido. 5.-Se observa una banda estrecha de forma acústica por fuera del borde externo. Si se cumplen con todos estos criterios no es necesario realizar más estudios para diagnosticar la masa. Sin embargo, ocasionalmente la ecografía puede no resultar la técnica más idónea (por ejemplo cuando hay calcificación de la pared del quiste, éste es hemorrágico o existen múltiples quistes). -Brenner and rector The Kidney 2 volúmenes 2004 Saunders 2525 pag.

99.- Se trata de primigesta de 26 años con control prenatal regular, cursa con ruptura de membranas a las 30 semanas de gestación, se interrumpe el embarazo por cesárea. Es probable que el recién nacido prematuro disminuya la posibilidad de presentar enfermedad de membranas hialinas por la siguiente medicación materna: a) b) c) d)

Antibioticos IV Oxitocina IV Sulfato de magnesio IV Esteroides IM

Uso prenatal, un pilar en neonatología

Por su efecto en la maduración fetal, los corticoesteroides se han utilizado prenatalmente desde hace más de tres décadas. Liggins y Howie en 1972, reportaron por primera vez los efectos benéficos de los esteroides, administrados prenatalmente, en la maduración pulmonar y en la disminución de la incidencia de SDR. Desde ese primer reporte a la fecha, se han descrito otros efectos positivos. Su uso "adecuado" y oportuno ha demostrado tener beneficio en el recién nacido pretérmino.Ahora se sabe que la administración de esteroides a la madre embarazada se ha asociado a disminución en la incidencia de SDR, hemorragia intraventricular, displasia bronco–pulmonar y mortalidad neonatal. Se ha observado que el máximo beneficio en el recién nacido se obtiene de 24 horas a siete días después de la administración materna de esteroides. Sin embargo, aún antes de 24 horas de administrados, se ha reportado beneficio. Aunque los cambios bioquímicos generados con los esteroides desaparecen después de siete días, los cambios estructurales persisten. Algunos estudios clínicos han demostrado efecto benéfico de esteroides aún después de siete días de administrados. Un metanálisis de 12 estudios, realizado por Crowley y col. en 1990, demostró que el uso prenatal de esteroides disminuía la incidencia de SDR en aproximadamente 50%. Es claro que el efecto más importante se encuentra en los fetos menores de 34 semanas de edad gestacional; sin embargo, también se ha demostrado disminución en la incidencia de SDR en neonatos de mayor edad gestacional.18 Probablemente el efecto benéfico más importante asociado al uso prenatal de esteroides sea la disminución en la mortalidad neonatal. Crowley y col. demostraron en el metanálisis, con más de 3 000 neonatos, que ésta disminuía considerablemente (razón de momios 0.59, intervalo de confianza =0.47–0.75). La disminución en mortalidad se ha detectado aún en pacientes menores de 800 g. Bol. Med. Hosp. Infant. Mex. v.62 n.5 México sep. /oct. 2005 Esteroides en Neonatología: entusiasmo, uso, abuso y desuso. ¿Dónde está el justo medio? Steroids in neonatology: enthusiasm, use, abuse and disuse. Is there an equilibrium? 1.

2.

3. 4.

5.

Liggins GC, Howie RN.A controlled trial of antepartum glucocorticoid treatment for prevention of the respiratory distress syndrome in premature infants. Pediatrics. 1972; 50: 515–25. [ Crowley P, Chalmers I, Keirse MJNC. The effects of corticosteroid administration before preterm delivery:An overview of the evidence from controlled trials. Br J Obstet Gynaecol. 1990; 97: 11–25. [ Van Marter LJ, Leviton A, Kuban KCK. Maternal glucocorticoid therapy and reduced risk of bronchopulmonary dysplasia. Pediatrics. 1990; 86: 331 –6. Morales WJ, Diebel D, Lazar AJ, Zadrozny D.The effect of antenatal dexamethasone administration on the prevention of respiratory distress syndrome in preterm gestations with premature rupture of membranes. Am J Obstet Gynecol 1986; 154:591–5. Gamsu HR, Mullinger BM, Donnai P, Dash CH. Antenatal administration of betamethasone to prevent respiratory distress syndrome in preterm infants: Report of a UK multicentre trial. Br J Obstet Gynaecol. 1989; 96:401–10.

6. Young BK, Klein SA, Katz M. Intravenous dexamethasone for prevention of neonatal respiratory distress: A prospective controlled study. Am J Obstet Gynecol. 1980; 138:203–9. 7. Doyle LW, Permezel MJ, Kitchen WH. Is there a lower limit for birth– weight/gestational age and antenatal steroid therapy? Aust NZ J Obstet Gynaecol. 1992; 32: 193–5.

100.-Femenino de 5 años, es atendida en consulta para control de infecciones de vías urinarias comprobada. Antecedentes: Infección de vías urinarias en 6 ocasiones desde el nacimiento, actualmente con mejoría acentuda después de que recibió tratamiento con antibióticos. Exploración física: actualmente asintomática. El siguiente paso para confirmar el diagnóstico en este paciente es: a) b) c) d)

Examen general de orina. Ultrasonido abdominal. Ultrasonido renal y de vias urinarias. Urocultivo.

Revisión sistemática bien realizada con poco riesgo de sesgo. NICE 2007 En el paciente pediátrico con infección de vías urinarias comprobada, se debe investigar la presencia de alteraciones anatómicas y funcionales. Evidencia compuesta por estudios clasificados como 1+ y con gran consistencia entre ellos. Baumer JH, 2007

Los síntomas y signos más comunes en menores de cinco años por los que acuden a urgencias con el primer episodio de IVU son: fiebre 80%, irritabilidad 52%, anorexia 49%, malestar 44%, vómito 42%, diarrea 21%. Los síntomas menos comunes (en menos de 20%): disuria, orina fétida, dolor abdominal, frecuencia y hematuria.

La presencia de fiebre >38ºC, bacteriuria y dolor lumbar sugiere pielonefritis, mientras que la presencia de síntomas urinarios como disuria asociada a bacteriuria, pero no a síntomas sistémicos, sugiere cistitis o IVU baja.

En niños mayores de tres años puede utilizarse de primera instancia la búsqueda de nitritos, estearasa leucocitaria, estudio microscópico y urocultivo. En niños menores de tres años con sospecha de IVU debe tomarse de inmediato la muestra para urocultivo. La búsqueda de nitritos y estearasa leucocitaria solo se realizará cuando el estudio microscópico sea imposible de realizar.

Prevención, diagnóstico y tratamiento de la infección de vías urinarias no complicada en menores de 18 años en el primero y segundo nivel de atención de atención México:

Secretaría de Salud; 2008. M

La infección urinaria (ITU) se define como la invasión, multiplicación y colonización del tracto urinario por gérmenes que con mayor frecuencia provienen de la región perineal. La confirmación diagnóstica está dada por un urocultivo con un recuento de colonias superior a 1 ufc/ml si la muestra es tomada por punción vesical, superior a 10.000 ufc/ml si es tomada por bolsa recolectora o sondeo, y mayor de 100.000 ufc/ ml si es tomada durante la mitad de la micción (segundo chorro)1. Bacteremia se define como la presencia de bacterias viables en sangre confirmada por hemocultivos. Debe corresponder a un sólo germen, el cual debe ser el mismo encontrado en el urocultivo. Dada la alta prevalencia de alteraciones anatómicas y funcionales del riñón y vías urinarias que presentan los niños con ITU, se debe realizar estudio imagenológico inicial con ecografía renal y vesical, y uretrocistografía, independiente de su edad y sexo, excepto en la mujer mayor de 5 años con un primer episodio de ITU baja2. Se define malformación de la vía urinaria como una alteración en las diferentes etapas de la nefrogénesis, la cual comienza en la octava semana de la gestación, donde surge la yema ureteral derivada del conducto mesonéfrico de Wolf, penetrando el blastema metanéfrico para desarrollar los riñones fetales2,3 .

Referencias: 1.

2. 3. 4.

5. 6. 7.

Garin, Eduardo; Olavarría, Fernando; García, Víctor Nieto; Valenciano Blanca; Campos, Alfono and Young, Linda. Clinical Significance of Primary Vesicoureteral Reflux and Urinary Antibiotic Prophylaxis After Acute Pyelonephritis: A Multicenter, Randomized, Controlled Study, Pediatrics 2006;117;626-632. Glassberg, K. Special Article: Annual Meeting of the Section on Pediatric Urology. Pediatrics , 1988;81: 588-594. Gordillo, P. Malformaciones Urológicas y Renales. En: Nefrología Pediatrica. México. 1999,.2ª edición Mosby. Páginas.133 - 157 . American Academy of Pediatrics, Committee on Quality Improvement, Subcommittee on Urinary Tract Infection. Practice Parameters: The Diagnosis, Treatment, and Evaluation of the Initial Urinary Tract Infection in Febrile Infants and Young Children. Pediatrics , 1999;103:843-852 Hodson, C: The radiologic diagnosis of pyelonephritis. Proc R soc Med 1959;52:669. Winberg, J; Andersen, H.; Bergstrom, T. et al. Epidemiology of symptomatic urinary tract infection in childhood . Acta pathol scand suppl 1974;252:1. Salas, Paulina; Alvarez, Enrique y Saieh, Carlos. Pautas de diagnóstico y tratamiento en infección urinaria en niños. Documento de la Rama de Nefrología de la Sociedad Chilena de Pediatría. Rev. chil. pediatr. , jun. 2003, vol.74, nº 3: 311-314.

8. Cavagnaro, Felipe. Infección urinaria en la infancia. Rev. chil. infectol. , jun. 2005, vol.22, no.2, p.161-168.

View more...

Comments

Copyright ©2017 KUPDF Inc.
SUPPORT KUPDF